Gastrointestinal system Flashcards

1
Q

The acute abdomen DR DEAC PIMP

A

D: The ‘acute abdomen’ is defined as a sudden onset of severe abdominal pain of less than 24 hours duration. It has a large number of possible causes and so a structured approach is required.

The initial assessment should attempt to determine if the patient has an acute surgical problem that requires immediate and prompt surgical intervention, or urgent medical therapy.

R:

D:

E:

A:

C:

How well did you know this?
1
Not at all
2
3
4
5
Perfectly
2
Q

Acute abdomen assessment and management.

A

The first decision when you first see any patient is “Are they critically unwell?”. A 10-second assessment of their clinical state can be made by a general look (the “end-of-bed-o-gram”), their observations, and whether they can talk to you.

If they are critically unwell, give oxygen, start suitable initial steps, and call for help early before going into detail on the history and examination.

Presentations Requiring Urgent Surgery”
1. Bleeding
The most serious cause of intra-abdominal bleeding is a ruptured abdominal aortic aneurysm, which requires swift referral to the vascular team and immediate surgical intervention.

Other common causes usually involve a slower rate of bleeding, but with urgent surgery still required, include ruptured ectopic pregnancy, bleeding gastric ulcer, and trauma.

These patients will typically go into hypovolemic shock. Clinical features include tachycardia and hypotension, pale and clammy on inspection, and cool to touch with a thread pulse.

  1. Perforated Viscus
    Peritonitis is the inflammation of the peritoneum, and a generalised peritonitis is most commonly caused by perforation of an abdominal viscus.

The causes of perforation are broad but include peptic ulceration, small or large bowel obstruction, diverticular disease, and inflammatory bowel disease.

Patients with a generalised peritonitis present with some characteristic features:

Patients often lay completely still, not to move their abdomen, and look unwell
This is especially important when compared to a renal colic, whereby patients are constantly moving and cannot get comfortable.
Tachycardia and potential hypotension
A completely rigid abdomen with percussion tenderness
Involuntary guarding – the patient involuntarily tenses their abdominal muscles when you palpate the abdomen
Reduced or absent bowel sounds, suggesting the presence of a paralytic ileus
3. Ischaemic Bowel
Any patient who has severe pain out of proportion to the clinical signs has ischaemic bowel until proven otherwise. They are often acidaemic with a raised lactate and physiologically compromised.

Patients will often complain of a diffuse and constant pain, however the examination can often otherwise be unremarkable. Definitive diagnosis is via a CT scan with IV contrast, with early surgical involvement. Ischaemic bowel is discussed in more detail here.

Presentations That Are Less Acute
Colic
Colic is an abdominal pain that crescendos to become very severe and then goes away completely. This is most typically seen in either ureteric obstruction or bowel obstruction.

Biliary ‘colic’ is not a true colic as the pain does not go away completely, instead getting periodically better and worse (colloquially termed ‘waxes and wanes’).

Peritonism
Peritonism (not peritonitis) refers to the localised inflammation of the peritoneum, usually due to inflammation of a viscus that then irritates the visceral (and subsequently, parietal) peritoneum.

This leads to patients stating that their abdominal pain starts in one place (irritation of the visceral peritoneum) before localising to another area* (irritation of the parietal peritoneum) or becoming generalised.

*The classic example of this is acute appendicitis, with the pain migrating from the umbilical region to the right iliac fossa.

The location of abdominal pain is perhaps the most useful initial feature to help narrow your differential. These can be classified based upon quadrant or region affected, as shown in Fig 2.

It must be remembered to always consider extra-abdominal organs as the cause for abdominal pain, including cardiac, respiratory and gynaecological or testicular conditions.)
On next fc is abdominal pain and locations.

How well did you know this?
1
Not at all
2
3
4
5
Perfectly
3
Q

Acute abdomen assessment and management.

A

The first decision when you first see any patient is “Are they critically unwell?”. A 10-second assessment of their clinical state can be made by a general look (the “end-of-bed-o-gram”), their observations, and whether they can talk to you.

If they are critically unwell, give oxygen, start suitable initial steps, and call for help early before going into detail on the history and examination.

Presentations Requiring Urgent Surgery”
1. Bleeding
The most serious cause of intra-abdominal bleeding is a ruptured abdominal aortic aneurysm, which requires swift referral to the vascular team and immediate surgical intervention.

Other common causes usually involve a slower rate of bleeding, but with urgent surgery still required, include ruptured ectopic pregnancy, bleeding gastric ulcer, and trauma.

These patients will typically go into hypovolemic shock. Clinical features include tachycardia and hypotension, pale and clammy on inspection, and cool to touch with a thread pulse.

  1. Perforated Viscus
    Peritonitis is the inflammation of the peritoneum, and a generalised peritonitis is most commonly caused by perforation of an abdominal viscus.

The causes of perforation are broad but include peptic ulceration, small or large bowel obstruction, diverticular disease, and inflammatory bowel disease.

Patients with a generalised peritonitis present with some characteristic features:

Patients often lay completely still, not to move their abdomen, and look unwell
This is especially important when compared to a renal colic, whereby patients are constantly moving and cannot get comfortable.
Tachycardia and potential hypotension
A completely rigid abdomen with percussion tenderness
Involuntary guarding – the patient involuntarily tenses their abdominal muscles when you palpate the abdomen
Reduced or absent bowel sounds, suggesting the presence of a paralytic ileus
3. Ischaemic Bowel
Any patient who has severe pain out of proportion to the clinical signs has ischaemic bowel until proven otherwise. They are often acidaemic with a raised lactate and physiologically compromised.

Patients will often complain of a diffuse and constant pain, however the examination can often otherwise be unremarkable. Definitive diagnosis is via a CT scan with IV contrast, with early surgical involvement. Ischaemic bowel is discussed in more detail here.

Presentations That Are Less Acute
1. Colic
Colic is an abdominal pain that crescendos to become very severe and then goes away completely. This is most typically seen in either ureteric obstruction or bowel obstruction.

Biliary ‘colic’ is not a true colic as the pain does not go away completely, instead getting periodically better and worse (colloquially termed ‘waxes and wanes’).

  1. Peritonism
    Peritonism (not peritonitis) refers to the localised inflammation of the peritoneum, usually due to inflammation of a viscus that then irritates the visceral (and subsequently, parietal) peritoneum.

This leads to patients stating that their abdominal pain starts in one place (irritation of the visceral peritoneum) before localising to another area* (irritation of the parietal peritoneum) or becoming generalised.

*The classic example of this is acute appendicitis, with the pain migrating from the umbilical region to the right iliac fossa.

The location of abdominal pain is useful feature to help narrow your differential. These can be classified based upon quadrant or region affected,

It must be remembered to always consider extra-abdominal organs as the cause for abdominal pain, including cardiac, respiratory and gynaecological or testicular conditions.)
On next fc is abdominal pain and locations.
Importantly, there are non-general surgical causes of abdominal pain that must not be missed, including testicular torsion, ruptured ectopic pregnancy, diabetic ketoacidosis, and myocardial infarction.

Investigations:
Laboratory Tests
The investigations in all cases of the acute abdomen share the same generic outline:

Urine dipstick – for signs of infection or haematuria Include a pregnancy test for all women of reproductive age.
ABG – useful in bleeding or septic patients, especially for the pH, pO2, pCO2, and lactate for signs of tissue hypoperfusion, as well as a rapid haemoglobin.
Routine bloods – FBC, U&Es, Liver Function, CRP, amylase.
Consider measuring serum calcium in suspected pancreatitis.
Do not forget a group & save if the patient is likely to need surgery soon.
Blood cultures – if considering infection as a potential diagnosis
Note: Any amylase 3x greater than the upper limit is diagnostic of pancreatitis. Any raised value lower than this may also be due to another pathology, such as perforated bowel, ectopic pregnancy, or diabetic ketoacidosis (DKA)

Imaging
In the emergency setting, every patient with abdominal pain should have an ECG to exclude myocardial infarction.

Other imaging modalities that may be initially requested include:

Ultrasound:
Kidneys, ureters and bladder (‘KUB’) – for suspected renal tract pathology
Biliary tree and liver – for suspected gallstone disease
Ovaries, fallopian tubes and uterus – for suspected tubo-ovarian pathology
Radiological:
An erect chest X-ray (eCXR) – for evidence of bowel perforation (Fig. 3)
CT imaging, often best discussed with a senior depending on the suspected underlying diagnosis.

Management
The definitive management of acute abdomen depends largely on the cause. However, a good initial management plan includes the same key points – regardless of the underlying aetiology.

These include admission, IV access, NBM, analgesia +/- antiemetics, imaging (as discussed above), VTE prophylaxis, urine dip, bloods (as discussed above). Consider a urinary catheter and/or nasogastric tube if necessary. Start IV fluids and monitor fluid balance.

How well did you know this?
1
Not at all
2
3
4
5
Perfectly
4
Q

Central abdominal pain

A
Early appendicitis
SBO
Acute gastritis
Acute pancreatitis
Ruptured AAA
Mesenteric thrombosis
How well did you know this?
1
Not at all
2
3
4
5
Perfectly
5
Q

Epigastric pain

A
DU / GU
Oesophagitis
Acute pancreatitis
AAA
Gastritis
How well did you know this?
1
Not at all
2
3
4
5
Perfectly
6
Q

RUQ pain

A
Gallbladder disease
DU
Acute pancreatitis 
Pneumonia
Subphrenic abscess
How well did you know this?
1
Not at all
2
3
4
5
Perfectly
7
Q

LUQ pain

A
GU
Pneumonia
Acute pancreatitis 
Spontaneous splenic rupture
Subphrenic abscess
How well did you know this?
1
Not at all
2
3
4
5
Perfectly
8
Q

Suprapubic pain

A
Acute urinary retention
UTIs 
Cystitis 
PID
Ectopic pregnancy 
Diverticulitis
How well did you know this?
1
Not at all
2
3
4
5
Perfectly
9
Q

RIF pain

A
Acute appendicitis
Mesenteric adenitis (young)
Perf DU
Diverticulitis 
PID
Salpingitis
Ureteric colic 
Meckel’s diverticulum 
Ectopic pregnancy 
Crohn’s disease
Biliary colic (low-lying gall bladder)
How well did you know this?
1
Not at all
2
3
4
5
Perfectly
10
Q

Loin pain

A
Muscle strain
UTIs
Renal stones
Pyelonephritis 
AAA
How well did you know this?
1
Not at all
2
3
4
5
Perfectly
11
Q

LIF pain

A
Diverticulitis
Constipation
IBS
PID
Rectal Ca
UC
Ectopic pregnancy
How well did you know this?
1
Not at all
2
3
4
5
Perfectly
12
Q

Abdominal pain prevalence

A

Community prevalence 15-20%
Incidence increases in the elderly
25% consult their GP
6-10% of A/E visits

How well did you know this?
1
Not at all
2
3
4
5
Perfectly
13
Q

Guarding

A

When you pa;pate patients abdome, they tense their anterior abdominal wall. Guaarding can be voluntary or involuntary. If involuntary, patient has peritonitis, automatically they have an acute abdomen and they are potentially very seriously ill. Voluntary guarding is more subtle.

How well did you know this?
1
Not at all
2
3
4
5
Perfectly
14
Q

Abdominal pain management

A

A/B: airway/breathing, may have low oxygen sats.
-Supplemental oxygen will help them calm down.
C: Circulation
-IV access
-fluid administration
-Urinary catherisation
Disability: antiematics, abakgesics, antibiotics, radiology, re-evaluation with results (BM, blood tests, ABG, ECG, radiology).

How well did you know this?
1
Not at all
2
3
4
5
Perfectly
15
Q

Complications with pancreatitis

A

Inflamed pancreas, that inflammation can have a knock on effect on the splenic artery as it wiggles its way behind the spleen. So you can develop pseudoaneurysm of splenic artery which can rupture.

How well did you know this?
1
Not at all
2
3
4
5
Perfectly
16
Q

Gastrointestinal bleeding

A

D: Gastrointestinal (GI) bleeding is a symptom of conditions that damage the wall GI tract. GI bleeding is categorized into upper GI bleeding (UGIB) and lower GI bleeding (LGIB) depending on the source of the bleeding relative to the ligament of Treitz. In the majority of cases, bleeding is localized in the esophagus, stomach, or duodenum (UGIB). LGIB may occur in the rectum, colon, jejunum, and, in rare cases, the ileum. Gastric and duodenal ulcers are the most common causes; angiodysplasia, inflammatory diseases, and carcinomas can also cause GI bleeding. Depending on the source of the bleeding and how long the blood remains in the digestive tract, clinical symptoms may include vomiting blood (hematemesis), tarry black stool (melena), and fresh blood in the stool (hematochezia). Diagnosis includes evaluation of blood loss (e.g., hematocrit) and localization of the source of bleeding (e.g., endoscopy). Hospitalization is essential to monitor for signs of hemodynamic instability and shock caused by anemia and severe blood loss. The source of bleeding can often be located and treated simultaneously during endoscopy with injection therapy (e.g., epinephrine) or ligation.

R:

D: peptic ulcer disease, oesophagitis, cameron lesion, varices, angioma, tumours: oesophageal cancer, gastric cancer, traumatic or iatrogenic, coagulopathies, hemobilia. Erosive or inflammatory, hemorrhoids, tumours, colorectal cancer, trauma, or iatrogenic.

E:

A: Upper gastrointestinal bleeding (UGIB)
∼ 70–80% of GI hemorrhages [1][2]
The source of the bleeding is proximal to the ligament of Treitz.
Lower gastrointestinal bleeding (LGIB)
∼ 20–30% of all GI hemorrhages [3][2]
The source of the bleeding is distal to the ligament of Treitz (usually in the colon).
Other most common etiologies in document. Include:
UGIB: peptic ulcer disease, esophagitis, erosive gastritis and or duodenitis. Esophageal varices, angiodysplasia.
LGIB: diverticulosis, hemorrhoids.

C: Anemia due to chronic blood loss
Acute hemorrhage: signs of circulatory insufficiency or hypovolemic shock
Tachycardia, hypotension (dizziness, collapse, shock)
Altered mental status.
-Hematemesis: vomiting blood, which may be red or coffee-ground in appearance.
-Melenma: black, tarry stool with a strong offensive odor.
Hematochezia: The passage of bring red blood through the anus with or without stool (colonic bleeding: maroon, kelly like traces of blood in stools.
Rectal bleeding, streaks of fresh blood on stools.

P:

I: Fecal occult blood test, endoscopy, colonoscopy, nasogastric tube lavage.

M: Initial management [12]
Consider elective intubation in patients with altered mental or respiratory state and severe ongoing hematemesis.
Hemodynamic resuscitation
In case of hemodynamic instability and/or suspected ongoing bleeding
IV fluid to normalize blood pressure and heart rate (see “Fluid resuscitation”)
Transfusion of packed red blood cells in case of massive bleeding (e.g., hemoglobin < 7 g/dL)

Management of anticoagulants
INR 1.5–2.5: Endoscopic hemostasis is possible.
INR > 2.5: Reversal agents should be considered before endoscopy.
If there is any suspicion of GI bleeding, two large-caliber peripheral venous catheters should be inserted and preparations should be made for a possible blood transfusion.
Interventions to stop bleeding [14][15]
Endoscopy
Used to locate the site of the (suspected) bleeding and to initiate therapy
Injection therapy (e.g., epinephrine): actively bleeding ulcers or blood vessels
Hemostatic surgical procedures: sclerotherapy, band ligation, cauterization, or clip placement
Polypectomy in case of bleeding polyp (e.g., in the colon)
Angiography: vasoconstriction of a bleeding vessel via e.g., intraarterial vasopressin infusion or embolization
Surgery (laparotomy): if bleeding cannot be contained through endoscopic intervention (rarely the case)
Treatment of underlying disease
See “Treatment” sections in “Crohn disease“, “Ulcerative colitis“, “Peptic ulcer disease“, “Hemorrhoids“, “Intestinal ischemia“, “Gastric cancer“, “Colorectal cancer“, and “Portal hypertension.“
Peptic ulcer – you should act immediately to reduce the amount of acid in the stomach. This will inhibit the action of pepsin, and may encourage the initial bleed to heal by itself if the first 24 hours.
The rest is then exactly the same as you would treat a normal peptic ulcer!
Oesophageal Varices– you should treat the chronic liver disease with non-selective β-blockers to lower the general venous pressure and reduce the risk of variceal bleeding.
If the varices present before they have bled, then you can treat them with ‘banding’ whereby youjust stick a little band around the bulging varices and the weakest part of the vein will be cut off from the rest, and will eventually just drop off. This treatment can be done by endoscopy.
In acute severe variceal bleeding, where the patient is in immediate danger, you may wish to use a Minnesota tube. This is a tube that is inserted down the oesophagus, into the stomach, and a small bag of air inflated at the bottom with about 300ml of air. This first bag prevents the tube being pulled out of place. There is a second bag that can also be filled with air that sits in the oesophagus. This bag is inflated to around 40mmHg pressure – just above normal venous pressure – and thus this keeps blood in the veins, rather than letting it spurt out into the oesophagus. You must be very careful when inflating the second bag as you can perforate the oesophagus!
This is only a temporary solution. It is often very uncomfortable for the patient
There are some additional steps if oesophageal varices are suspected, for example in patients with a history of chronic liver disease:

Terlipressin
Prophylactic broad spectrum antibiotics
The definitive treatment is oesophagogastroduodenoscopy (OGD) to provide interventions that stop the bleeding, for example banding of varices or cauterisation of the bleeding vessel.

NICE recommend against using a proton pump inhibitor prior to endoscopy, however you may find senior doctors that do this.

P: Hypovolemic shock
Hepatic encephalopathy (in patients with liver cirrhosis)
Aspiration pneumonia [16][17]
We list the most important complications. The selection is not exhaustive.

How well did you know this?
1
Not at all
2
3
4
5
Perfectly
17
Q

Gastroesophageal reflux tomach cancer

A

D: Gastroesophageal reflux: regurgitation of stomach contents into the esophagus (can also occur in healthy individuals, e.g., after consuming greasy foods or wine)
Gastroesophageal reflux disease (GERD): A condition in which reflux causes troublesome symptoms (typically including heartburn or regurgitation) and/or esophageal injury/complications. The most common endoscopic finding associated with esophageal mucosal injury is reflux esophagitis. [1]
NERD (non-erosive reflux disease): characteristic symptoms of gastroesophageal reflux disease in the absence of esophageal injury, such as reflux esophagitis, on endoscopy (50–70% of GERD patients) [2]
ERD (erosive reflux disease): gastroesophageal reflux with evidence of esophageal injury, such as reflux esophagitis, on endoscopy (30–50% of GERD patients).

R: Risk factors include smoking, alcohol consumption, stress, obesity, and anatomical abnormalities of the esophagogastric junction (e.g., hiatal hernia).
Risk factors/associations
Lifestyle habits such as smoking, caffeine and alcohol consumption [5][6]
Stress [2]
Obesity [7]
Pregnancy [5][3]
Diaphragm dysfunction [3]
Angle of His enlargement (> 60°) [8][9]
Iatrogenic (e.g., after gastrectomy)
Inadequate esophageal protective factors (i.e., saliva, peristalsis) [5]
Gastrointestinal malformations and tumors: gastric outlet obstruction, gastric cardiac carcinoma
Scleroderma [5]
Sliding hiatal hernia: ≥ 90% of patients with severe GERD ,

D: Other forms of esophagitis
Infectious esophagitis: generally in immunocompromised patients
Esophageal candidiasis: Endoscopy shows white or yellowish adherent plaques.
Herpes esophagitis: Endoscopy shows superficial ulcers in the upper or mid esophagus in the absence of plaques.
CMV esophagitis: Endoscopy shows distal mucosal erosions and ulcers; viral inclusion bodies in cell nuclei on biopsy.
Drug-induced esophagitis: Some medications may cause esophageal mucosal irritation, leading to erosions and ulcers.[17]
Causes:
Antibiotics (e.g., tetracycline, doxycycline, and clindamycin)
Anti-inflammatory drugs (e.g., Aspirin)
Bisphosphonates (e.g., Alendronate)
Others (e.g., potassium chloride, quinidine, and iron compounds)
Endoscopic findings: punched-out ulcers with mild inflammatory changes of the surrounding mucosa
Eosinophilic esophagitis
Associated with allergic disease (allergic asthma, allergic rhinitis) in 50% of cases
Endoscopic findings
Circumferential mucosal lesions (rings/corrugations)
Mucosal fragility
Histological finding: increased number of eosinophils.
Cardiac: See differential diagnosis of chest pain (especially angina pectoris).
Gastrointestinal
Diffuse esophageal spasm
Achalasia
Osteochondrosis
Da Costa’s syndrome (or neurocirculatory asthenia).

E: Prevalence: ∼ 15–30% in the US (increases with age) [3]
Sex: ♀ = ♂

A: Main mechanism: transient lower esophageal sphincter relaxations (tLESRs) [5]
The dysfunctional LES loosens independent of swallowing and has a decreased ability to constrict, which allows stomach contents to uncontrollably flow back into the esophagus (otherwise known as sphincter insufficiency).
Causes ∼⅔ of reflux episodes

C: The chief complaint is retrosternal burning pain (heartburn), but a variety of other symptoms, such as dysphagia and a feeling of increased pressure, are also common. Suspected GERD should already receive empirical treatment, but further diagnostic steps, such as an upper endoscopy and/or 24-hour pH test, may be indicated to confirm the diagnosis.
Chief complaint: retrosternal burning pain (heartburn) that worsens while lying down (e.g., at night) and after eating
Pressure sensation in the chest
Belching, regurgitation
Dysphagia
Chronic non-productive cough and nocturnal cough
Nausea and vomiting
Halitosis
Triggers:
Bending down, supine position
Habits: smoking and/or alcohol consumption
Psychological factors: especially stress.

P: The histopathological findings vary depending on the severity of mucosal damage: [4]
Superficial coagulative necrosis in the non-keratinized squamous epithelium
Thickening of the basal cell layer
Elongation of the papillae in the lamina propria and dilation of the vascular channels at the tip of the papillae (→ hyperemia)
Inflammatory cells (granulocytes, lymphocytes, macrophages)
Transformation of squamous into columnar epithelium → Barrett’s metaplasia.

I: Empirical therapy: If GERD is clinically suspected and there are no indications for endoscopy, empiric therapy – ranging from lifestyle modifications to a short trial with PPIs – should be initiated. A GERD diagnosis is assumed in patients who respond to this therapeutic regimen. [7]
Upper endoscopy (esophagogastroduodenoscopy (EGD))
Used to classify reflux esophagitis and conduct biopsies [4]
Indications for endoscopy
Signs of complicated disease (e.g., dysphagia, painful swallowing, weight loss, iron deficiency anemia, and aspiration pneumonia) [12]
Extended course of symptoms [12][7]
Noncardiac chest pain [7]
No response to PPI treatment [13]
Esophageal pH monitoring
Measured over 24 hours via nasogastric tube with a pH probe [4]
Sudden drops to a pH ≤ 4 are consistent with episodes of acid reflux into the esophagus [4]
Indications
To confirm suspected NERD [4]
Before endoscopic or surgical treatment options are initiated in patients with NERD [7]
GERD is diagnosed when drops in esophageal pH correlate with symptoms of acid reflux and precipitating activities noted in the patient’s event diary.
Esophageal manometry
A pressure-sensitive nasogastric tube measures the muscle contractions in several sections of the esophagus while the patient swallows
Indications:
Ensure correct placement of pH probes
Evaluate peristaltic function prior to anti-reflux surgery
Exclude motor disorders that may mimic the symptoms of GERD

M: Management involves lifestyle modifications, medications, and possibly surgery. Proton pump inhibitors (PPIs) are the treatment of choice, although other agents – such as histamine H2-receptor antagonists (H2RAs) – may also be helpful. In addition to relieving symptoms, treating esophagitis is especially important, as chronic mucosal damage can lead to a premalignant condition known as Barrett’s esophagus, further progressing to adenocarcinoma of the esophagus.
Dietary
Small portions; avoid eating (< 3 hours) before bedtime [3][7]
Avoid foods with high fat content [13]
Physical
Normalize body weight [7]
Elevate the head of the bed for patients with nighttime symptoms [7]
Avoid toxins: nicotine, alcohol, coffee [13][7] , and certain drugs (e.g., calcium channel blockers, diazepam).
Medical therapy
Treatment of choice: Standard-dose of PPI for at least 8 weeks (once-daily therapy)
No response: further diagnostic evaluation
Partial response: increase the dose (to twice daily therapy) or switch to a different PPI
Good response: discontinue PPI after 8 weeks
Maintenance therapy: if symptoms recur after discontinuation of PPIs and in the case of complications (see “Complications” below)
After 8 weeks of initial treatment, reduce PPI to lowest effective dose or switch to H2RAs (only in patients without complications!).
Surgical therapy
Indications
Equally effective alternative to medical therapy in certain patients with chronic GERD [7]
Complications (e.g., Barrett esophagus, strictures, recurrent aspiration) [18][19]
Fundoplication
Symptoms resolve in 85% of cases, but recurrence is possible [19][20]
Technique: The gastric fundus is wrapped around the lower esophagus and secured with stitches to form a cuff, leading to a narrowing of the distal esophagus and the gastroesophageal junction (GEJ)and prevents reflux. [21]
Nissen fundoplication (= complete fundoplication) [18][19]
Complications [20]
Intraoperative damage to the stomach and/or surrounding organs, especially the esophagus, spleen, lungs/pleura (→ pneumothorax)
Gas bloat syndrome: inability to belch, leading to bloating and an increase in flatulence
Dysphagia (especially to solids)
Telescope phenomenon (“slipped Nissen”): the esophagus slides out of the wrapped stomach portion
Gastric denervation: Vagal nerve injury leads to bloating and cardiac complaints, resembling Roemheld syndrome
If hiatal hernia is present [19]
Hiatoplasty: margins of the widened hiatus are sutured together
Fundopexy or gastropexy: the protruding part of the stomach is tethered to the diaphragm → keeps it in place and relieves the tension placed on the cuff

P: Reflux esophagitis: most common complication of GERD [23]
Iron deficiency anemia: mucosal erosions and ulcerations → chronic bleeding → anemia
Esophageal stricture: most common sequela of reflux esophagitis [23]
Clinical features: cause solid food dysphagia
Diagnostics
Barium esophagram (best initial test): narrowing of the esophagus at the gastroesophageal junction
Endoscopy with biopsies: to rule out malignancy and eosinophilic esophagitis
Treatment
First-line treatment: dilation with bougie dilator/balloon dilator + proton pump inhibitors in patients with reflux
In refractory cases (multiple recurrences): steroid injection prior to dilation, endoscopic electrosurgical incision
Recurrence occurs in the majority of patients; often multiple treatment attempts necessary
Esophageal ring: [24]
Schatzki rings at the squamocolumnar junction are the most common type
Clinical features and management similar to that of an esopahgeal stricture
Aspiration of gastric contents leads to:
Aspiration pneumonia
Chronic bronchitis
Asthma (exacerbation)
Laryngitis and hoarseness
Barrett esophagus
Pathophysiology
Reflux esophagitis → stomach acid damages squamous epithelium → squamous epithelium becomes replaced by columnar epithelium and goblet cells (intestinal metaplasia, Barrett’s metaplasia) [3]
The physiological transformation zone (“Z-line”) between squamous and columnar epithelium is shifted upwards
Pathology
Short-segment Barrett’s esophagus (< 3 cm of columnar epithelium between Z-line and GEJ)
Long-segment Barrett’s esophagus (> 3 cm of columnar epithelium between Z-line and GEJ) → higher cancer risk!
Complications: precancerous condition for adenocarcinoma (see esophageal cancer)
Management and surveillance [25]
Medical treatment with PPIs
Endoscopy with four-quadrant biopsies at every 2 cm of the suspicious area (salmon colored mucosa)
If no dysplasia: repeat endoscopy every 3–5 years
If indefinite for dysplasia: repeat endoscopy with biopsies after 3–6 months of optimized PPI therapy
If low-grade dysplasia
Endoscopic therapy of mucosal irregularities
Alternatively: surveillance every 12 months with biopsies every 1 cm
If high-grade dysplasia: endoscopic therapy of mucosal irregularities

How well did you know this?
1
Not at all
2
3
4
5
Perfectly
18
Q

Barretts oesophagus

A

Constant reflux of acid results in the lower oesophageal epithelium changing in a process known as metaplasia from a squamous to a columnar epithelium. This change to columnar epithelium is called Barretts oesophagus. When this change happens patients typically get an improvement in reflux symptoms.

Barretts oesophagus is considered a “premalignant” condition and is a risk factor for the development of adenocarcinoma of the oesophagus (3-5% lifetime risk with Barretts). Patients identified as having Barretts oesophagus are monitored for adenocarcinoma by regular endoscopy. In some patients there is a progression from Barretts oesophagus (columnar epithelium) with no dysplasia to low grade dysplasia to high grade dysplasia and then to adenocarcinoma.

Treatment of Barretts oesophagus is with proton pump inhibitors (e.g. omeprazole). There is new evidence that treatment with regular aspirin can reduce the rate of adenocarcinoma developing however the is not yet in guidelines.

Ablation treatment during endoscopy using photodynamic therapy, laser therapy or cryotherapy is used to destroy the epithelium so that it is replaced with normal cells. This is not recommended in patients with no dysplasia but has a role in low and high grade dysplasia in preventing progression to cancer.

How well did you know this?
1
Not at all
2
3
4
5
Perfectly
19
Q

Barrett’s oesophagus

A

D: lso known as Columnar Lined oesophagus (CLO)

This is a complication of long term oesophageal reflux.
It looks a bit like white pasty finger like projections coming up from the bottom of the oesophagus.
Strictly speaking it is defined as >3cm columnar epithelium present at the bottom of the oesophagus.
The mechanism by which it occurs is not fully understood. Some argue that it is a migration of the epithelium of the bottom 2cm of the oesophagus further up the lumen – i.e. a migration of essentially the gastric epithelium. Others would say that as both squamous and columnar epithelium are derived from the same stem cells, it is just perhaps a change in the intra-luminal conditions that determine what those stem cells develop into; more acidic conditions will cause a columnar epithelium to form. The second idea is more popular.
It is found in 10% of patients who have an endoscopy for reflux symptoms.
It is seen endoscopically as a pinker/redder epithelium as opposed to the whiter epithelium that lines the rest of the oesophagus. At first it may just extend up as thin bands of epithelium, but later it may encircle the entire lower oesophagus.
On microscopic examination, the cells resemble those of the stomach, however they secrete intestinal-like mucins, and as a result, Barrett’s oesophagus is actually a form of intestinal metaplasia.
Metaplasia – This usually occurs as a result of an environmental factor. It is when an existing epithelium is replaced by a different one; one that now has an ‘advantage’ due to the change in environmental conditions.
It is a major risk factor in oesophageal adenocarcinoma – there is a lifetime risk of 5-10%. Only about 5% of Barrett’s patients will actually ever get dysplasia. The risk for a Barrett’s patient to get ANY CANCER in their lifetime is:
No dysplasia – 2% (this is almost exactly the same as the general population)
Low-grade dysplasia – 30%
High-grade dysplasia – 50%
As such is defined as a ‘premalignant condition’, and patients who have it are at 100x times greater risk of cancer than the general population.(i.e. risk of oesophageal adenocarcinoma goes from 4 in 100 000 to 400 in 100 000. This however, is still a relatively small risk overall (0.4%), and as such the patients overall risk of getting any cancer is still about 2-3%.
95% of Barrett’s patients will die of another cause other than oesophageal cancer.
Most cases of CLO are not detected until cancer develops.

R:

D:

E: Associated with men >50, particularly Caucasians.
Increased risk due to smoking (but not alcohol)

A:

C:

P:

I: When you first find Barrett’s/if you suspect oesophageal carcinoma then you would biopsy the oesophagus. This has very strict guidelines, and you have to biopsy every quadrant (i.e. imagine the oesophagus as having a square lumen, and biopsy all four sides) for every 2cm of Barrett’s present. So if the patient has 6cm of Barrett’s, you have to do 12 biopsies. The idea behind this is so that you don’t miss any possible areas of high grade dysplasia, because macroscopically (ie as you look at the oesophagus down the endoscope) you cannot see a difference between areas of low grade and high grade dysplasia.
The evidence for surveillance is not conclusive. The current BSG (British society of Gastroenterologists) guidelines state that you can do surveillance if you want to, but you don’t have to. As a result, some trusts offer it and some don’t.
If you do offer surveillance, then you must be sure that if anything is found, that the patient is fit enough to undergo surgery. E.g. do you really want to offer surveillance to a 90 year old man?
Where it is offered, surveillance is currently recommended for those with low grade dysplasia about every 6-12 months, and for those without dysplasia every 2-3 years. Dysplasia takes about 2-5 years to develop.
Even with surveillance, it is not that likely you will find a carcinoma any sooner (hence the option not to survey). For example, you might have missed an area of high dysplasia last time round, and by the time you survey again it has developed beyond a treatable stage.

M: There is no evidence that treatment with PPI’s or anti-reflux medicine leads to recession of Barrett’s.
However, treatment is basically that of GORD.

P:

How well did you know this?
1
Not at all
2
3
4
5
Perfectly
20
Q

Oesophageal carcinoma

A

Osophageal cancer typically assumes the form of adenocarcinoma or squamous cell carcinoma, although there are some rarer tumors. Adenocarcinomas are considered the fastest-growing neoplasia in Western countries, while squamous cell carcinoma is still most common in the developing world. Adenocarcinoma, which usually affects the lower third of the esophagus, may be preceded by Barrett’s esophagus, a complication of gastroesophageal reflux disease (GERD). In addition to GERD, other risk factors include obesity and smoking. Squamous cell carcinomas mostly occur in the upper two-thirds of the esophagus. Known risk factors for squamous cell carcinoma include carcinogen exposure from alcohol and tobacco consumption, and dietary factors (e.g., diet low in fruits and vegetables). Esophageal cancers are often asymptomatic in early stages of the disease. Locally advanced disease is common at presentation, progressive dysphagia being the primary symptom. Hoarseness, weight loss, and hematemesis may also be present. Endoscopy is the primary diagnostic test, enabling direct visualization and biopsy of the lesion for histopathological confirmation. Curative surgical resection may be considered for locally invasive cancers. Esophageal cancer is unresectable at presentation in about 60% of patients. Chemotherapy, radiation, and palliative stenting play a role in the management of unresectable disease.

How well did you know this?
1
Not at all
2
3
4
5
Perfectly
21
Q

oesophageal cancer

A

D:

R: Adenocarcinoma
Risk factors
Gastroesophageal reflux: Barrett’s esophagus
Obesity
Smoking
Achalasia
Localization: mostly in the lower third of the esophagus
Squamous cell carcinoma (SCC)
Risk factors [4]
Alcohol consumption
Smoking
Diet low in fruits and vegetables
Drinking hot beverages
Achalasia
Nitrosamines exposure (e.g., cured meat, fish, bacon) [5]
Plummer-Vinson syndrome
Caustic strictures
Diverticula (e.g., Zenker’s diverticulum)
Radiotherapy
Esophageal candidiasis
Betel or areca nut chewing
Localization: mostly in the upper two-thirds of the esophagus
The primary risk factors for squamous cell esophageal cancer are alcohol consumption, smoking, and dietary factors (e.g., diet low in fruits and vegetables)!

D:

E: Sex: ♂ > ♀ (3:1)
Peak incidence: 60–70 years of age
Adenocarcinoma: most common type of esophageal cancer in the US [1]
Squamous cell carcinoma (SCC): most common type of esophageal cancer worldwide [2]
Adenocarcinoma is more common in the US of America.

A:

C: Early stages: Often asymptomatic but may present with swallowing difficulties or retrosternal discomfort
Late stages
Common
Progressive dysphagia (from solids to liquids) with possible odynophagia
Weight loss
Retrosternal chest or back pain
Anemia
Less common
Hematemesis, melena
Hoarseness
Esophageal cancer is a “silent” disease and typically becomes symptomatic at advanced stages!

P: Adenocarcinoma
Histological characteristics: often present with adjacent Barrett mucosa (columnar epithelium with goblet cells) and high-grade dysplasia
Squamous cell carcinoma
Histological characteristics
Breakdown of uniform tissue structure
Squamous cell carcinoma clusters with circular keratinization
Lymphocytic infiltration between the carcinoma clusters

I: Esophagogastroduodenoscopy (best initial and confirmatory test) [8]
Direct visualization of the tumor
With biopsy of any suspicious lesions
Barium swallow: asymmetrical and irregular borders of the esophagus with characteristic stenosis and proximal dilatation (apple core lesion)
Sensitive, but does not allow confirmation or staging of a malignancy. Inferior to endoscopy, but indicated in the case of:
Severe stricture that inhibits endoscopic evaluation
Suspected tracheoesophageal fistula due to esophageal cancer
Staging [8]
Transesophageal endoscopic ultrasound
Chest and abdominal CT and/or PET
Bronchoscopy or laparoscopy

M: Curative
Indication
Locally invasive disease that has not invaded surrounding structures
High-grade metaplasia in Barrett syndrome
Methods
Neoadjuvant chemoradiation
For downstaging → potentially allows for later resection
As definitive treatment in patients with proven complete response (e.g., during endoscopy)
Surgical resection
Endoscopic submucosal resection for removal of superficial, epithelial lesions
Subtotal or total esophagectomy with gastric pull-through procedure or colonic interposition
Palliative
Indication: patients with advanced disease (majority of patients)
Methods:
Chemoradiation
Stent placement
Other endoscopic treatments (e.g., laser therapy

P:  Esophageal stenosis
Tracheoesophageal fistula: passage of food and fluid into the respiratory tract
Postoperative
Anastomotic leak or stricture
Recurrent laryngeal nerve injury. 

Generally poor prognosis due to an aggressive course (due to an absent serosa in the esophageal wall) and typically late diagnosis.
The more distal the tumor, the better the prognosis
Esophageal cancer has an aggressive course and metastasizes early because of the absence of serosa in parts of the esophagus!

How well did you know this?
1
Not at all
2
3
4
5
Perfectly
22
Q

Peptic ulcers

A

Peptic ulcer disease (PUD) refers to the presence of one or more ulcerative lesions in the stomach or lining of the duodenum. Possible etiologies include infection with the bacterium Helicobacter pylori (most common), prolonged use of nonsteroidal anti-inflammatory medicines (possibly in combination with glucocorticoids), conditions associated with an overproduction of stomach acid (hypersecretory states), and stress. Epigastric pain is a typical symptom of PUD, however, some patients may remain asymptomatic. Diagnosis occurs via direct visualization of the ulcer on esophagogastroduodenoscopy (EGD) and H. pylori detection (via biopsy or non-invasive testing). The first-line treatment for most peptic ulcers involves H. pylori eradication via triple therapy (a course of two different antibiotics in combination with a proton-pump inhibitor) and the withdrawal of offending agents. Antisecretory drugs (e.g., proton-pump inhibitors, or PPIs), which reduce stomach acid production, are continued for 4–8 weeks after eradication therapy and may be considered for maintenance therapy if symptoms recur. Surgical intervention may be necessary in rare cases, especially if complications such as perforation or massive bleeding occur. Stomach cancer is an important differential diagnosis and must be ruled out if risk factors are present.

How well did you know this?
1
Not at all
2
3
4
5
Perfectly
23
Q

Peptic ulcers DR DEAC PIMP

A

D: Gastric ulcer: an ulcerative lesion in the stomach lining; typically manifests along the lesser curvature and the gastric antrum
Duodenal ulcer: an ulcerative lesion located in the duodenum, typically in the first part (i.e., the duodenal bulb)
Erosive gastritis: acute mucosal inflammation of the stomach that does not extend beyond the muscularis mucosae
An atypical location is suspicious for carcinoma!

R: Risk factors
Chronic gastritis caused by H. pylori, a curved, flagellated gram-negative rod
Duodenal ulcers: up to 90% are due to H. pylori infection
Gastric ulcers: up to 80% are due to H. pylori infection
Chronic gastritis of other etiology
Long-term use of NSAIDs (e.g., patients with rheumatoid arthritis, SLE, etc.):
Risk for gastroduodenal ulcers increases 5-fold.
NSAID use seems to have a stronger association with gastric ulcers than with duodenal ulcers.
Long-term use of NSAIDs plus glucocorticoids: Risk increases 10 to 15-fold!
SSRIs
Smoking
Chronic alcohol consumption
Patients with blood type O have a higher risk for duodenal ulcers.
Age > 65 years
Stress (see “Subtypes and variants” below)
Rare risk factors:
Zollinger-Ellison syndrome (can result in duodenal ulcer)
Hyperparathyroidism

E:Incidence: > 6 million cases annually in the US
Duodenal ulcers are 3 times more common than gastric ulcers.
Duodenal ulcers occur on average 10–20 years earlier than gastric ulcers.
Sex: ♂ = ♀

A:

C:Findings common to both (gastric and duodenal):
Dyspepsia: postprandial heaviness, early satiety, and gnawing, aching or burning epigastric pain
Pain relief with antacids
Potential signs of internal bleeding (anemia, hematemesis, melena)
∼ 70% of patients with PUD are asymptomatic
Stool sample positive for occult blood (see gastrointestinal bleeding).
For gastric ulcers:
-pain increases shortly after eating -> weight loss.
-Nocturnal pain in 30-40% of patients.

Duodenal ulcer:
Pain increases 2-5 hours after eating. Pain on an empty stomach (hunger pain) that is relieved with food intake -> weight gain.
Nocturnal pain in 50-80% of patients.
Gastric ulcer is associated with pain after light (weight loss) Gorging. Duodenal ulcer is associated with relief after massive (weight gain) Desserts.

Taking NSAIDs can often mask PUD symptoms until complications such as hemorrhage and perforation occur!

P:

I: ≤ 60 years of age without alarm features: Urea breath test for H. pylori
> 60 years of age or presence of ≥ 1 alarm features: EGD with biopsies and rapid urease testing for H. pylori
Negative for H. pylori infection and NSAID intake; trial therapy unsuccessful
Measure serum gastrin level at baseline and after secretin stimulation test: high levels in gastrinoma (Zollinger-Ellison syndrome)
Measure serum calcium and parathyroid hormone: high levels in primary hyperparathyroidism
Testing for Helicobacter pylori
See “Helicobacter pylori diagnostics”
Esophagogastroduodenoscopy (EGD)
Most accurate test
Patients > 60 years of age or presence of ≥ 1 alarm features, which include:
Certain symptoms: progressive dysphagia, painful swallowing (odynophagia), and/or persistent vomiting
Signs of active GI bleeding (e.g., melena, unexplained iron-deficiency anemia)
Signs of malignancy (e.g., unintended weight loss, lymphadenopathy, palpable mass)
Family history of upper GI malignancy in a first-degree relative
Jaundice
Biopsy samples from:
Edge and base of the ulcer (essential to rule out malignancy) .
Stomach lining distant from the ulcer (Helicobacter pylori testing for detection of underlying type B gastritis)
If active bleeding, EGD can be performed for diagnosis and subsequent hemostasis treatment (electrocautery) in the same session.
Alarm features of PUD include progressive dysphagia, odynophagia, persistent vomiting, jaundice, signs of GI bleeding, signs of malignancy, and a family history of upper GI malignancy!
To rule out gastric cancer, patients with stomach ulcers should undergo follow-up EGD and histology until the ulcer has healed completely!

M: General management of dyspepsia
H. pylori positive → eradication therapy (with antibiotics and a PPI) and supportive treatment → continue PPIs for 4–8 weeks → follow-up
H. pylori negative → medical acid suppression (with a PPI) and supportive treatment for 4–8 weeks → follow-up
Medical treatment
Helicobacter pylori eradication therapy (with antibiotics)
Acid suppression: PPIs (most effective), H2 blockers, antacids (mainly used for symptom relief)
Mucosal protection: misoprostol , sucralfate (both substances are rarely used in PUD)
Supportive treatment
Discontinue NSAIDs
Restrict alcohol use/smoking/emotional stress
Avoid eating before bedtime
Surgical treatment
With the advent of potent acid suppression in the form of PPIs, surgical intervention is rarely needed.
Indications
Refractory syndromes despite appropriate medical treatment
If cancer is suspected
Complications that cannot be treated endoscopically (see “Complications” below)
Partial gastrectomy (Billroth)
Billroth I: distal gastrectomy with end-to-end or side-to-end gastroduodenostomy
Billroth II: resection of the distal ⅔ of the stomach with a blind-ending duodenal stump and end-to-side gastro-jejunostomy. The Billroth I and II methods without a Brown’s anastomosis often lead to bile reflux into the stomach. This may result in type C gastritis in the region of the anastomosis. The chronic inflammation causes atrophic changes and increases the risk of cancer (anastomosis carcinoma).
Vagotomy

P: Bleeding (see gastrointestinal bleeding)
Most common complication of PUD
Located posterior more commonly than anterior
Perforated gastric ulcers of the lesser curvature may cause hemorrhage of the left gastric artery.
Duodenal ulcers of the posterior wall are more likely to cause massive bleeding because of their proximity to the gastroduodenal artery.
Gastric/duodenal perforation (see also secondary peritonitis and gastrointestinal perforation)
Second most common complication of PUD
Located anterior more commonly than posterior
Duodenal ulcers of the anterior wall are more likely to perforate into the abdominal cavity, causing pneumoperitoneum (free air below the diaphragm) and irritation of phrenic nerve (e.g., shoulder pain).
Subhepatic abscess
Etiology: may result from a perforated duodenal or gastric ulcer
Clinical presentation: fever and vomiting (see also “Gastric/duodenal perforation” above)
Diagnosis: subhepatic gas on abdominal x-ray
Management
Treat underlying cause
IV antibiotics, percutaneous drainage
See also “Therapy” in pyogenic liver abscess.
Gastric outlet obstruction (GOO)
Definition: mechanical obstruction of the pyloric channel or duodenum
Etiology
Malignancy (most common)
PUD
Acute PUD → inflammation and edema
Chronic PUD → scarring and fibrosis
Gastric volvulus
Less common causes that cause strictures in the pyloric channel: Crohn disease, history of ingestion of a caustic substance, chronic pancreatitis
Clinical presentation
Postprandial, nonbilious vomiting
Succussion splash
Early satiety
Progressive gastric dilation
Weight loss
Diagnosis
Barium swallow
Upper endoscopy (confirmatory test): identification of the gastric pathology
Laboratory tests: hypokalemic hypochloremic metabolic alkalosis
Saline load test
Management
Symptomatic: nasogastric suction, electrolyte and fluid replacement, and parental nutrition
Definitive: surgery or endoscopic dilation
Fistula formation
Clinical presentation
Increased severity of pain, which is no longer relieved by eating; radiation of pain to the back
Weight loss, diarrhea
Malignant transformation
Gastric ulcers: high malignant potential (progression to cancer in 5–10% of cases) → malignancy should be ruled out with biopsy [20]
Duodenal ulcers: usually benign → routine biopsy is not required
Postgastrectomy syndromes
Posterior ulcers are more likely to bleed and anterior ulcers are more likely to perforate: Postal workers wear Blue collars and should not have an Antisocial Personality.

How well did you know this?
1
Not at all
2
3
4
5
Perfectly
24
Q

Gastric cancer Dr Deac Pimp

A

D: Gastric cancer refers to neoplasms in the stomach, including cancers of the esophagogastric junction. The incidence is declining in the United States and Europe, while it is rising in Japan and South Korea. Gastric cancer is associated with several risk factors (e.g., consumption of foods high in nitrates, increased nicotine intake, Helicobacter pylori infection). In its early stages, the disease is often asymptomatic or accompanied by nonspecific symptoms (e.g., epigastric discomfort, postprandial fullness, or nausea). Late-stage disease may present with gastric outlet obstruction (mechanical obstruction of the pyloric canal), leading to weight loss and vomiting. Biopsy during endoscopy confirms the diagnosis. Adenocarcinomas are the most common form of gastric cancer. Treatment includes endoscopic or surgical resection. Depending on staging, chemotherapy may be indicated before or after surgery (neoadjuvant or adjuvant chemotherapy), or as a palliative therapy.

R:

D:
Gastric ulcer
Gastroesophageal reflux disease (GERD)
Ménétrier's disease
Non-ulcer dyspepsia
Other types of cancer
MALT lymphoma
Sarcoma
Gastrointestinal stromal tumor (GIST)

E: Sex: ♂ > ♀
Peak incidence: 70 years
Geographical distribution: strong regional differences
High incidence in South Korea and Japan
Declining incidence in the United States and Europe

A: Exogenous risk factors
Diet rich in nitrates and/or salts (e.g., dried, preserved food)
Nicotine use
Low socioeconomic status
Endogenous risk factors
Diseases associated with a higher risk of gastric cancer
Atrophic gastritis
H. pylori infection: associated with a higher risk of intestinal gastric cancer but not with diffuse gastric cancer
Gastric ulcers
Partial gastrectomy
Gastroesophageal reflux disease (GERD; for cancers of the gastroesophageal junction)
Adenomatous gastric polyps
Hereditary factors (positive family history, hereditary non‑polyposis colorectal cancer)
Higher incidence in individuals with blood type A.

C: Gastric cancer is often asymptomatic. Early signs are nonspecific and often go unnoticed. At later stages the following symptoms may occur:
General signs
Weight loss
Chronic iron deficiency anemia (paleness, fatigue, headaches)
Gastrointestinal signs
Abdominal pain
Early satiety
Nausea or vomiting
Dysphagia
Acute gastric bleeding (hematemesis or melena)
Late stage gastric cancer
Palpable tumor in epigastric region
Gastric outlet obstruction
Hepatomegaly, ascites
Virchow’s node: left supraclavicular adenopathy, located where the thoracic duct joins the subclavian vein at the venous angle.
Sister Mary Joseph’s node: umbilical node indicating metastasis from a gastrointestinal or abdominopelvic malignancy
Malignant acanthosis nigricans (in particular associated with gastric adenocarcinoma).
Subtypes and variants

Metastatic Disease
Lymphangitic spread
All local lymph nodes (lesser and greater curvature)
Celiac, paraaortic, and mesenteric lymph nodes
Carcinoma of the cardia may spread to mediastinal lymph nodes.
Hematogenous spread: liver, lung, skeleton, brain
Local invasion of adjacent structures
Peritoneal carcinomatosis
Esophagus, transverse colon, pancreas, etc.
Direct seeding
To the ovaries (Krukenberg tumor): an ovarian malignancy comprised of signet ring cells that has metastasized from a primary site, most commonly the stomach
To the pouch of Douglas.

P: Adenocarcinoma (90% of cases)
Typically localized, exophytic lesion +/- ulceration
Arise from glandular cells in the stomach; usually located on the lesser curvature of the stomach
Signet ring cell carcinoma
Diffuse growth
Multiple signet ring cells = round cells filled with mucin, with a flat nucleus in the cell periphery
Less common
Adenosquamous carcinoma
Squamous cell carcinoma.
Lauren classification of gastric adenocarcinoma
Intestinal type (∼ 50% of cases): polypoid, glandular formation; expanding (not infiltrative) growth pattern; clear border
Diffuse type (∼ 40% of cases): infiltrative growth and diffuse spread in the gastric wall, no clear border
Mixed type (∼ 10% of cases)

I: Diagnostic procedures
Upper endoscopy with biopsy (best initial test) : Biopsy confirms the diagnosis
Barium upper GI series may be considered and would show loss of intestinal folds and stenosis
Laboratory test
Iron deficiency anemia
Serologic markers : TNF-α as possible future tumor marker
Staging
Abdominal ultrasound
Endosonography
Assessment of tumor depth and local lymph nodes
Abdominal and pelvic CT-scan using intravenous and oral contrast;
Thoracic CT-scan
Diagnostic laparoscopy.

M: Exact therapy, which may be either curative or palliative, depends on staging and the type of tumor.
Endoscopic resection
Surgery
Perioperative chemotherapy, sometimes radiotherapy
Trastuzumab is indicated for HER2+ gastric adenocarcinomas
Surgery
Radical gastrectomy and lymphadenectomy (operative standard)
Resection of the lesser and greater omentum and radical lymphadenectomy
Roux-en-Y gastric bypass
The surgeon separates the proximal jejunum from the duodenum and creates an end-to-end anastomosis of the jejunum with the remaining part of the stomach (gastrojejunostomy), or in the case of a total gastrectomy, with the esophagus (esophagojejunostomy).
Duodenal stump is connected distally with the jejunum using an end-to-side anastomosis.
Alternative: subtotal gastrectomy
Total gastrectomy (with Roux-en-Y anastomosis)Subtotal gastrectomy (with Roux-en-Y anastomosis)

P: Malignant acanthosis nigricans
A paraneoplastic syndrome seen in adenocarcinomas of GI origin, especially in gastric adenocarcinoma
Pathophysiology: caused by exogenous transforming growth factor TGF-α and epidermal growth factor (GF)
Clinical findings
Brown to black, intertriginous and/or nuchal hyperpigmentation that can turn into itching, papillomatous, poorly-defined efflorescence
Rapid growth and verrucous or papulous surface helps to differentiate it from benign acanthosis nigricans
Localization: axilla, groin, neck
Malignant acanthosis nigricans always requires further diagnostic measures to look for malignancy!
Postgastrectomy syndromes
Related to resorption
Maldigestion
Consequences and management
Iron deficiency → supplement iron
Pernicious anemia due to lack of intrinsic factor, usually produced by gastric parietal cells → supplement vitamin B12
Related to anastomosis
Small intestinal bacterial overgrowth (SIBO)
Definition: bacterial overgrowth within the small intestine
Causes
Anatomic abnormalities: (e.g., surgery causing blind intestinal loops – blind loop syndrome ), strictures)
Motility disorders
Pathophysiology: bacterial overgrowth → bacteria deconjugate bile acids, increase vitamin B12 turnover, and produce increased amounts of vitamin K and folic acid
Clinical features: diarrhea, steatorrhea, weight loss, malabsorption (e.g., deficiency of vitamin B12, A, E, D, zinc, and iron)
Diagnostics
Jejunal aspirate cultures collected during endoscopy
Positive lactulose breath test
Treatment: antibiotics and parenteral supplementation of vitamins and proteins, possibly surgical treatment
Related to motility
Dumping syndrome: rapid gastric emptying due to either defective gastric reservoir function or pyloric emptying mechanism, or anomalous postsurgery gastric motor function.
Early dumping
Cause: rapid emptying of undiluted chyme into the small intestine caused by a dysfunctional or bypassed pyloric sphincter
Clinical features
Appears within 15–30 minutes after ingestion of a meal
Symptoms may include nausea, vomiting, diarrhea, and cramps, as well as vasomotor symptoms such as sweating, flushing, and palpitations.
Management
Dietary modifications: Small meals that include a combination of complex carbohydrates and foods rich in protein and fat to cover protein and energy requirements are preferable.
30–60 min of rest in the supine position after meals
Often spontaneous improvement after a couple of months
Late dumping
Cause: postprandial hypoglycemia; dysfunctional pyloric sphincter → chyme containing glucose immediately reaches the small intestine → glucose is quickly resorbed → hyperglycemia → excessive release of insulin → hypoglycemia and release of catecholamines
Treatment
Dietary modifications
Octreotide and surgery are second and third-line therapies
Suspect late dumping syndrome in a patient with previous gastric surgery and hypoglycemia!
Remnant carcinoma is a complication associated with the remnant stomach. Follow-up is important!

prognosis: Since there are no early signs, gastric cancer is often diagnosed very late. At diagnosis, 60% of cancers have already reached an advanced stage that does not allow for curative treatment.
Early gastric cancer has the best prognosis .
Distant metastases or peritoneal carcinomatosis dramatically worsen the prognosis and are lethal most of the time.

How well did you know this?
1
Not at all
2
3
4
5
Perfectly
25
Q

Jaundice Dr Deac Pimp

A

D: Jaundice: yellowish discoloration of the skin, sclerae, and mucous membranes due to the deposition of bilirubin
Scleral icterus: discoloration of the whites of the eyes, starting at serum bilirubin levels > 2–3 mg/dL
Jaundice: discoloration of the skin and mucous membranes, starting at serum bilirubin levels > 4–5 mg/dL
Cholestasis: any condition affecting bile formation or secretion (nonobstructive intrahepatic cholestasis), or leading to biliary obstruction within the liver (obstructive intrahepatic cholestasis) or in the biliary ducts between the liver and the duodenum (obstructive extrahepatic or posthepatic cholestasis).

R:

D: Pseudojaundice
Deposition of carotene in the skin (carotenoderma) can also cause yellowing and usually occurs following excessive consumption multivitamin supplements or fruits and vegetables rich in carotenes, such as carrots, sweet potatoes, kale, and oranges.
In contrast to jaundice, it does not lead to scleral icterus.

E:

A: Unconjugated hyperbilirubinemia
Increased hemoglobin breakdown
Hemolysis (e.g., glucose-6-phosphate dehydrogenase deficiency, sickle cell anemia, spherocytosis, hemolytic disease of the fetus and newborn, and blood transfusions )
Dyserythropoiesis (e.g., thalassemia, pernicious anemia, erythropoietic porphyria, and sideroblastic anemia)
Resolving hematoma or internal hemorrhage (e.g., in trauma patients or postoperatively)
Impaired hepatic uptake of bilirubin
Drugs (e.g., rifampin, probenecid, sulfonamides)
Portosystemic shunt
Defective conjugation of bilirubin
Inherited hyperbilirubinemia: Gilbert syndrome and Crigler-Najjar syndrome
Neonatal jaundice
Liver disease: hepatitis (e.g., viral, autoimmune, toxic/alcoholic), cirrhosis, Wilson disease
Hyperthyroidism
Conjugated hyperbilirubinemia
Decreased excretion/reuptake of bilirubin
Inherited disorders: Dubin-Johnson syndrome, Rotor syndrome, and progressive familial intrahepatic cholestasis )
Intrahepatic cholestasis
Liver disease (e.g., hepatitis or cirrhosis)
Primary biliary cholangitis
Drugs (e.g., estrogens, arsenic)
Postoperative cholestasis: precipitated by hypotension/massive blood loss with subsequent hepatic ischemia requiring transfusions
Sepsis
Pregnancy
Infiltrative process (e.g., tuberculosis, sarcoidosis, and amyloidosis)
Extrahepatic cholestasis (biliary obstruction)
Choledocholithiasis
Tumors (pancreatic cancer, cholangiocellular carcinoma, gallbladder cancer)
Inflammatory processes (e.g., primary sclerosing cholangitis, acute and chronic pancreatitis, and abscesses)
Malformations of the bile ducts (e.g., postoperative/inflammatory strictures, and biliary cysts)
Parasitic infection (e.g., liver flukes)
Postoperative bile leaks or biliary duct strictures
Jaundice is not always a sure sign of cholestasis; it may also indicate prehepatic causes. Conversely, cholestasis may be present in the absence of jaundice, particularly during the early stages of cholestasis!
Common causes of hyperbilirubinemia can be remembered with HOT Liver: Hemolysis, Obstruction, Tumor, and Liver disease!
Hepatitis and cirrhosis can cause conjugated or unconjugated hyperbilirubinemia!

C: In addition to jaundice, the following signs may also indicate cholestasis:
Pale, clay-colored (acholic) stool
Darkening of urine
Pruritus
Fat malabsorption (steatorrhea, weight loss)

P: Jaundice is due to an elevated level of serum bilirubin, which may be caused by prehepatic, intrahepatic, or posthepatic defects. Serum bilirubin concentration depends on the rate of formation and hepatobiliary elimination of bilirubin.
Causes of ↑ unconjugated (indirect) serum bilirubin
Increased hemoglobin breakdown
Defective hepatic uptake/conjugation of unconjugated bilirubin
Causes of ↑ conjugated (direct) serum bilirubin
Reduced drainage via biliary tract
Increased reuptake

I: Liver function tests (LFTs)
Parameters of cholestasis
↑ Alkaline phosphatase (ALP)
↑ Gamma glutamyltransferase (γ-GT)
↑ Bilirubin
Elevated liver enzymes: alanine aminotransferase (ALT) and aspartate aminotransferase (AST)
See AST/ALT ratio.
Albumin and PT/INR
Other liver tests: e.g., hepatitis serology, markers of hemolysis (LDH, haptoglobin, indirect bilirubin), autoantibodies
Inflammatory markers
Imaging
Ultrasound → high specificity and sensitivity for differentiating between different forms of cholestasis
Obstructive cholestasis
Dilated bile ducts
Extrahepatic: dilated common bile duct ; may show the double-duct sign
Intrahepatic: double barrel shotgun sign
Cause of biliary obstruction, if present (e.g., stones, tumors, cysts, cholangitis)
Nonobstructive cholestasis
No signs of the dilation observed in obstructive cholestasis
Ultrasound may provide evidence of underlying disease (e.g., liver cirrhosis).
Further imaging, if necessary (e.g., ERCP, MRCP, or CT).

M: Management of the underlying condition
Cirrhosis
Hepatitis B, hepatitis C
Alcoholic liver disease
Primary biliary cholangitis, primary sclerosing cholangitis
Inherited hyperbilirubinemia
Pancreatic cancer, cholangiocarcinoma
Cholelithiasis and cholecystitis
Surgical excision of biliary cysts is the preferred treatment. Cholecystectomy is not necessary unless the gallbladder contains stones.

P: complications: Hepatomegaly: Chronic biliary obstruction leading to backflow of bile may result in inflammation.
Risk of kernicterus in newborns with neonatal jaundice

26
Q

Dissection of thoracic aorta

A

Tearing interscapular pain
Discrepancy in arterial blood pressures taken in both arms
May show mediastinal widening on chest x-ray

27
Q

Diffuse oesophageal spasm

A

Spectrum of oesophageal motility disorders
Caused by uncoordinated contractions of oesphageal muscles
May show ‘nutcracker oesophagus’ on barium swallow
Symptoms include dysphagia, retrosternal discomfort and dyspepsia

28
Q

Gastro-oesphageal reflux

A

Common cause of retrosternal discomfort
Usually associated with symptoms of regurgitation, odynophagia and dyspepsia
Symptoms usually well controlled with PPI therapy
Risk factors include obesity, smoking and excess alcohol consumption

29
Q

Boerhaaves syndrome

A

Spontaneous rupture of the oesophagus
Caused by episodes of repeated vomiting often in association with alcohol excess
Typically there is an episode of repetitive vomiting followed by severe chest and epigastric pain
Diagnosis is by CT and contrast studies
Treatment is surgical; during first 12 hours primary repair, beyond this usually creation of controlled fistula with a T Tube, delay beyond 24 hours is associated with fulminent mediastinitis and is usually fatal.

30
Q

Achalasia

A

Difficulty swallowing, dysphagia to both liquids and solids and sometimes chest pain
Usually caused by failure of distal oesphageal inhibitory neurones
Diagnosis is by pH and manometry studies together with contrast swallow and endoscopy
Treatment is with either botulinum toxin, pneumatic dilatation or cardiomyotomy

31
Q

Pancreatic cancer DR DEAC PIMP

A

D: Pancreatic cancer is the fourth leading cause of cancer deaths in the US and typically affects older individuals in the sixth to eighth decades of life. Underlying risk factors include smoking, obesity, heavy alcohol consumption, and chronic pancreatitis. Pancreatic carcinomas are mostly ductal adenocarcinomas and frequently located in the pancreatic head. The disease is commonly diagnosed at an advanced stage because of the late onset of clinical features (e.g., epigastric pain, painless jaundice, and weight loss). In many cases, the tumor has already spread to other organs (mainly the liver) when it is diagnosed. Treatment is often palliative as surgical resection is only possible in approx. 15% of cases. The most commonly used surgical technique is the pancreaticoduodenectomy (“Whipple procedure”). Five-year survival rates range from 1–20% depending on the extent, spread, and resectability of the tumor.

R: Risk factors
Smoking
Chronic pancreatitis
High alcohol consumption
Type 2 diabetes
Obesity
Occupational exposure to chemicals used in the dry cleaning and metal working industries
Cirrhosis of the liver
H. pylori infection; excess stomach acid
Inherited genetic syndromes (10% of pancreatic cancers)
Familial pancreatic carcinoma
Hereditary pancreatitis (mutations in the PRSS1 gene)
Peutz-Jeghers syndrome
Familial atypical multiple mole melanoma (FAMMM) syndrome
Hereditary breast and ovarian cancer syndrome (BRCA1 and BRCA2 mutations)
HNPCC
Von-Hippel-Lindau syndrome
Neurofibromatosis type 1
Multiple endocrine neoplasia type 1.

D: Pancreatitis
Metastasis (e.g., breast carcinoma, bronchial carcinoma)
Pancreatic pseudocyst
Pancreatic cyst: Epithelium-lined cyst, filled with serous or mucous liquid, often associated with the rare von-Hippel-Lindau syndrome; can be benign, precancerous or cancerous.
Clinical features: abdominal pain, back pain, jaundice, and in case of infection, fever and sepsis
CT scan: cyst appears as a well-circumscribed hyperdense mass in comparison to the surrounding tissue. Pancreatic cancer, on the other hand, is hypodense.
ERCP: cyst shows contrast-enhancement
Treatment
Asymptomatic cyst: no surgical treatment
Symptomatic cyst: CT-guided, endoscopic, or surgical drainage

E: Sex: ♂ > ♀ 
Age of onset: 60–80 years
More common in African Americans
Accounts for ∼ 3% of all cancers in the US and ∼ 7% of cancer deaths
The average lifetime risk: ∼ 1.5%

A:

C: Often no early signs present!
Belt-shaped epigastric pain which may radiate to the back
Jaundice
Courvoisier sign: enlarged gallbladder and painless jaundice
Pale stools, dark urine, and pruritus
Weight loss, nausea, weakness, poor appetite
Diarrhea (possibly steatorrhea secondary to exocrine pancreatic insufficiency)
Superficial thrombophlebitis (in 10% of cases, also called Trousseau syndrome or thrombophlebitis migrans)
Recurring thrombophlebitis in various locations
Classically associated with pancreatic cancer
Thrombosis (e.g., phlebothrombosis, splenic vein thrombosis)
Impaired glucose tolerance (rarely)
The symptoms of pancreatic cancer may be similar to those of chronic pancreatitis. Differential diagnosis is difficult since carcinoma may be accompanied by pancreatitis!

P: Location
Pancreatic head: 75% of cases
Pancreatic body: 15–20% of cases
Pancreatic tail: 5–10% of cases
Pancreatic exocrine tumors (95%)
Mostly ductal adenocarcinoma
Less common: acinar adenocarcinoma (acinar cells produce digestive enzymes) and others
Pancreatic endocrine tumors (neuroendocrine tumors/NET, < 5% of tumors)
Insulinomas (result in hypoglycemia)
Gastrinomas
Vasoactive intestinal peptide-producing tumors (VIPomas), pancreatic polypeptide-secreting endocrine tumors of the pancreas, glucagonomas, somatostatinomas

I: Blood
No screening tests available
Tumor markers: CA 19-9 and CEA
Used to monitor the progression of cancer and treatment efficacy
Possibly ↑ lipase
Imaging
First test: usually either contrast-enhanced abdominal CT or ultrasound → if ultrasound reveals a pancreatic mass → subsequent CT
Poorly defined, hypodense/hypoechoic and hypovascular mass
Double-duct sign : With increasing size, tumors of the pancreatic head may block bile drainage in both the common bile duct and the pancreatic duct, leading to dilatation of both structures.
Endoscopic or magnetic retrograde cholangiopancreatography (ERCP/MRCP): to rule out choledocholithiasis and/or if biliary decompression is indicated, e.g., in case of palliative treatment to alleviate jaundice
Endoscopic ultrasound (EUS)
Used when other diagnostic tests are inconclusive or to perform fine needle aspiration
Findings similar to transcutaneous ultrasound
Fine needle aspiration
Not routinely performed
Can help differentiate pancreatic cancer from pancreatitis (e.g., chronic or autoimmune)
Can be done via EUS (preferred) or percutaneously (US or CT-guided).

M: As the only curative treatment option for pancreatic cancer is surgical resection, patients with operable tumors (∼ 20%) are always recommended for surgery. If surgical tumor resection is not possible or distant metastasis is present, a palliative approach is chosen.
Curative approach
Surgery
Pancreatic head carcinoma: pancreaticoduodenectomy (“Whipple procedure”; method of choice)
Resection of pancreatic head, distal stomach, duodenum, gallbladder, and common bile duct
Lymphadenectomy
Reconstruction by enteroenterostomy or Roux-en-Y anastomosis
Pylorus-preserving pancreaticoduodenectomy (Traverso-Longmire procedure): a modification of the Whipple procedure that preserves the gastric antrum, the pylorus and a small part of the duodenum (anastomosed to the jejunum) to provide a more physiologic stomach emptying
Pancreatic body and tail carcinoma
Resection of the left side of the pancreas with splenectomy
In some cases, duodenopancreatectomy with splenectomy
Indicated in a curative treatment approach if partial removal of the pancreas is insufficient.
Neoadjuvant or adjuvant chemoradiotherapy
To reduce tumor size, improve symptoms, and prolong life
Chemotherapy or radiation therapy without surgery cannot cure the patient.
Palliative approach
Palliative chemotherapy: indicated in patients with advanced or metastatic pancreatic cancer
Analgesia according to the WHO step-by-step plan
Early consultation with a pain therapist for optimizing the need of coanalgesics and adjuvants may be indicated.
Cholestasis : ERCP with stent implantation
Percutaneous transhepatic bile duct drainage (PTCD)
Indication: if endoscopic access path is complicated (e.g., in duodenal stenosis, duodenal resection, and inaccessibility of the biliodigestive anastomosis)
Gastroenterostomy: best supportive care in patients with gastric outlet stenosis. The stomach is anastomosed with the small intestine bypassing the duodenum.
Percutaneous endoscopic gastrostomy (PEG) tube as a relief tube: indicated for severe palliative patients with chronic ileus and subileus that are inoperable

P: Lymphogenic and hematogenous metastasis
Early stage: nearby lymph nodes and liver
Advanced stage: surrounding visceral organs (duodenum, stomach, colon) and lungs
Stenosis
Gastric outlet stenosis
Stenosis of the common bile duct (cholestasis)
Other complications
Secondary diabetes mellitus
Disseminated intravascular coagulation (DIC)
Necrolytic migratory erythema.

The overall 5-year survival rate is 7%, but mainly depends on the stage of disease.
Median survival for patients who undergo successful resection: ∼ 12–19 months, with a 5-year survival rate of 15–20%
A metastatic pancreatic cancer has a 5-year survival rate of ∼ 2%

32
Q

Inflammatory bowel disease-ulcerative colitis

A

Ulcerative colitis (UC) is an inflammatory bowel disease (IBD) characterized by chronic mucosal inflammation of the colon and cecum. Common symptoms include bloody diarrhea, abdominal pain, and fever. Laboratory findings typically show elevated inflammatory markers and the presence of autoantibodies (pANCA). Definitive diagnosis requires biopsies showing abnormal colonic mucosa and characteristic histopathology. Aminosalicylic acid derivatives are the mainstay of treatment, although severe episodes typically require corticosteroids and immunosuppressants to achieve remission. In the case of distal colitis, some drugs may be administered topically (e.g., via enema), whereas more proximal inflammation requires systemic treatment. Proctocolectomy is curative and indicated for complicated UC or dysplasia. Individuals with UC are predisposed to colorectal cancer and should thus undergo regular surveillance colonoscopy.

D: Crohn disease (see Differential diagnostic considerations: Crohn disease and ulcerative colitis)
Exudative-inflammatory diarrhea
Diverticular disease
Appendicitis
Ischemic colitis
Infectious colitis
C. difficile colitis
Shigella dysenteriae
Salmonella enterocolitis
Escherichia coli colitis
Campylobacter enterocolitis
Yersiniosis
Tuberculosis
CMV colitis
Radiation colitis
Celiac disease
Inflammatory diarrhea
Microscopic colitis: An idiopathic form of colitis that is characterized by a normal macroscopic appearance of bowel on colonoscopy and collagenous or lymphocytic infiltrates on microscopy.
Patients present with chronic, non-bloody diarrhea, weight loss, and abdominal pain.

R: Genetic predisposition (e.g., HLA-B27 association)
Ethnicity (white populations, individuals of Ashkenazi Jewish descent)
Family history of inflammatory bowel disease
Episodes of previous intestinal infection
Increased fat intake (esp. saturated fat and animal fat)
Oral contraceptive intake
NSAIDs may exacerbate UC

D:

E: Prevalence
Approx. 600,000 adults in the U.S. are affected by UC [1]
Ethnicity
Higher in the white than in the black, Hispanic, or Asian populations
Highest among individuals of Ashkenazi Jewish descent.
Slightly higher in men than women [2]
Peak incidence
15–35 years [3]
Another smaller peak may be observed in individuals > 55 years.

A: Gross pathology
Early stages
Inflamed, erythematous, edematous mucosa
Friable mucosa with bleeding on contact with endoscope
Fibrin-covered ulcers
Small mucosal ulcerations
Loss of superficial vascular pattern
Chronic disease
Loss of mucosal folds
Loss of haustra
Strictures
Pseudopolyps
Raised areas of normal mucosal tissue that result from repeated cycles of ulceration and healing
Ulceration → formation of granulation tissue → deposition of granulation tissue → epithelization
Morphologically resemble polyps but do not undergo neoplastic transformation
Found in advanced disease
In ulcerative colitis, the extent of intestinal inflammation is limited to the mucosa and submucosa. In contrast, Crohn disease shows a transmural pattern of intestinal involvement.
Histological findings
Early stages
Granulocyte (neutrophil) infiltration: limited to mucosa and submucosa
Crypt abscesses: an infiltration of neutrophils into the lumen of intestinal crypts due to a breakdown of the crypt epithelium
Chronic disease
Lymphocyte infiltration
Mucosal atrophy
Altered crypt architecture
Branching of crypts
Irregularities in size and shape
Epithelial dysplasia
Noncaseating granulomas are seen in Crohn disease but are not a feature of ulcerative colitis!

C: Intestinal symptoms
Bloody diarrhea with mucus
Fecal urgency
Abdominal pain and cramps
Tenesmus
Extraintestinal symptoms
Skeletal (most common extraintestinal manifestation of ulcerative colitis): osteoarthritis, ankylosing spondylitis, sacroiliitis [9][10]
Ocular: uveitis, episcleritis, iritis
Biliary: primary sclerosing cholangitis (PSC)
Rare for patients with UC to develop PSC, but up to 90% of all patients affected by PSC will also be affected by UC.
Cutaneous: erythema nodosum, pyoderma gangrenosum, aphthous stomatitis,
General: fatigue, fever
In children/adolescents: growth retardation and delayed puberty.
Primary sclerosing cholangitis (PSC) is often associated with inflammatory bowel disease, especially UC. However, only around 4% of people with inflammatory bowel disease develop PSC!
To remember the characteristics of ulcerative colitis, think “ULCCCERS” for Ulcers, Large intestine, Continuous/Colon cancer/Crypt abscesses, Extends proximally, Red diarrhea, Sclerosing cholangitis.

P: Pathogenesis
The exact mechanism is unknown but studies suggest that ulcerative colitis is the result of abnormal interactions between host immune cells and commensal bacteria. [8][2]
Dysregulation of intestinal epithelium: increased permeability for luminal bacteria → activation of macrophages and dendritic cells → antigen presentation to macrophages and naive CD4+ cells leads to
Secretion of pro-inflammatory cytokines (IL-6, IL-12, TNF-α) and chemokines (CXCL1, CXCL3, and CXCL8) → recruitment of other immune cells (e.g., neutrophils) to the site
Differentiation of naive CD4+ cells to Th2 effector cells
Recruitment of NK cells
Dysregulation of the immune system: upregulation of lymphatic cell activity in bowel walls (T cells, B cells, plasma cells) → enhanced immune reaction and cytotoxic effect on colonic epithelium → inflammation with local tissue damage (ulcerations, erosions, necrosis) in the submucosa and mucosa
Autoantibodies (pANCA) against cells of the intestinal epithelium
Th2 cell-mediated response
Pattern of involvement
Ascending inflammation beginning in the rectum and spreading continuously proximally throughout the colon
Mucosal and submucosal inflammation
The rectum is always involved in UC!
Risk factors [8][2][4]
Genetic predisposition (e.g., HLA-B27 association)
Ethnicity (white populations, individuals of Ashkenazi Jewish descent)
Family history of inflammatory bowel disease
Episodes of previous intestinal infection
Increased fat intake (esp. saturated fat and animal fat)
Oral contraceptive intake
NSAIDs may exacerbate UC
Protective factors [8][2]
Appendectomy
Smoking has a protective effect.

I: Laboratory tests
Blood
↑ ESR, ↑ CRP, leukocytosis
Anemia
Thrombocytosis in some cases
↑ Perinuclear ANCA (pANCA)
Medium sensitivity but high specificity [14][15]
No correlation between titer and disease activity
Serologic testing is not recommended for definitive diagnosis or exclusion of UC but can support the diagnosis. [3]
In case of concurrent PSC: elevated gamma-glutamyl transferase
Stool analysis
Test for bacteria to rule out infectious causes. [3]
Calprotectin and lactoferrin are indicators of mucosal inflammation.
Endoscopy
Endoscopy (e.g., colonoscopy) with histological examination is considered the best test to definitively diagnose UC.
Typical findings: see “Gross pathology” below
Pattern of disease involvement [3]
Proctosigmoiditis: limited to the rectum, with possible sigmoid involvement
Left-sided colitis: extends distally to the splenic flexure
Extensive colitis: extends beyond the splenic flexure
Recommendations [3]
Evaluate the ileum to rule out Crohn disease
Stepwise biopsy (for findings, see “Pathology” below)
Colonoscopy is contraindicated in patients with acute flare because of the high risk of perforation but should be performed once symptoms improve.
Sigmoidoscopy may be considered as an alternative.
Imaging [16][3]
Imaging studies may serve as useful adjunct diagnostic procedures for UC, particularly when it comes to detecting complications.
Radiography
Plain radiography
Typically normal in mild to moderate disease
Findings
Loss of colonic haustra (lead pipe appearance) may be seen in severe cases
Massive distention in cases of toxic megacolon
Pneumoperitoneum in cases of perforation
Barium enema radiography
Able to detect very early changes
Findings
Granular appearance of the mucosa
Deep ulcerations
Loss of haustra
Pseudopolyps that appear as filling defects
CT: Detection of bowel wall thickening is possible in severe disease.
MRI: can be helpful in assessing disease severity and extent of bowel wall involvement
Ultrasound: can detect bowel wall thickening (manifests with absent hyperechoic reflection from the lumen.

M: Initially, UC is treated conservatively with drugs to induce and maintain disease remission. Curative proctocolectomy is generally indicated if medical therapy fails or complications arise.
General management
Rehydration
Supplementation of nutritional deficiencies (e.g., iron)
Supplementation of nutrition: severe cases may warrant consideration of a feeding tube or parenteral nutrition.
Medical therapy [3]
Supportive care
Antidiarrheal agents (e.g., loperamide)
Anticholinergic medication (e.g., propantheline, dicyclomine): relieves abdominal cramping
NSAIDs, opioids, and anticholinergics should be avoided in severe disease.
Recommended medical therapy by the severity of disease [3]
Mild disease
5-aminosalicylic acid derivatives (5-ASAs)
Anti-inflammatory and immunosuppressive effects on the bowel
Can be administered orally, as suppositories, or as enemas
Examples: mesalamine, sulfasalazine , olsalazine
If no improvement or 5-ASA agents are not tolerated
Topical corticosteroids (e.g., budesonide)
Oral systemic corticosteroids
Moderate disease
Oral and topical 5-ASAs
Topical corticosteroids (e.g., budesonide) → systemic corticosteroids only if no response
Anti-TNF therapy (adalimumab, golimumab, or infliximab)
Severe or refractory disease
High‑dose oral and topical 5-ASAs
Systemic corticosteroids
Anti-TNF therapy (e.g., adalimumab, golimumab, or infliximab)
Calcineurin antagonists (e.g., cyclosporine, tacrolimus)
Thiopurines (e.g., azathioprine) may be considered but are no longer recommended as monotherapy due to lack of efficacy [3]
Referral for surgical proctocolectomy (see below)
Systemic corticosteroids should only be used for the treatment of an active flare and are not recommended as a maintenance medication for ulcerative colitis!
Surgical intervention
Goal
Curative approach with full recovery
Reduce risk of colorectal cancer
Indications
Emergent: Acute complications despite adequate conservative management (e.g., toxic megacolon, perforation, sepsis, uncontrolled bleeding, etc.)
Elective: epithelial dysplasia, severe relapses, long-term dependence on steroids, impairment of the patient’s general condition
Procedure: proctocolectomy with an ileal pouch-anal anastomosis (IPAA or J pouch)
Resection of the entire colon and rectal mucosa while sparing the anal sphincters.
Loops of small intestine (serving as the pouch) are used to create an artificial rectum (reservoir for feces) and thus a continence-conserving connection between the ileum and anus.
In contrast to Crohn disease, ulcerative colitis can be cured surgically (proctocolectomy)!

P: Gastrointestinal bleeding (both acute and chronic)
Toxic megacolon
Perforation → peritonitis
Fulminant colitis: A condition of severe bowel inflammation that typically causes > 10 stools per day, lower gastrointestinal bleeding, abdominal pain, and abdominal distension.
↑ Risk of cancer (see colorectal carcinoma)
Risk increases with increased duration and/or extent of disease
Risk is not significantly increased in patients with mild UC
Prevention: Screening colonoscopy with biopsies every 1–3 years starting 8 years after the initial diagnosis to screen for colorectal cancer [3]
Colonic stenosis
Amyloidosis.
On average, the life expectancy of patients with UC is normal.

33
Q

Crohn disease -Inflammatory bowel disease IBD

A

D: Crohn disease (CD) is an inflammatory bowel disease (IBD), the pathogenesis of which is not fully understood. The clinical presentation of CD may be similar to ulcerative colitis (UC), the other most common IBD. CD mostly affects young adults and adolescents between the ages of 15 and 35. It typically affects the terminal ileum, but can discontinuously affect the entire gastrointestinal tract and commonly leads to complications such as fistulas, abscesses, and stenosis. Clinical features include diarrhea, weight loss, and abdominal pain in the right lower quadrant (RLQ), as well as extraintestinal manifestations in the eyes, joints, or skin. Diagnosis is based on the patient’s medical history, physical examination, lab tests, imaging (e.g., MRI), endoscopy, and serological testing. Acute episodes are treated with corticosteroids; immunosuppressants may be indicated in severe cases. Antibiotics and surgical intervention may be needed to help treat complications. As Crohn disease is not localized to a specific region of the GI tract, surgical resection is not a curative option (unlike in UC), and treatment instead focuses on limiting the progression and recurrence of inflammatory episodes.

R:

D: Ulcerative colitis. 
Other differential diagnoses
Acute appendicitis
Infectious gastroenteritis/colitis
Noninfectious colitis (ischemic, after radiation therapy, after ingestion of drugs, etc.)
Diverticulitis
Irritable bowel syndrome
Gastrointestinal tuberculosis
Malignant intestinal transformations

E: Prevalence: 200 cases per 100,000 population
Incidence: ∼ 6 cases per 100,000 population per year [1]
Sex: ♂ = ♀
Typical age of onset: bimodal distribution with one peak at 15–35 years and another one at 55–70 years [2][3]
Populations with higher prevalence [4]
Individuals of Northern European descent
Individuals of Ashkenazi Jewish descent.

A: Cause: Immune dysregulation and dysbiosis, which promotes chronic inflammation, the ultimate cause of which is not fully understood.
Risk factors [4]
Active and passive smoking of tobacco
Familial aggregation
Genetic predisposition (e.g., mutation of the NOD2 gene, HLA-B27 association)
Nicotine consumption is the only (known) controllable risk factor for Crohn disease. Therefore, smoking cessation is especially important in patients with CD.

C: CD typically occurs episodically with a 30%-risk of recurring inflammation over the span of one year. If symptoms persist for six months, the disease is considered chronic. Without treatment, relapses and complications are to be expected.
Constitutional symptoms [6]
Low-grade fever
Weight loss
Fatigue
Intestinal symptoms [6]
CD most commonly affects the terminal ileum and colon, but involvement of any part of the GI tract (from mouth to anus, except rectum) is possible.
Chronic diarrhea, typically nonbloody
Abdominal pain, typically in the RLQ
Malabsorption (see “Signs of malabsorption” in “Complications” below)
Palpable abdominal mass in the RLQ
Enterocutaneous perianal fistulas, often associated with abscess formation [7]
Oral aphthae
Perianal fistulas and abscesses are often the first signs of Crohn disease.
Crohn disease
Extraintestinal symptoms [8]
Joints: enteropathic arthritis (e.g., sacroiliitis, spondylitis, inflammation of peripheral joints)
Eyes
Uveitis
Iritis
Episcleritis
Liver/bile ducts: cholelithiasis
Urogenital system: urolithiasis (mostly calcium oxalate stones)
Skin
Erythema nodosum
Acrodermatitis enteropathica
Pyoderma gangrenosum
Associated with various conditions (e.g., IBD, rheumatoid arthritis, and trauma)
Manifests with very painful, rapidly-progressive, red spots that can change into purulent pustules or deep ulcerated lesions with central necrosis
Commonly located at extensor side of the lower limbs
Treated with immunosuppressants (e.g., corticosteroids, cyclosporine A)
Pyostomatitis vegetans

P: Inflammation
Inflammation is most likely caused by immune dysregulation.
Dysregulation of IL-23-Th17 signaling → unrestrained Th17 cell function → inflammation → local tissue damage (edema, erosions/ulcers, necrosis) → obstruction, fibrotic scarring, stricture, and strangulation of the bowel [5]
There is evidence that mutations in the nucleotide oligomerization binding domain 2 (NOD2) protein are involved in the development of Crohn disease, but the exact mechanism is not fully understood. [5]
Loss of function mutations in NOD2 → ↑ susceptibility for bacterial invasion of the intestinal mucosa → unregulated inflammation
Dysfunctional NOD2 → overactivity of NF-κB signaling pathway → ↑ production of proinflammatory cytokines and antimicrobial peptides → chronic autoinflammation
Abscess and fistula formation
Intestinal aphthous ulcers → transmural fissures and inflammation of the intestinal walls → adherence of other organs or the skin → penetration → microperforation and abscess formation → macroperforation into these structures → fistula formation

I: Approach
Diagnosing CD requires the integration of clinical presentation, laboratory tests, and endoscopic, histologic, pathologic and radiologic findings.
If a patient presents with symptoms suggestive of CD, conduct blood tests and stool tests (see “Laboratory tests” below) to rule out other possible causes for bowel inflammation/GI symptoms. [9]
Confirm diagnosis with endoscopy and/or radiographic imaging and/or biopsy.
Perform contrast radiological studies and/or ultrasonography to assess extent, severity, and complications (e.g., abscesses, fistulas, and stenoses)
Laboratory tests
Blood [9]
Blood work
Complete blood count may show signs of pernicious anemia
↑ Inflammatory markers (↑ CRP, ↑ ESR, ↑ thrombocytes, and ↑ leukocytes)
Serology: routine use to establish diagnosis is not recommended due to low sensitivity [10]
↑ Anti-Saccharomyces cerevisiae antibodies (ASCA)
pANCA most likely negative
Stool
Stool analysis
Stool culture to rule out bacterial gastroenteritis
Microscopy to examine presence of worm larvae or eggs (ova and parasites)
Identification of bacterial toxins (e.g., toxin of Clostridium difficile)
Detection of fecal calprotectin and/or fecal lactoferrin [11][12]
Proteins associated with neutrophils that can be used as a diagnostic tool for inflammatory bowel disease
Direct correlation between detected amount of proteins and severity of intestinal inflammation
Fecal occult blood test (FOBT)
Imaging [13][14]
Plain x-ray abdomen: may show bowel distention or pneumoperitoneum
Upper GI series with barium swallow and small bowel follow-through (enteroclysis): used to detect fistulas or stenoses, characteristic findings are:
Procedure
Water-soluble contrast medium is inserted into the small intestine via a nasopharyngeal tube.
Multiple x-rays are taken in a chronological sequence to evaluate each section.
Findings
String sign: contrast-filled bowel segment that resembles a string on x-ray
Creeping fat: pathognomonic hyperplasia of adipose tissue that results in accumulation of mesenteric fat around the circumference of the intestine [15]
May cover more than 50% of the inflamed bowel
Correlates with severity of transmural inflammation
Ultrasound findings
Gastrointestinal wall thickening caused by inflammation and edema
Possible detection of abscesses/fistulas
MR enterography: noninvasive, highly sensitive and specific imaging technique that involves the visualization of an oral contrast medium on MRI and is used in the diagnosis of IBD.
Used to assess the extent and pattern of intestinal inflammation, detect perianal and pelvic disease, and to predict disease activity
Characteristic findings are an edematous thickening of the intestinal wall and enlarged lymph nodes.
Can be done as invasive MRI enteroclysis, during which contrast medium is applied via nasoduodenal tube and the small bowel is distended via an electric infusion pump.
Crohn diseaseStenosis of the terminal ileum
Endoscopy [16]
Endoscopy confirms the diagnosis, assesses the extent of the disease, differentiates CD from other diseases (e.g., ulcerative colitis, peptic ulcers, etc.), and may also be used as a therapeutic tool (e.g., dilatation of ducts, intestinal loops).
Ileocolonoscopy: endoscopic examination of the rectum, colon, and terminal ileum that allows for direct visualization of the intestinal mucosa and sampling of tissue
Procedure: ileocolonoscopy with biopsies at various locations throughout the terminal ileum, colon, and rectum
Characteristic macroscopic findings
Segmental/discontinuous pattern of involvement
Snail trails: longitudinal ulcerations
Pinpoint lesions: small, aphthous hemorrhagic mucosal defects
Cobblestone sign: inflamed sections followed by deep ulcerations that resemble cobblestones
Erythema and transmural inflammation (all mucosal layers of the intestinal wall are involved)
Fissures, fistulas
Esophagogastroduodenoscopy
Used to evaluate for involvement of the esophagus, stomach, and duodenum
Findings include aphthae on mucosa
Video capsule endoscopy [13]
Used to evaluate small bowel morphology as an adjunctive to regular endoscopy
Should be performed prior to regular endoscopy in suspected small bowel obstruction.

M: Approach [13]
Therapy of CD is based on the following steps:
Treating acute disease
Inducing clinical remission
Maintaining response/remission
Patients should be stratified according to their specific prognostic risk factors.
For optimal results, therapy should be as individually tailored as possible.
Disease activity should be monitored regularly based on objective markers.
Patients should be motivated to engage in lifestyle modifications (see below).
Pharmacotherapy
Treatment of Crohn disease can be approached in two different ways: step-up therapy and top-down therapy.

Surgery
Overview
Minimally-invasive resection of affected and nonfunctional intestinal loops while preserving as much intestinal length and function as possible
Indicated when medical therapy fails or patient develops severe complications (e.g., obstruction, stricture, abscess)
Methods
Balloon dilatation: to treat intestinal stenosis
Percutaneous drainage: prevents retention of secretions and abscessation
Surgical drainage: when application of percutaneous drainage fails
Strictureplasty
A surgical procedure that opens up a bowel stricture without having to resect the bowel (bowel-sparing technique)
Indicated after multiple resections
Limited resection (e.g., proctocolectomy): in case of obstructions or strictures
Crohn disease is mainly treated with medication, but surgical interventions may be required to treat complications or if medical therapy fails.
Surgical intervention alone cannot cure Crohn disease and should therefore be considered as a last resort to avoid complications in which significant amounts of bowel are lost (e.g., short bowel syndrome)!
Additional considerations
Lifestyle modifications
Smoking cessation
Avoiding certain drugs (e.g., NSAIDs )
Minimizing stress
Management of complications and comorbidities
Malabsorption syndrome: appropriate substitution of vitamins, calories, protein, zinc, calcium, and iron
Bile acid diarrhea: administration of ion-exchange resins to bind bile acids (e.g., cholestyramine)
Depression and anxiety: See “Major depressive disorder” and “Generalized anxiety disorder.”
Prevention of malignancies
Individuals with CD have an increased risk of cancer (especially of the small intestine, colon , and lymphatics).
Regular colonoscopies should be performed to minimize risk.
People without major colonic involvement are managed according to the general screening guidelines (see ”Screening for colorectal cancer”)
Nutrition
Enteral nutrition always take preference over parenteral
If oral food intake is not possible, nasogastric or nasoenteric tube should be used
Secondary lactose intolerance (approx. 30% of cases): lactose-free diet [22]
During acute episodes: avoid dietary fiber

NOTES

P: CD is a chronic disease that is currently not curable.
Life expectancy is normal with proper treatment. [25]
70–90% of all patients will require surgery at some point during their lifetime.
complications: Intestinal complications
Colorectal cancer (especially in the case of pancolitis)
Short bowel syndrome and associated issues after surgery
Stenosis/strictures → bowel obstruction/(sub)ileus
Intestinal perforation → peritonitis
Primary sclerosing cholangitis
Impaired bile acid reabsorption
Bile acid diarrhea
Bile acid malabsorption → steatorrhea and deficiencies in fat-soluble vitamins
Abscess formation/phlegmons: See below.
Intestinal fistulas and abscesses [23]
Overview
Typically involve the terminal ileum and/or perianal region
Recurrences are common
See “Anal abscess and fistula.”
Etiology
Iatrogenic (e.g., abdominal surgery, percutaneous drainage)
Inflammation (e.g., inflammatory bowel disease, appendicitis)
Pathophysiology [24]
Inflammation → epithelial defects → epithelial-mesenchymal transition → deeper penetration of cell layers by epithelial cells → tissue damage that organizes as tubular structures that ultimately connect to other organs or the surface
After surgery or percutaneous drainage: deficient anastomoses/sutures or improper healing following intervention (e.g., due to reduced organ blood flow) → leakage of intestinal contents → local infection → abscess formation and/or erosion → fistula formation
Clinical features: depend on location of the fistula
Enterovesical/colovesical fistula → pneumaturia (passing of urine together with air)/fecaluria (passing of stool together with urine) → recurrent urinary tract infections (UTIs)
Enterocutaneous fistula → drainage of intestinal content through the skin
Gastrocolic fistula → abdominal pain, weight loss, foul‑smelling (feculent) belching
Systemic complications
Signs of malabsorption syndrome
Weight loss
Failure to thrive and growth failure in children
Anemia
Iron deficiency anemia
Anemia of chronic disease
Pernicious anemia (due to impaired vitamin B12 absorption in the chronically inflamed ileum)
Osteoporosis .
Amyloidosis

Pathology: Skip lesions: a pattern of patchy, discontinuous inflammation in the bowel (affected areas interspersed with normal tissue)
Creeping fat
Hypertrophic lymph nodes
Transmural inflammation
Noncaseating granulomas [17]
Giant cells
Distinct lymphoid aggregates of the lamina propria

34
Q

Signs of chronic liver disease

A

A constellation of 8 signs associated with liver disease: jaundice, scleral icterus, spider angioma, gynecomastia, palmar erythema, ascites, asterixis, and encephalopathy .

35
Q

colonic polyps

A

Colonic polyps are abnormal colonic mucosal overgrowths. They are a common finding in people over the age of 50. In rare cases, they may be seen in younger individuals as part of hereditary polyposis syndromes (e.g., familial adenomatous polyposis, Peutz‑Jeghers syndrome, etc.). On macroscopic examination, colonic polyps are either pedunculated (with a stalk) or sessile (without a stalk). Histologically, colonic polyps are most commonly adenomas (∼ 70%), followed by hyperplastic polyps, and, rarely, serrated polyps and hamartomatous polyps. Colonic polyps may be benign or have malignant potential. Adenomas have the highest malignant potential (5%) and most colonic carcinomas arise from them (adenoma‑carcinoma sequence). Most patients are asymptomatic, but they may present with altered bowel habits, blood/mucus in stool, and iron deficiency anemia. Colonoscopy is the diagnostic modality of choice, since it allows for direct visualization of the polyps, biopsy, and therapeutic snare polypectomy (of pedunculated polyps) or endoscopic submucosal resection (of sessile polyps). Large polyps (> 2 cm) or malignant polyps require surgical resection.

36
Q

Colonic polyps DR DEAC PIMP

A

Epidemiology
Incidence increases with age: ∼ 30% of individuals > 50 years .
Frequency
∼ 70% are adenomatous polyps
∼ 20% are hyperplastic polyps
< 10% are other kinds of polyps (traditional serrated adenomas, sessile serrated adenomas, and mixed mucosal polyps).
Sex: ♂ > ♀

Aetiology
The exact etiology is unknown.
Risk factors
Lifestyle
Diet: high in red meat and fat; low in fiber and folic acid 
Obesity and lack of exercise
Cigarette smoking
Alcohol consumption
Genetic predisposition
Black populations 
Hereditary polyposis syndromes

Classification on word document.

Clinical features
Mostly asymptomatic
If symptomatic
Blood in stool (hematochezia) is the most common symptom.
Change in bowel habits (constipation/diarrhea)
Mucus in stool
Physical examination
Pallor
Digital rectal examination: for rectal polyps

Subtypes and variants on doc.

Diagnostics
Laboratory tests
Fecal occult blood test
Hemoglobin levels: Anemia
Genetic studies: in patients with a strong family history of polyposis syndromes
Imaging
Virtual colonoscopy:
Air contrast barium enema: can detect left-sided colonic polyps
Flexible sigmoidoscopy
Advantage: does not require extensive bowel preparation before the procedure.
Disadvantage: The rest of the colon is not visualized.
Colonoscopy (confirmatory test)
Preferred screening tool for diagnosed cases of hereditary polyposes/colonic cancer
Advantages: Enables biopsy, histological confirmation, and therapeutic polypectomy (see “Classification” above)
Disadvantages: Requires bowel preparation with laxatives; Risk of bowel perforation.

Treatment
Treatment depends on the etiology and nature of the polyp or polyps.
Snare polypectomy: of pedunculated polyps (< 2 cm in size)
Endoscopic mucosal resection: large sessile polyps
Surgical resection
For large polyps (> 2 cm)
For suspected malignancy: see colorectal cancer
Hereditary polyposis syndromes
For follow‑up, see colorectal cancer prevention.

37
Q

Acute pancreatitis Dr deac pimp

A

Acute pancreatitis is an inflammatory condition of the pancreas most commonly caused by biliary tract disease or alcohol abuse. Damage to the pancreas causes local release of digestive proteolytic enzymes that autodigest pancreatic tissue. Acute pancreatitis usually presents with epigastric pain radiating to the back, nausea and vomiting, and epigastric tenderness on palpation. The diagnosis is made based on the clinical presentation, elevated serum pancreatic enzymes, and findings on imaging (CT, MRI, ultrasound) that suggest acute pancreatitis. Treatment is mostly supportive and includes bowel rest, fluid resuscitation, and pain medication. Enteral feeding is usually quickly resumed once the pain and inflammatory markers begin to subside. Interventional procedures may be indicated for the treatment of underlying conditions, such as ERCP or cholecystectomy in gallstone pancreatitis. Localized complications of pancreatitis include necrosis, pancreatic pseudocysts, and abscesses. Systemic complications involve sepsis, ARDS, organ failure, and shock and are associated with a considerable rise in mortality.

Aetiology:
Most common causes
Biliary pancreatitis (e.g., gallstones, constriction of the ampulla of Vater) ∼ 40% of cases 
Alcohol-induced (∼ 30% of cases)
Idiopathic (∼ 15%–25% of cases)
Other causes
Severe hypertriglyceridemia (> 1,000 mg/dl), hypercalcemia
Post-ERCP 
Certain drugs
Steroids
Azathioprine
Sulfonamides
Loop and thiazide diuretics
Estrogen
Protease inhibitors
NRTIs
Anticonvulsants
Scorpion stings
Viral infections (e.g., coxsackievirus B, mumps)
Trauma (especially in children)
Autoimmune and rheumatological disorders (e.g., Sjögren syndrome)
Pancreas divisum 
Hereditary (e.g., mutation of the trypsinogen gene, cystic fibrosis)
To remember the most common causes of acute pancreatitis, think: "I GET SMASHED" (Idiopathic, Gall stones, Ethanol, Trauma, Steroids, Mumps, Autoimmune, Scorpion poison, Hypercalcemia, Hypertriglyceridemia, ERCP, Drugs).

Pathophysiology:
Mechanisms of development
Sequence of events leading to pancreatitis
Intrapancreatic activation of pancreatic enzymes: secondary to pancreatic ductal outflow obstruction (e.g., gallstones, cystic fibrosis) or direct injury to pancreatic acinar cells (e.g., alcohol, drugs)
Increased proteolytic and lipolytic enzyme activity → destructionof pancreatic parenchyma
Attraction of inflammatory cells (neutrophils, macrophages) → release of inflammatory cytokines → pancreatic inflammation (pancreatitis)
Sequelae of pancreatitis (depending on the severity of pancreatitis)
Capillary leakage: release of inflammatory cytokines and vascular injury by pancreatic enzymes → vasodilation and increased vascular permeability → shift of fluid from the intravascular space into the interstitial space (third space loss) → hypotension, tachycardia → distributive shock
Pancreatic necrosis: uncorrected hypotension and third space loss → decreased organ perfusion → multiorgan dysfunction (mainly renal) and pancreatic necrosis
Hypocalcemia: lipase breaks down peripancreatic and mesenteric fat → release of free fatty acids that bind calcium → hypocalcemia (fatty saponification) [4]
Fulminant acute pancreatitis.

Clinical features
Constant, severe epigastric pain
Classically radiating towards the back
Worse after meals and when supine
Improves on leaning forwards
Nausea, vomiting
General physical examination
Signs of shock: tachycardia, hypotension, oliguria/anuria
Possibly jaundice in patients with biliary pancreatitis
Abdominal examination
Abdominal tenderness, distention, guarding
Ileus with reduced bowel sounds and tympany on percussion
Ascites
Skin changes (rare)
Cullen’s sign: periumbilical ecchymosis and discoloration (bluish-red)
Grey Turner’s sign: flank ecchymosis with discoloration
Fox’s sign: ecchymosis over the inguinal ligament.

Diagnostics
Acute pancreatitis is diagnosed based on a typical clinical presentation, with abdominal pain radiating to the back, and either detection of highly elevated pancreatic enzymes or characteristic findings on imaging. Serum hematocrit is an easy test that should be conducted to help quickly predict disease severity.
Laboratory tests
Tests to confirm clinical diagnosis
↑ Serum pancreatic enzymes
Lipase: if ≥ 3 x the upper reference range → highly indicative of acute pancreatitis
Amylase (nonspecific)
The enzyme levels are not directly proportional to severity or prognosis!
Tests to assess severity
Hematocrit (Hct)
Should be conducted at presentation as well as 12 and 24 hours after admissions
↑ Hct (due to hemoconcentration) indicates third space fluid loss and inadequate fluid resuscitation
↓ Hct indicates the rarer acute hemorrhagic pancreatitis
WBC count
Blood urea nitrogen
↑ CRP and procalcitonin levels
↑ ALT
Tests to determine etiology
Alkaline phosphatase, bilirubin levels (evidence of gallstone pancreatitis)
Serum calcium levels
Serum triglyceride levels (fasting)
Determining calcium values is very important: Hypercalcemia may cause pancreatitis, which may then, in turn, cause hypocalcemia!
Imaging
Ultrasound (most useful initial test): indicated in all patients with acute pancreatitis
Main purpose: detection of gallstones and/or dilatation of the biliary tract (indicating biliary origin)
Signs of pancreatitis
Indistinct pancreatic margins (edematous swelling)
Peripancreatic build-up of fluid ; evidence of ascites in some cases
Evidence of necrosis, abscesses, pancreatic pseudocysts
CT scan: not routinely indicated
Indications
At admission: only when the diagnosis is in doubt (e.g., not very highly elevated pancreatic enzymes, non-specific symptoms)
> 72 hours of symptom onset: if complications such as necrotizing pancreatitis or pancreatic abscess (e.g., persistent fever and leukocytosis, no clinical improvement or evidence of organ failure > 72 hours of therapy) are suspected
Findings
Enlargement of the pancreatic parenchyma with edema; indistinct pancreatic margins with surrounding fat stranding
Necrotizing pancreatitis: lack of parenchymal enhancement or presence of air in the pancreatic tissue
Pancreatic abscess: circumscribed fluid collection
MRCP and ERCP
Indications: suspected biliary or pancreatic duct obstructions
MRCP is noninvasive but less sensitive than ERCP
ERCP can be combined with sphincterotomy and stone extraction; but may worsen pancreatitis.
Conventional x-ray
Sentinel loop sign: dilatation of a loop of small intestine in the upper abdomen (duodenum/jejunum)
Colon cut off sign: gaseous distention of the ascending and transverse colon that abruptly terminates at the splenic flexure
Evidence of possible complications: pleural effusions, pancreatic calcium stones; helps rule out intestinal perforation with free air.

Ddx:
Intestinal manifestations
Acute peritonitis
Appendicitis
Acute mesenteric ischemia
Acute cholecystitis
Acute cholangitis
Peptic ulcer disease
Biliary colic
Abdominal aorta aneurism
Extraintestinal manifestations
Myocardial infarction
Bacterial pneumonia

Treatment
General measures
Admission to hospital and assessment of disease severity (consider ICU admission)
Fluid resuscitation: aggressive hydration with crystalloids (e.g., lactated Ringer’s solution , normal saline)
Analgesia: IV opioids (e.g., fentanyl)
Bowel rest (NPO)and IV fluids are recommended until the pain subsides
Nasogastric tube insertion: not routinely recommended; indicated in patients with vomiting and/or significant abdominal distention
Nutrition
Begin enteral feeding (oral/nasogastric/nasojejunal) as soon as the pain subsides
Total parenteral nutrition: only in patients who cannot tolerate enteral feeds (e.g., those with persistent ileus and abdominal pain)
Drug therapy
Analgesics: fentanyl or hydromorphone; consider pump administration (patient controlled analgesia = PCA)
Antibiotics
Prophylactic antibiotic therapy is not recommended.
Antibiotics should only be used in patients with evidence of infected necrosis.
Fenofibrates: in hyperlipidemia-induced acute pancreatitis
Procedures/surgery
Biliary pancreatitis
Urgent ERCP and sphincterotomy (within 24 hours): in patients with evidence of choledocholithiasis and/or cholangitis; followed by cholecystectomy
Cholecystectomy (preferably during same admission once the patient is stabilized; or within 6 weeks): in all patients with biliary pancreatitis
The most important therapeutic measure is adequate fluid replacement (minimum of 3–4 liters of crystalloids per day)!
“PANCREAS” - Perfusion (fluid replacement), Analgesia, Nutrition, Clinical (observation), Radiology (imaging), ERC (endoscopic stone extraction), Antibiotics, Surgery (surgical intervention, if necessary).

Complications:
Localized
Bacterial superinfection of necrotic tissue → fever
Diagnosis: CT-guided percutaneous drainage + culture of the aspirate
Treatment: surgical debridement, antibiotics
High mortality rate; multiple organ failure in ∼ 50% of cases
Pancreatic pseudocysts
Pancreatic abscess
Walled-off infected necrotic tissue or pancreatic pseudocyst; typically develops > 4 weeks after an attack of acute pancreatitis
Abdominal CT: visible contrast-enhanced abscess capsule with evidence of fluid (pus)
Ultrasound: complex cystic, fluid collection with irregular walls and septations
Treatment: cannulation and drainage; necrosectomy if other measures are not effective
Pleural effusion
Abdominal compartment syndrome
Blood vessel erosion with bleeding
Systemic
SIRS, sepsis, DIC
Pneumonia, respiratory failure, ARDS
Shock
Prerenal failure due to volume depletion
Hypocalcemia
Pleural effusion, pancreatic ascites
Paralytic ileus.

Prognosis:
Mortality
In patients without organ failure: < 1%
In patients with organ failure: ∼ 30%
Higher mortality in patients with biliary pancreatitis than in patients with alcoholic pancreatitis
Important predictors of severity
Age > 55
Gastrointestinal bleeding
Abnormal hematocrit within 48 hours
Acute hemorrhagic pancreatitis: ↓ Hct
Third space fluid loss: ↑ Hct
Hypocalcemia and/or hyperglycemia
Inflammatory markers: ↑↑ CRP, ↑ IL-6, ↑ IL-8
Evidence of shock and/or organ failure
↑ AST, ↑ ALT
↑ BUN, creatinine
↑ LDH
ABG: pO2 < 60 mmHg, metabolic acidosis with a base deficit > 4 mmol/L
CT findings: pancreatic edema, peripancreatic fluid collection, and/or necrosis of > 33% of the pancreas
Amylase and lipase, which are used for the diagnosis of pancreatitis, cannot be used to predict the prognosis!
Numerous scoring systems exist (e.g, Ranson criteria) for assessing the severity and predicting the prognosis of acute pancreatitis

38
Q

Chronic pancreatitis

A

Chronic pancreatitis is caused by progressive inflammation and irreversible damage to the structure and function (exocrine and endocrine) of the pancreas. Alcohol abuse is the most common known etiological factor, followed by pancreatic ductal obstruction. Idiopathic pancreatitis accounts for up to 30% of cases. Patients may be asymptomatic or present with abdominal pain and features of pancreatic enzyme insufficiency (e.g., steatorrhea, weight loss, impaired glucose tolerance). Diagnosis is confirmed on imaging, which demonstrates pancreatic calcifications, ductal strictures, and ductal dilations. Pancreatic function tests (e.g., fecal elastase-1 measurement, 72-hour fecal fat estimation) assess the degree of enzyme deficiency. Symptomatic patients are successfully managed with oral pancreatic enzyme replacements and analgesics. Patients with chronic pain require additional interventions (e.g., celiac ganglion block, partial/complete pancreatic resection).

Aetiology
Alcohol abuse (60–70% of cases, esp. men) ,
Pancreatic ductal obstruction (< 10%): strictures (e.g., due to trauma, stones)
Tobacco use
Idiopathic pancreatitis (20–30%)
Hereditary pancreatitis (∼1%): autosomal dominant inheritance (PRSS1 gene mutation), age of onset < 20 years
Autoimmune pancreatitis
Systemic disease
Severe hypertriglyceridemia (levels > 1000mg/dl)
Primary hyperparathyroidism (hypercalcemia)
Cystic fibrosis: ∼ 2% of cystic fibrosis patients develop chronic pancreatitis
Tropical pancreatitis: most common cause in the Tropics (esp. Southern India); young age at onset.

Pathophysiology:
Autodigestion and inflammation: damage to pancreatic acinar cells (e.g., alcohol), outflow obstruction of pancreatic enzymes or premature activation of trypsinogen to trypsin → intrapancreatic activation of digestive enzymes (e.g., amylase and lipase) → autodigestion of pancreatic tissue → inflammatory reaction
Fibrosis: exposure to toxins and/or inflammatory mediators (e.g., alcohol, cytokines) → activation of pancreatic stellate cells (which play a key role in the formation of pancreatic fibrosis)
Pancreatic insufficiency (due to tissue atrophy and fibrosis)
Exocrine insufficiency → deficiency of lipase, amylase, and protease → maldigestion, steatorrhea, malabsorption
Endocrine insufficiency → destruction of β-cells → pancreatic diabetes.

Clinical features:
Epigastric abdominal pain (main symptom)
Pain radiates to the back, is relieved on bending forward, and is exacerbated after eating.
Pain is initially episodic and becomes persistent as the disease progresses.
Often associated with nausea and vomiting
Features of pancreatic insufficiency
Late manifestation (after 90% of the pancreatic parenchyma is destroyed)
Steatorrhea (exocrine enzyme deficiency)
Cramping abdominal pain, bloating, diarrhea, weight loss
May cause fat-soluble vitamin deficiencies (A, D, E, and K)
Malabsorption and weight loss
Pancreatic diabetes (endocrine hormone deficiency)
In the later stages of chronic pancreatitis, patients may not experience any pain!

Diagnostics/investigations:
Imaging
-Abdominal CT (plain and contrast-enhanced): best initial imaging modality to screen for CP
Findings: pancreatic atrophy, pancreatic ductal dilations; pancreatic ductal calcifications on plain CT (more sensitive than x-ray); “chain of lakes” appearance
Can simultaneously rule out a pancreatic carcinoma or a gastrointestinal malignancy as a possible cause of epigastric pain and weight loss
-MRCP: indicated when CT findings are equivocal but clinical suspicion of CP is high
Findings: ductal strictures and dilations, pancreatic calcifications
-Abdominal x-ray: visible pancreatic calcifications (highly specific, but only seen in ∼ 30% of cases)
-Ultrasound
Abdominal ultrasound: indistinct margins and enlargement; pancreatic calcifications; ductal strictures, dilation or stones
Endoscopic ultrasound: parenchymal lobularity and hyperechoic foci; ductal dilation and calcification
-ERCP: detection of early pathologies and simultaneous treatment possible (e.g., duct dilation, stent insertion)
Ductal stones, seen as filling defects
“Chain of lakes” or “string of pearls” appearance (characteristic feature)
Irregularity, dilation of the main pancreatic duct.

Laboratory tests
-Serum pancreatic enzyme levels: lipase (specific) and amylase (non-specific)
-Pancreatic function tests
Indirect tests:
Fecal elastase-1 (FE-1) activity: confirms that steatorrhea is due to pancreatic lipase insufficiency.
Elastase-1 level < 200 μg/g → pancreatic exocrine insufficiency
Elastase-1 level < 100 μg/g → severe pancreatic exocrine insufficiency
72-hour quantitative fecal fat estimation: Fecal fat > 7 g per day is diagnostic of steatorrhea.
Direct tests (e.g., cholecystokinin test, secretin test, cholecystokinin-secretin pancreatic function test)

Genetic testing

  • Indication: family history of chronic pancreatitis; young patients with idiopathic pancreatitis
  • PRSS-1 mutations: diagnostic of hereditary pancreatitis
  • CFTR gene: 40% of patients with idiopathic chronic pancreatitis have a CFTR gene mutation.

Treatment
General measures
Abstinence from alcohol and nicotine
Small, regular meals (rich in carbohydrates, low in fat), supplementation with medium-chain triglycerides (MCT)
Pancreatic enzyme replacement (with meals)
Parenteral administration of fat-soluble vitamins (A, D, E, K) if necessary
Endocrine insufficiency: Insulin administration
For management of acute attacks see “Treatment” in acute pancreatitis.

Pain management
Analgesics: NSAIDs, opioids for severe pain (e.g., long-acting fentanyl/morphine), low dose tricyclic antidepressants (e.g., amitriptyline)
Intractable pain
Celiac ganglion block (offers temporary relief)
Endoscopic papillotomy + ductal dilation and stent placement + removal of stones, if present
Extracorporeal shock wave lithotripsy (ESWL): for intraductal stones

Surgery
Indication: if pancreatic cancer is suspected or in those with intractable pain
Procedures
Pancreaticojejunostomy: if the main pancreatic duct is dilated (> 5 mm)
Resection of the affected part of the pancreas (distal pancreatectomy, Whipple’s procedure)
Thoracoscopic bilateral splanchnicectomy.

Complications:
Pancreatic insufficiency
Tissue atrophy and fibrosis cause:
Exocrine insufficiency → deficiency of lipase, amylase, and protease → maldigestion, steatorrhea, malabsorption
Destruction of beta cells → endocrine insufficiency with pancreatic diabetes [14]
Pancreatic pseudocysts
Definition: encapsulated collection of pancreatic fluid that develops 4 weeks after an acute attack of pancreatitis; can occur in both acute and chronic pancreatitis
Pathophysiology: pancreatic secretions leak from damaged ducts → inflammatory reaction of surrounding tissue → encapsulation of secretions by granulation tissue
Clinical features
Often asymptomatic
Painless abdominal mass
Pressure effects
Gastric outlet obstruction (early satiety, nonbilious vomiting, abdominal pain)
Obstruction of the distal duodenum (bilious vomiting)
Results in steatorrhea
Bile duct obstruction with jaundice
Diagnostics: abdominal ultrasound/CT/MRI → extrapancreatic fluid collection within well-defined wall/capsule; no solid cyst components detectable
Treatment : surgical/endoscopic cystogastrostomy/cystoduodenostomy/cystojejunostomy ; ultrasound/CT-guided percutaneous drainage
Complications
Infection → fever, abdominal pain, sepsis
Rupture → pancreatic ascites/pancreaticopleural fistula
Erosion into an abdominal vessel with hemorrhage into the cyst → sudden abdominal pain, signs of hemorrhagic shock.

Splenic vein thrombosis

Pancreatic ascites
Pancreatic abscess
Portal vein thrombosis
Pancreatic diabetes 
Pancreatic cancer (especially in patients with hereditary pancreatitis)
39
Q

Acute appendicitis

A

D:
Appendicitis: acute inflammation of the vermiform appendix
Uncomplicated appendicitis: appendicitis with no evidence of an appendiceal fecalith, an appendiceal tumor, or complications, such as perforation, gangrene, abscess, or mass [1]
Complicated appendicitis: appendicitis associated with perforation, gangrene, abscess, an inflammatory mass, an appendiceal fecalith, or an appendiceal tumor [1]

Epidemiology:
Common cause of acute abdomen [2]
Lifetime risk: ∼ 8%
Peak incidence: 10–19 years of age [3]
Sex: ♂ > ♀

Aetiology:
Caused by obstruction of the appendiceal lumen due to: [4]
Lymphoid tissue hyperplasia (60% of cases): most common cause in children and young adults
Fecalith and fecal stasis (35% of cases): most common cause in adult
Neoplasm (uncommon): more likely in patients > 50 years of age [5]
Parasitic infestation (uncommon): e.g., Enterobius vermicularis, Ascaris lumbricoides, and species of the Taenia and Schistosoma genera [6]

Pathophysiology
Obstructed proximal appendiceal lumen (closed-loop obstruction), resulting in: [8]
Stasis of mucosal secretions → bacterial multiplication and local inflammation → transmural spread of infection → clinical features of appendicitis
Increased intraluminal pressure → obstruction of veins → edema of the appendiceal walls → obstruction of capillaries → ischemia → gangrenous appendicitis with/without perforation
Inflammation can spread to serosa, leading to peritonitis.

Clinical features:
Migrating abdominal pain: most common and specific symptom
Typically constant and rapidly worsens
Most patients present within 48 hours of symptom onset.
Initial diffuse periumbilical pain: caused by the irritation of the visceral peritoneum (pain is referred to T8–T10 dermatomes) [10]
Localizes to the RLQ within ∼ 12–24 hours: caused by the irritation of the parietal peritoneum
Associated nonspecific symptoms
Nausea
Anorexia: up to 80% of cases
Hamburger sign: If there is no loss of appetite, appendicitis is unlikely. [7]
Vomiting
Low-grade fever
Diarrhea
Constipation
Clinical signs of appendicitis [8][3]
McBurney point tenderness (RLQ tenderness)
Tenderness at the junction of the lateral third and medial two-thirds of a line drawn from the right anterior superior iliac spine to the umbilicus
This point corresponds to the location of the base of the appendix.
RLQ guarding and/or rigidity
Rebound tenderness (Blumberg sign), especially in the RLQ
Rovsing sign: RLQ pain elicited on deep palpation of the LLQ [8]
Psoas sign
Can be elicited on flexing the right hip against resistance
RLQ pain may be elicited on passive extension of the right hip
Obturator sign: RLQ pain on passive internal rotation of the right hip with the hip and knee flexed
Hyperesthesia within Sherren triangle: formed by the anterior superior iliac spine, umbilicus, and symphysis pubis
Lanz point tenderness: at the junction of the right third and left two-thirds of a line connecting both the anterior superior iliac spines
Pain in the Pouch of Douglas: pain elicited by palpating the rectouterine pouch on rectal examination
Baldwin sign: pain in the flank when flexing the right hip (suggests an inflamed retrocecal appendix)
The location of the pain may be variable as the appendix’s location varies, especially in pregnant women.

Management:
The initial approach to management should be guided by the likelihood of acute appendicitis based on clinical features and laboratory parameters (see Risk stratification tools for acute appendicitis).
Low likelihood of appendicitis: Consider alternative diagnoses (see differential diagnoses of acute abdomen).
Perform further diagnostic tests if needed (see diagnostic workup of acute abdomen).
Consider discharging the patient with advice to follow up after 12–24 hours (or earlier if symptoms worsen).
Consider hospitalization for observation if the index of suspicion is high or the underlying cause is unclear.
Reassess scores after 12–24 hours (every 6–8 hours in hospitalized patients).
Moderate likelihood of appendicitis: Confirm diagnosis with imaging.
Imaging confirms appendicitis: Initiate treatment.
Imaging rules out appendicitis: Follow steps for low likelihood of appendicitis.
Imaging is inconclusive [12]
Low index of suspicion: Follow steps for low likelihood of appendicitis.
High index of suspicion
Consider admission and serial abdominal examination.
Consider diagnostic laparoscopy. [12][13][14][15]
Consider empiric antibiotic therapy (for a minimum duration of 3 days) and supportive care with analgesics and antipyretics as needed. [12]
High likelihood of appendicitis: Initiate treatment.
Empiric antibiotic therapy is recommended in all patients.
Emergency appendectomy
Nonoperative management (NOM) with antibiotic therapy may be considered in the setting of a clinical trial in patients who present with early uncomplicated appendicitis.
NOM is recommended for the management of appendiceal mass or appendiceal abscess; recurrences may require interval appendectomy.

Diagnostics
Acute appendicitis is usually a clinical diagnosis based on history, physical examination, and laboratory studies. The appendicitis scoring systems should be used to guide management decisions. Imaging is recommended if the clinical diagnosis is uncertain.
Laboratory studies [11][16]
Routine studies
CBC: mild leukocytosis with left shift ; normal WBC count does not rule out acute appendicitis
CRP: elevated (> 10 mg/L) [16]
BMP: ↑ creatinine, electrolyte abnormalities may be present in patients with severe vomiting and diarrhea
Urinalysis: typically normal in appendicitis; possible findings of mild pyuria and/or hematuria
Tests to rule out differential diagnoses
Urine/serum β-hCG test: perform in all women of reproductive age to rule out pregnancy (including ectopic pregnancy) [12]
See “diagnostics” in acute abdomen for further information.
Risk stratification tools for acute appendicitis
There are several scoring systems to estimate the likelihood of acute appendicitis in a patient with RLQ pain. The following three scoring systems are most frequently used in clinical practice and are based on clinical features and laboratory studies. These scores guide management decisions, including determining when imaging may be unnecessary.
Alvarado score (MANTRELS) [11][13][17][18][19]
A 10-point scoring system that uses eight parameters to estimate the likelihood of appendicitis
Leukocytosis and RLQ tenderness carry the greatest weight.
Accuracy is higher in young to middle-aged adults than in children < 10 years of age and adults > 60 years of age. [16] [18]
The Alvarado score may not provide a reliable estimated likelihood of acute appendicitis in patients who are either very young or very old. [16]
Pediatric appendicitis score [21][22]
A scoring system to estimate the likelihood of appendicitis in patients 3–18 years of age
The parameters that carry the greatest scores are RLQ tenderness and RLQ pain elicited on coughing, jumping, and percussion.
Appendicitis inflammatory response score [11]
A relatively new scoring system that places emphasis on laboratory parameters and the gradation of clinical features to provide a more objective clinical evaluation
Provides a more accurate estimated likelihood of acute appendicitis than the Alvarado scoring system [24]
Imaging [11][16][12][26][27]
The risks and benefits of imaging in patients with suspected appendicitis must be carefully weighed. Appendicitis scoring systems can be used to guide imaging decisions. [17][16]
Recommended indications [16][13][17][23][27]
Moderate likelihood of appendicitis
Patients > 60 years of age, regardless of the likelihood scores [13][28]
To rule out suspected differential diagnoses
Relative indications [17]
A moderately low likelihood of appendicitis (Alvarado score 2–4)
A moderately high likelihood of appendicitis (Alvarado score 7–8)
Imaging likely unnecessary [17]
A very low likelihood of appendicitis (Alvarado score ≤ 2)
A very high likelihood of appendicitis (Alvarado score ≥ 9)
CT abdomen with IV contrast
Indications: preferred initial imaging modality in adults (except for pregnant women) [26]
Supportive findings [26]
Distended appendix (diameter > 6 mm)
Edematous appendix with periappendiceal fat stranding
Possible appendiceal fecalith: focal hyperdensity within the appendiceal lumen
Evidence of complications
Additional considerations
Consider low-dose CT scan (with IV contrast) to minimize radiation exposure. [29]
Consider CT without contrast in patients with contrast allergy.
Appendiceal abscessPerforated appendicitis due to fecalith
Abdominal ultrasound
Indications
Preferred initial imaging modality in children or pregnant patients [26]
An alternative to CT scan if ultrasound findings are inconclusive [27]
Supportive findings [16] [30]
Distended appendix (diameter > 6 mm)
Noncompressible, aperistaltic, distended appendix
Target sign: concentric rings of hypo- and hyperechogenicity in the axial/transverse section of the appendix
Possible appendiceal fecalith: focal hyperechogenicity with posterior acoustic shadowing
Abdominal ultrasound is more reliable for confirming acute appendicitis than ruling it out. [16]
Acute appendicitisAcute appendicitisAppendicitis with target signAppendicitis with target sign
MRI abdomen and pelvis [13][26][31]
Indications
MRI without IV contrast: pregnant patients with inconclusive ultrasound findings [26][31]
MRI with IV contrast: nonpregnant patients with inconclusive ultrasound findings and contraindications for CT scan
Findings: similar to CT scan findings
Diagnostic laparoscopy
Indications: Consider in the following groups of patients with inconclusive findings on imaging. [12][13][14]
Women of reproductive age
Patients with obesity
Patients > 65 years of age
Findings [32]
Acute uncomplicated appendicitis: inflamed, distended, erythematous appendix
Possible signs of complications: perforation, gangrene, pus
Additional steps based on findings [33][15]
Normal appendix on diagnostic laparoscopy: Leave in situ.
Appendicitis confirmed: Perform laparoscopic appendectomy.

Treatment:
Supportive care
Bowel rest (NPO)
Intravenous fluids (see IV fluid therapy)
Electrolyte repletion as needed
IV analgesics (see pain management) [11]
IV antiemetics as needed
Antipyretic therapy
Antibiotic therapy [12][27][34][35]
Indication: all patients with acute appendicitis
Required coverage: against gram-negative and anaerobic organisms.
Operative management with appendectomy [12][13][27][39][31]
Definition: surgical removal of the appendix
Timing of surgery [12][31][39]
Emergency appendectomy : performed as soon as the diagnosis is confirmed, without a trial of conservative management
Delayed appendectomy: performed within 24 hours of diagnosis and without a trial of conservative management [31][13][40]
Indications: Emergency appendectomy is the current standard of care for acute appendicitis (without periappendiceal mass or abscess). [31][41]
Contraindications [31][41]
Appendiceal mass
Appendicular abscess
Approach [31]
Laparoscopic appendectomy
Open appendectomy (via a transabdominal incision in the RLQ)
Appendectomy in patients with an appendiceal abscess or an inflammatory appendiceal mass is associated with an increased risk of intraoperative hemorrhage, postoperative wound infection, and fecal fistula formation.
Nonoperative management of acute appendicitis (NOM; conservative management) [13][36][31][43][44]
Indications [12][36][31][41][44]
Inflammatory appendiceal mass [31][41]
Appendiceal abscess [31][41]
Patient refusal of surgery
High surgical risk due to comorbidities
History of previous surgical/anesthesia complications
Contraindications [36][31]
Septic shock
Generalized peritonitis
Inability to percutaneously drain an appendiceal abscess
Appendiceal fecalith [45][46]
Early uncomplicated appendicitis
The use of NOM in early uncomplicated appendicitis is an area of ongoing research. [12][36]
Steps of nonoperative management [12][36]
Empiric parenteral antibiotic therapy for 2–3 days: See ‘‘Mild or moderate infection’’ under ‘‘Community-acquired infections’’ in empiric antibiotic therapy for intra-abdominal infections. [12][36][31]
Supportive care (See above)
Periappendiceal abscess > 4 cm: image-guided percutaneous drainage; send aspirate for cultures
Monitor vitals and serial abdominal examinations every 6–12 hours.
Insignificant improvement/worsening of symptoms : urgent surgical intervention [31]
Symptomatic improvement within 24–48 hours
Slow introduction of enteral nutrition
Switch to oral antibiotics for 7-day course. [36][31]
Schedule interval colonoscopy in patients > 40 years of age following NOM of acute appendicitis to rule out early colonic malignancy. [48][13][41]
Think of PAIN to remember the conservative management of appendicitis: Pain management, Antibiotics, Intravenous fluid therapy, NPO!
Interval appendectomy [13][31][44][49][50]
Definition: appendectomy performed 6–8 weeks following the resolution of an acute episode of appendiceal mass or appendiceal abscess to minimize surgical complications [50]
Indications
Currently not routinely recommended [13][44]
Consider in persistent or recurrent symptoms of appendicitis in a patient with an appendiceal mass or appendiceal abscess treated conservatively. [13][31][41]
Consider in patients > 40 years of age if there is concern for an underlying appendiceal tumor.

Pathophysiology
The appendix is composed of the same four histological layers of the alimentary canal.
See “Microscopic anatomy” in large intestine for the histological features of a healthy appendix.
Transmural neutrophilic infiltration is the characteristic histological feature of acute appendicitis.
Blood vessel thrombosis, mucosal ulceration, and/or gangrene of the appendiceal wall may also be present.

Ddx:
Ectopic pregnancy
Pseudoappendicitis [54]
Meckel diverticulum
Diverticulitis (especially in elderly patients)
Inflammatory bowel disease
Gastroenteritis
Colon cancer
Urolithiasis and renal colic
Urinary tract infections
Psoas abscess (in patients with a positive psoas sign)
Gynecological diseases (e.g., pelvic inflammatory disease, ovarian cyst)
See differential diagnoses of acute abdomen.
Right-sided carcinoma of the colon may manifest with clinical features similar to those of acute appendicitis!

Complications:

  • Inflammatory appendiceal mass (appendiceal phlegmon)
  • Appendiceal abscess
  • Gangrenous appendicitis
  • Perforated appendix
  • Pylephlebitis

Prognosis
Uncomplicated appendicitis with adequate management (surgical intervention) has an excellent prognosis. [9]
Perforation and peritonitis: ∼ 1% mortality rate [9]
Up to 20% of patients are found to have a normal appendix following surgery. [8]
The mortality rate is higher (∼ 5%) in elderly patients with complicated appendicitis. [9]

40
Q

Gallstones

A
Gallstones
There are different types of gallstones (for more information on gallstone pathologies, see cholelithiasis and cholecystitis):
Cholesterol stones (∼80% of cases)
Radiolucent with radiopaque areas due to calcifications (< 20%)
Composed of calcium carbonate
Pigment gallstones
Black pigment gallstones
Radiopaque
Associated with hemolytic diseases such as sickle cell anemia, hereditary spherocytosis
Composed of calcium bilirubinate
Brown pigment gallstones
Radiolucent
Associated with infection
41
Q

Acute cholecystitis summary

A

Acute cholecystitis refers to the acute inflammation of the gallbladder, which is typically due to cystic duct obstruction by a gallstone (acute calculous cholecystitis). Acalculous cholecystitis is less common and is seen predominantly in critically ill patients. RUQ pain, a positive Murphy sign, and fever are the characteristic clinical features of acute cholecystitis. RUQ ultrasound is the preferred initial imaging modality, which would show gallbladder distension, edema, and pericholecystic fluid. Empiric antibiotic therapy and laparoscopic cholecystectomy are the mainstays of treatment. Laparoscopic cholecystectomy should be performed as soon as possible, preferably within 72 hours of admission, unless operative and anesthesia risks outweigh the benefits of urgent surgery. In high-risk patients with severe cholecystitis, a temporizing gallbladder drainage procedure (e.g., percutaneous cholecystostomy, endoscopic gallbladder stenting) should be performed and elective interval cholecystectomy scheduled after the resolution of acute symptoms. Complications of acute cholecystitis include gangrenous cholecystitis, emphysematous cholecystitis, gallbladder perforation, biliary-enteric fistula, gallstone ileus, and pyogenic liver abscess. Chronic cholecystitis may result from recurrent attacks of acute cholecystitis or due to chronic cholelithiasis. Chronic gallbladder inflammation increases the risk of gallbladder carcinoma.

42
Q

Acute cholecystisis

A

Epidemiology
Sex: ♀ > ♂
Prevalence: most common complication of cholelithiasis
Peak incidence: > 50 years.

Aetiology
Acute calculous cholecystitis: most common form [1]
Cause: obstructing cholelithiasis
Pathophysiology
Cholelithiasis → passage of gallstones into the cystic duct → cystic duct obstruction → distention and inflammation of the gallbladder
Secondary bacterial infection may also be present (E. coli, Klebsiella, Enterobacter, Enterococcus spp. most common) but is not necessary for the development of cholecystitis.
Acalculous cholecystitis: 5–10% of acute cholecystitis [2]
See ”Acalculous cholecystitis” in “Subtypes and variants” section
Approximately 90% of acute cholecystitis is caused by cholelithiasis. Acalculous cholecystitis accounts for the remaining 10%.

Clinical features:
RUQ pain
Typically more severe and prolonged (> 6 hours) than in biliary colic
Postprandial
Radiation to the right scapula (due to referred pain from phrenic nerve irritation)
Positive Murphy sign: sudden pausing during inspiration upon deep palpation of the RUQ due to pain
Murphy sign may be falsely negative in patients > 60 years. [4][5]
Guarding
Fever, malaise, anorexia
Nausea and vomiting

Acute cholecystitis should always be suspected in a patient with a history of gallstones who presents with RUQ pain, fever, and leukocytosis.

Diagnostics:
The diagnosis of acute cholecystitis is based on characteristic clinical features, systemic signs of inflammation (leukocytosis, ↑ CRP), and evidence of gallbladder inflammation on imaging.
Approach
Initial evaluation: laboratory studies and RUQ ultrasound
If ultrasound findings are inconclusive, consider HIDA scan, abdominal MRI, or abdominal CT to confirm the diagnosis.
Assess for choledocholithiasis (see “Diagnosis of choledocholithiasis”).
Once the diagnosis is confirmed, determine the severity (see “Severity grading of acute cholecystitis”).

Laboratory studies

Tests to support the clinical diagnosis
CBC: leukocytosis
CRP: elevated
Blood cultures: should be obtained, especially in patients with grade III acute cholecystitis (see “Severity grading of acute cholecystitis”) [14]
Bile cultures: should be obtained in patients undergoing laparoscopic cholecystectomy or gallbladder drainage [14]
Tests to assess the severity of disease (see “Severity grading of acute cholecystitis”)
Blood gas analysis: PaO2/FiO2 ratio < 300 in severely ill patients
BMP: AKI, electrolyte derangements may be present in patients with severe disease
PT/INR: coagulopathy in patients with severe disease
Tests to rule out related biliary comorbidities: should be obtained in all patients with suspected cholecystitis
LFTs [13][15]
Mild elevations in AST and ALT are possible in acute cholecystitis.
Signs of cholestasis (↑ bilirubin, ↑ ALP), ↑ GGT) are uncommon in cholecystitis; if present consider biliary obstruction (see ‘‘Diagnosis of choledocholithiasis’’ and ‘‘Cholangitis’’)
Lipase, amylase
Mild elevation of amylase may be seen in acute cholecystitis [16]
Elevation of lipase or amylase ≥ 3 times the normal is suggestive of acute biliary pancreatitis.
Tests to rule out differential diagnoses: See “Diagnostic workup of acute abdominal pain.”
Imaging [10][9]
RUQ transabdominal ultrasound

Hepatoiminodiacetic acid scintigraphy (HIDA scan) [9][18][2]

MRI abdomen without and with IV contrast

CT abdomen with IV contrast.

DDx:
RUQ pain with fever
Acute cholangitis
Liver abscess
Acute hepatitis
Acute necrotizing pancreatitis
Acalculous cholecystitis
Chronic cholecystitis
Bile leak (Iatrogenic) 
RUQ pain without fever
Biliary colic
PUD
Acute gastritis
Acute pancreatitis
Chronic pancreatitis
Acute GERD
Sphincter of Oddi dysfunction
Congestive hepatopathy
Nephrolithiasis
Malignancy
HCC
Cholangiocarcinoma
Gallbladder cancer
Pancreatic cancer

Treatment
Empiric antibiotic therapy and cholecystectomy are the mainstays of treatment for acute cholecystitis after initial supportive therapy. Laparoscopic cholecystectomy should be performed as soon as possible unless operative and anesthesia risks outweigh the benefits of urgent surgery. In grade II–III acute cholecystitis, or patients at high risk of surgical complications, a temporizing gallbladder drainage procedure can be performed and elective interval cholecystectomy scheduled for when operative and anesthesia risks are minimized. Different treatments for diff stages on word doc.

Complications
Gangrenous cholecystitis [34][7][35]
Definition: ischemic necrosis of the gallbladder
Etiology: most common complication of acute cholecystitis [7]
Clinical features: difficult to distinguish from uncomplicated acute cholecystitis
Imaging
Ultrasound: features of acute cholecystitis plus echogenic membranes floating within the gallbladder lumen
CT with IV contrast: nonenhancement of the gallbladder wall
Treatment: emergency laparoscopic cholecystectomy and empiric antibiotic therapy for biliary infection
Emphsematous cholecystitis
Gallbladder perforation [7][36][35]
Definition: break in the continuity of the gallbladder wall, typically as a consequence of ischemic necrosis
Clinical features: variable; symptoms typically progress rapidly
May be indistinguishable from uncomplicated acute cholecystitis
Potentially accompanied by a palpable RUQ mass and/or signs of generalized peritonitis
Imaging : focal defect in the gallbladder wall; extraluminal stone may be visualized
Treatment: emergency laparoscopic cholecystectomy and empiric antibiotic therapy for biliary infection
Cholecystoenteric fistula [7][37][35]
Definition: an abnormal communication between the gallbladder lumen and the lumen of the adjacent bowel (usually stomach or duodenum)
Etiology: gallbladder perforation or pressure necrosis from a large gallstone
Clinical features [38]
Potentially asymptomatic
Bile acid diarrhea
Can manifest as cholangitis (see ‘‘Clinical features’’ in “Cholangitis” for details)
Gallstone ileus, which may manifest as:
Mechanical bowel obstruction
Gastric outlet obstruction (Bouvouret syndrome) [39]
Imaging [38]
Air within the biliary tree (pneumobilia)
Fistulous tract between the gallbladder (typically the fundus) and the adjacent bowel
Treatment
Patients with cholangitis: empiric antibiotic therapy for biliary infection (combination regimen) and laparoscopic cholecystectomy with closure of the fistula. [10]
Patients with gallstone ileus: enterolithotomy with or without cholecystectomy and closure of the fistula (see ‘‘Treatment’’ in “Gallstone ileus” for details) [40][41]
Gallbladder empyema (suppurative cholecystitis) [7]
Definition: distended pus-filled gallbladder
Clinical features: similar to uncomplicated acute cholecystitis
Imaging: gallbladder distention with hyperechoic (on ultrasound) or hyperintense (on CT abdomen with IV contrast) material within its lumen
Treatment [42]
Empiric antibiotic therapy for biliary infection
Emergency source control procedure
Low surgical risk: laparoscopic cholecystectomy
High surgical risk: image-guided percutaneous drainage of empyema followed by interval laparoscopic cholecystectomy
Chronic cholecystitis [3][7][43]
Definition: chronic inflammation of the gallbladder
Etiology
Chronic irritation of gallbladder mucosa by cholelithiasis
Recurrent attacks of acute cholecystitis
Clinical features: recurrent symptoms similar to acute cholecystitis but typically less severe and often self-limiting
Diagnostics
Laboratory studies: may be normal [43]
Ultrasound abdomen or CT abdomen with IV contrast:
Thickened gallbladder wall
No evidence acute inflammatory changes (e.g., pericholecystic fluid);
Cholelithiasis commonly present
HIDA scan: delayed visualization of the gallbladder [44]
All patients should also be evaluated for choledocholithiasis before treatment (see ‘‘Imaging’’ in “Choledocholithiasis”).
Treatment: elective laparoscopic cholecystectomy [7][45]
Complications [7]
Porcelain gallbladder [7][46][44]
Definition: calcification of the gallbladder wall due to chronic inflammation
Imaging (x-ray or noncontrast CT abdomen): focal or diffuse hyperdensity (radiopaque appearance) of the gallbladder wall
Clinical significance: a risk factor for gallbladder cancer [44]
Treatment: laparoscopic cholecystectomy even if asymptomatic
Gallbladder cancer [47]
Cholecystoenetric fistula and gallstone ileus.
Chronic gallbladder inflammation increases the risk of gallbladder carcinoma, especially when porcelain gallbladder is present.
Other [7]
Pericholecystic abscess
Pyogenic liver abscess
Hemorrhagic cholecystitis [7]
Gallbladder cancer

43
Q

Cholangiocarcinoma summary

A

Biliary tract cancers include cholangiocarcinoma and gallbladder carcinoma, which are both rare diseases with very poor prognoses. Cholangiocarcinoma is classified as either intrahepatic or extrahepatic according to the anatomical site of the tumor. Extrahepatic carcinoma, which is the more common form, can be further classified as perihilar (Klatskin tumor; occurs at the bifurcation of the hepatic duct) and distal extrahepatic carcinoma. Risk factors for cholangiocarcinoma include primary sclerosing cholangitis and chronic biliary tract inflammation. The greatest risk factor for gallbladder carcinoma is cholelithiasis. Patients are often initially asymptomatic or only present with nonspecific symptoms (e.g., abdominal pain, fever, weight loss) until late stages of disease, meaning that most tumors are already advanced at the time of diagnosis. Extrahepatic cholangiocarcinoma may manifest with signs of cholestasis (e.g., jaundice, dark urine, pruritus) and a painless, enlarged gallbladder (Courvoisier sign). If liver transaminases and tumor markers are raised and/or ultrasound imaging suggests bile duct or gallbladder carcinoma, MRCP or MDCT are recommended for diagnosis. Although surgical resection of early-stage tumors is curative, approximately 90% of patients have more advanced, unresectable tumors at the time of diagnosis.

44
Q

cholangiocarcinoma dr deac pimp

A

Cholangiocarcinomas: originate in the bile ducts and are classified based on their anatomical site of origin [1][2]
Extrahepatic cholangiocarcinoma is the most common form (90% of cases).
Perihilar (Klatskin tumor): junction of the right and left hepatic ducts (50% of cases)
Distal extrahepatic: common bile duct (40% of cases)
Intrahepatic cholangiocarcinoma: intrahepatic bile ducts (10% of cases).

Epidemiology 
Cholangiocarcinoma [2][4]
Incidence: ∼ 1/100,000 per year in the US
Prevalence: < 1% of all cancers
Peak incidence: 60–70 years of age
Sex: ♂ > ♀

Aetiology
Risk factors for cholangiocarcinoma [2][3][5]
Primary sclerosing cholangitis
Liver fluke infection (e.g., Clonorchis sinensis, Opisthorchis viverrini)
Choledocholithiasis (both with and without hepatic duct involvement)
Chronic viral hepatitis (e.g., hepatitis B, hepatitis C)
Liver cirrhosis
Environmental toxin exposure (e.g., asbestos, Thorotrast contrast agent)
Congenital biliary tract abnormalities (e.g., choledochal cysts, congenital hepatic fibrosis).

Clinical features
Cholangiocarcinoma [2]
Extrahepatic cholangiocarcinoma
Signs of cholestasis: jaundice, pale stools, dark urine, pruritus
Abdominal pain and weight loss is usually a sign of late (unresectable) disease.
Courvoisier sign
Intrahepatic cholangiocarcinoma
Usually asymptomatic in early stages
Nonspecific symptoms (e.g., weight loss, nausea, fever, weakness, fatigue)
Dull abdominal pain (RUQ or epigastric)
Signs of cholestasis are rare.

Investigations
The diagnostic approach varies depending on the location of the tumor and whether PSC is present. If the patient’s presentation, laboratory findings (especially tumor markers), and/or transabdominal ultrasound suggest biliary cancer, then MRCP or MDCT is generally recommended for diagnosis. Biopsy is generally unnecessary.
Laboratory tests [2][4]
Liver function tests: possible ↑ INR, ↑ ALT, and ↑ AST (with chronic biliary obstruction and eventual hepatic dysfunction)
Parameters of cholestasis (e.g., ALP, GGT, total bilirubin) may initially be normal.
Tumor markers (to determine the baseline; should not be used to confirm the diagnosis)
↑ AFP
↑ CA 19-9 and ↑ CEA
Especially helpful for diagnosing cholangiocarcinoma with primary sclerosing cholangitis
Useful for assessing therapeutic responsiveness and detecting relapse
Imaging [2][4]
Transabdominal ultrasound should be the initial imaging modality for suspected biliary tract carcinoma
Localization of obstruction and evaluation for gallstones
Findings: gallbladder and/or bile duct dilatation
Useful for identifying local metastases and performing ultrasound-guided biopsy
MRCP
Recommended for definitive diagnosis
Findings: bile duct dilatation and/or mass lesion
Abdominal MDCT is commonly used as an alternative to MRCP
Endoscopic ultrasound (especially for distal extrahepatic lesions and staging)
Chest CT for staging
PET scan: indicated if other diagnostic procedures are inconclusive and for staging
Biopsy [2][5]
Cholangiocarcinoma: ERCP with ductal brushings or biopsy is recommended but not always necessary for diagnosis.
May be indicated if MRCP or MDCT are inconclusive
Allows for confirmation of tumor type via tissue sampling
Sometimes preferred by surgeons to assess tumor resectability; allows for immediate placement of a stent if the tumor is inoperable
Rarely, percutaneous transhepatic cholangiography (PTC) is used for proximal lesions.
Gallbladder cancer: biopsy is usually unnecessary; typically proceeds directly to surgical exploration/resection if clinical suspicion is high
Surgical exploration: Exploratory laparoscopy is often performed for definitive diagnosis and/or staging prior to resection.

Pathology 
Cholangiocarcinoma: usually a well-differentiated adenocarcinoma
Gallbladder cancer [3]
Adenocarcinoma is the most common form.
< 10% are squamous cell tumors.
Ddx:
Cholangiocarcinoma
Hepatocellular carcinoma
Pancreatic cancer
Choledocholithiasis
Chronic hepatitis
Chronic pancreatitis
Acalculous cholecystitis
Calculous cholecystitis
Acute cholangitis
Autoimmune cholangitis. 

Treatment
The prognosis for cholangiocarcinoma and gallbladder cancer is generally poor, especially for gallbladder cancer and intrahepatic cholangiocarcinoma. Treatment is determined by tumor resectability.
Resectable disease [4]
< 10% of cases are resectable.
Resection is the only curative therapy.
Procedure: radical resection plus lymphadenectomy
Preoperative biliary drainage is performed in some cases.
Adjuvant chemotherapy: usually a fluoropyrimidine-based regimen.

Unresectable disease [4]
Unresectable carcinoma or metastatic disease accounts for > 90% of cases.
Treatment is palliative
Chemotherapy: fluoropyrimidine-based or gemcitabine-based regimen
Biliary stent placement: in patients with jaundice and extrahepatic cholangiocarcinoma or gallbladder carcinoma
Transarterial chemoembolization (TACE): local application of chemotherapy and occlusive substance → induces fibrosis and shrinkage of intrahepatic cholangiocarcinoma.
Table on word document.

45
Q

Abdominal hernias Summary

A

Abdominal hernias are defined as the abnormal protrusion of intra-abdominal contents through congenital/acquired areas of weakness in the abdominal wall. The four categories of anatomically-classified abdominal hernias include the following: ventral hernias (e.g., epigastric, umbilical, incisional hernias), groin hernias (inguinal and femoral hernias), pelvic hernias (obturator, sciatic, and perineal hernias), and flank/lumbar hernias. Groin hernias are discussed in more detail in their respective articles. Inguinal, incisional, and umbilical hernias are the most common types of hernias. Persistently raised intra-abdominal pressure (e.g., due to ascites, pregnancy, intra-abdominal tumors, chronic cough, etc.) increases the risk of developing an abdominal hernia. Uncomplicated hernias are asymptomatic, nontender, and completely reducible with an expansile cough impulse. Complicated hernias include incarcerated, obstructed, and strangulated hernias and are characterized by tenderness, irreducibility, features of bowel obstruction, and an absent cough impulse. Abdominal hernias are often diagnosed on clinical examination. Imaging (e.g., ultrasound, CT scan) is used to confirm the diagnosis and evaluate the contents of the hernia. Complicated hernias and those with a narrow neck (e.g., femoral hernia, obturator hernia, paraumbilical hernia) should be surgically repaired (primary repair/mesh repair). Congenital umbilical hernias typically close spontaneously by 5 years of age, have a wide neck, and a low risk of complications; surgical intervention is rarely necessary.

46
Q

Abdominal hernias classification

A

Ventral hernias:
Epigastric hernia: herniation through the linea alba, between the xiphoid process and the umbilicus
Umbilical hernia: see below
Incisional hernia: see below
Spigelian hernia: a rare type of hernia that can occur anywhere along the semilunar line; most commonly below the arcuate line (i.e., below the umbilicus)
Parastomal hernia: herniation through a surgically created abdominal wall defect (i.e., a stoma).

Groin hernias
Inguinal hernia (direct/indirect)
Femoral hernia.

Pelvic hernia (rare)
Obturator hernia: herniation through the obturator foramen, especially the right side (since the sigmoid colon blocks the obturator canal on the left).
Sciatic hernia: herniation through greater or lesser sciatic foramen
Perineal hernia herniation through the pelvic floor
Obturator herniaSciatic herniasPerineal hernias.

Flank hernias
Incisional hernias (see below)
Lumbar hernias
Superior lumbar hernia: herniation through the superior lumbar triangle
Inferior lumbar hernia: herniation through the inferior lumbar triangle

Clinical features:
-Reducible hernia
Hernial contents completely return to the abdominal cavity through the abdominal wall defect on lying down or upon application of mild external pressure.
Most reducible hernias manifest as an asymptomatic nontender mass.
Increases on straining (e.g., sitting up from a recumbent position)
Decreases completely on lying down
Visible cough impulse present: expansion of the hernia when the patient is asked to cough
Edges of the fascial defect are palpable
Bowel sounds may be heard over the mass (if the hernial content is bowel)
-Irreducible/incarcerated hernia
Hernial contents become adhered to the hernial sac and cannot be reduced into the abdominal cavity.
Irreducible nontender mass
Visible cough impulse present
May decrease partially on lying down
Increased risk of obstruction and strangulation
-Obstructed hernia
The abdominal wall defect acts as a tourniquet around the hernial contents, causing edema and distension of the hernial contents.
Acute pain at the site of the hernia
Features of closed-loop bowel obstruction (if the hernial content is bowel)
Absent cough impulse
-Strangulated hernia
Ischemia and necrosis of the hernial contents due to compromised vascular supply
Acute pain at the site of the hernia
Features of bowel obstruction (if the hernial content is bowel)
Signs of strangulation
A tender, irreducible hernia
Absent cough impulse
Edematous, erythematous, warm overlying skin
Toxic appearance, fever, signs of sepsis
May lead to intestinal gangrene
Fatal if left untreated

The smaller the hernial orifice, the higher the risk of incarceration!

Investigations/diagnostics
Usually a clinical diagnosis
Imaging: indicated if the diagnosis is unclear and/or to identify contents of the hernial sac (e.g., loops of bowel)
Ultrasound: especially useful to identify ventral hernias (e.g., epigastric, Spigelian, incisional, or umbilical hernia)
IV and oral contrast enhanced CT scan: useful for suspected hernias that may be difficult to identify on physical examination (e.g., lumbar, obturator, perineal or sciatic hernia)
Findings: abdominal wall defect with/without protrusion of intra-abdominal contents through it
Abdominal x-ray
Indicated if an obstructed and/or strangulated hernia is suspected
Findings: features of bowel obstruction
Dilated bowel loops proximal to obstruction
Collapsed bowel loops distal to obstruction
Multiple air-fluid levels within dilated bowel loops.

Ddx:
-Differential diagnosis of a ventral hernia
Diastasis recti: a > 2 cm separation of the right and left rectus abdominis muscle with resultant protrusion of abdominal organs on straining
In newborns: omphalocele, gastroschisis
Abdominal wall tumor (e.g., desmoid tumor)
Lipoma
Rectus sheath hematoma
-Differential diagnosis of pelvic and groin hernias
Inguinal lymphadenopathy
Cryptorchid testes
Lipoma
Femoral artery aneurysm
-Differential diagnosis of a strangulated hernia
Abscess
Hematoma
Lymphadenitis (strangulated groin hernias).

Treatment
Surgical hernia repair is recommended for the management of most abdominal hernias.
Surgery: open or laparoscopic tension-free closure of the abdominal wall defect with/without a mesh
Elective surgery is indicated in reducible and incarcerated hernias.
Emergency surgery is indicated in obstructed or strangulated hernias.
Conservative management (observation) is indicated in:
Congenital umbilical hernia in children < 5 years of age
Asymptomatic wide-necked hernias in patients with high operative risk: A truss or corset may be considered in these patients to decrease the risk of obstruction and strangulation.

47
Q

Diverticular disease summary

A

Diverticular disease encompasses a set of colonic pathologies that result from abnormal outpouchings of the colonic mucosa (diverticula). It includes diverticulosis (noninflamed diverticula) and diverticulitis (inflamed diverticula). Colonic diverticula develop due to a combination of chronically elevated intraluminal pressures due to chronic constipation (e.g., due to low-fiber diets, lack of physical exercise) and age-related weakening of connective tissue. This causes the colonic mucosa to herniate through areas of weakness in the muscular layer. The sigmoid colon is most commonly involved. Incidence increases with age, and approx. 50% of individuals are affected by the 7th decade of life. Diverticulosis is typically asymptomatic but may manifest with lower gastrointestinal bleeding, altered bowel habits, and/or abdominal pain. Diverticulitis may remain localized (mild uncomplicated diverticulitis) or progress, resulting in complications such as abscess or perforation (complicated diverticulitis). Diverticulitis typically manifests with fever and left lower quadrant abdominal pain (as the sigmoid colon is most commonly involved). Colonoscopy is the diagnostic modality of choice for symptomatic diverticulosis but is contraindicated if acute diverticulitis is suspected. Abdominal CT scan with IV contrast is preferred in suspected acute diverticulitis. Uncomplicated diverticulitis usually responds to conservative management with bowel rest and analgesics; oral broad-spectrum antibiotics are reserved for patients at high risk of complications. In complicated diverticulitis, management consists of parenteral antibiotics, treatment of any complications, and, in some cases, emergency colonic resection. Once the acute phase has resolved, a colonoscopy is indicated to rule out malignancy. Elective colectomy is recommended for all patients with complicated diverticulitis that is managed conservatively. The procedure is not routinely indicated for uncomplicated diverticulitis.

48
Q

Diverticular disease Dr Deac Pimp

A

Diverticula: blind pouches that protrude from the gastrointestinal wall and communicate with the lumen
True diverticulum: a type of diverticulum that affects all layers of the intestinal wall.
Rare (except Meckel diverticulum )
Typically congenital
Most commonly occur in the cecum
Occur less commonly in the colon
False diverticulum or pseudodiverticulum: type of diverticulum that involves only the mucosa and submucosa and does not contain muscular layer or adventitia.
Most common type of gastrointestinal diverticula
Typically acquired
Diverticulosis: the presence of multiple colonic diverticula without evidence of infection [1]
Diverticulitis: inflammation or infection of colonic diverticula.

Epidemiology
In the US, ∼ 50% of individuals > 60 years have diverticulosis [2]
More common in high-income countries due to the higher prevalence of a high-fat, low-fiber diet.

Aetiology
Caused mainly by lifestyle and environmental factors
Diet (low-fiber, rich in fat and red meat)
Obesity
Low physical activity
Increasing age
Smoking
Other causes: genetic factors
Connective tissue disorders (e.g., Marfan syndrome, Ehler-Danlos syndrome) [1][3]
Autosomal dominant polycystic kidney disease.

Pathophysiology
Diverticulosis: The formation of diverticula is considered multifactorial.
Increased intraluminal pressure, e.g., due to chronic constipation
Weakness of the intestinal wall
Age-related loss of elasticity of the connective tissue
Physiological gaps in the intestinal wall, which occur where blood vessels penetrate, predispose to protrusion and herniation of intestinal mucosa and submucosa.
Localized particularly in the sigmoid colon
Diverticulitis
Most commonly: chronic inflammation and increased intraluminal pressure → erosion of diverticula wall → inflammation and bacterial translocation
Rarely: stool becomes lodged in diverticula → obstruction of intestinal lumen → inflammation

Clinical features
Diverticulosis
Usually asymptomatic
May manifest with abdominal discomfort or pain, especially if associated with chronic constipation
Diverticulitis
Low-grade fever
Sigmoid colon most commonly affected → left lower quadrant pain
Possibly tender, palpable mass (pericolonic inflammation)
Change in bowel habits (constipation in ∼ 50% of cases and diarrhea in 25–35% of cases)
Nausea and vomiting (caused by bowel obstruction or ileus)
Acute abdomen: indicates possible perforation and peritonitis
↑ Urinary urgency and frequency (in ∼ 15% of cases)
In elderly or immunocompromised patients, clinical symptoms may only be mild.

Diagnostics
Diverticulosis
Asymptomatic diverticulosis [8]
Typically an incidental diagnosis
No workup required
Symptomatic diverticulosis [9][10][11][12][13]
Colonoscopy: diagnostic modality of choice for suspected symptomatic diverticulosis [12]
Indications
Lower GI bleed
Recurrent abdominal pain and/or diarrhea
Concern for underlying malignancy [13]
Findings: well-defined outpouching from the colonic wall
Avoid if acute diverticulitis is suspected.
Biopsy and histological analysis can be performed, if necessary
DiverticulosisDiverticulosis
Double-contrast barium enema: highly sensitive test to detect diverticulosis but not commonly performed [9]
Consider in the workup of the following: [10]
Recurrent LLQ pain without signs of acute inflammation
Altered bowel habits
Lower GI bleed in a hemodynamically stable patient if colonoscopy cannot be performed
Contraindications: suspected diverticulitis or perforated diverticulum [13][11]
Findings: outpouching of the colonic wall of variable size
Diverticulosis
Abdominal ultrasound
Indications: may be performed as part of the workup for nonspecific LLQ pain [11]
Findings: outpouching from the colonic wall [13]
Colonoscopy is the diagnostic modality of choice for symptomatic diverticulosis.
Diverticulitis [14]
Suspect acute diverticulitis in adult patients presenting with LLQ pain, fever, and leukocytosis. The diagnosis is confirmed with imaging, preferably with IV contrast-enhanced CT scan. Once the acute phase has resolved, a colonoscopy should be performed to rule out malignancy. Colonoscopy is contraindicated in the acute phase because of the risk of perforation.
Laboratory studies [14][1][15]
Routine tests
CBC: leukocytosis, possible anemia
BMP: electrolyte abnormalities, ↑ BUN, ↑ creatinine
CRP: ↑ CRP
FOBT: positive in patients with diverticular bleeding
Imaging [11][14][15][16][17][18]
CT abdomen and pelvis with IV contrast [11]
Indications
Preferred initial imaging modality for suspected diverticulitis [11]
Staging the severity of diverticulitis
Supportive findings
Colonic outpouching
Signs of inflammation
Bowel wall thickening > 3 mm
Peridiverticular mesenteric fat stranding
Complications may also be identified
Peridiverticular abscess: hypodense collections with peripheral contrast enhancement
Diverticular perforation: pneumoperitoneum
Intestinal obstruction: dilated intestinal loops with multiple air-fluid levels
Diverticulitis
MRI abdomen and pelvis (without and with IV contrast)
Indications: suspected diverticulitis in patients with contraindications to CT [11]
Findings: similar to those on CT scan
Ultrasound abdomen
Indications: an alternative to MRI in patients with contraindications to CT
Supportive findings: diverticula with surrounding inflammation, abscess formation (detectable fluid), bowel wall thickening
DiverticulitisDiverticulitis
Abdominal x-ray [15]
Not useful in diagnosing uncomplicated diverticulitis
Indications
Suspected perforation or bowel obstruction
May be performed as part of the routine workup for acute abdominal pain
Findings that may be seen in complicated diverticulitis include
Bowel perforation: pneumoperitoneum
Bowel obstruction: dilated bowel loops and multiple air-fluid levels
Screening colonoscopy [19][17][14][15]
Recommended 6–8 weeks after the resolution of the acute episode to assess the extent of diverticulitis and rule out malignancy [14]
Colonoscopy is contraindicated during an acute episode because of the increased risk of perforation.
Not required if a recent evaluation of the colon has been performed [19]
Avoid colonoscopy during the acute phase of diverticulitis because of the risk of perforation!
Classification of diverticulitis
To choose the best treatment approach and determine the prognosis, identifying the stage of acute diverticulitis is recommended. [15]
Uncomplicated diverticulitis: localized inflammation of a colonic diverticulum with no evidence of complications
Complicated diverticulitis: inflammation of a colonic diverticulum associated with complications such as perforation, abscess, fecal peritonitis, bowel obstruction, or fistula formation [14]
The modified Hinchey classification is based on CT findings and is the most commonly used classification.

Differential diagnosis 
Differential diagnoses of uncomplicated acute diverticulitis
Crohn disease, ulcerative colitis
Colorectal cancer
Intestinal ischemia (ischemic colitis)
Acute appendicitis
Ileus , colonic obstruction
Ectopic pregnancy
Ovarian torsion
Ovarian cancer
Inguinal hernia
Renal colic
Urinary tract infection
Differential diagnoses of perforated diverticulitis: See gastrointestinal perforation.

Treatment
Diverticulosis
Asymptomatic diverticulosis
No treatment can reverse the growth of existing diverticula.
The goal is the prevention of progression (see “Prevention of recurrence and disease progression” below).
Symptomatic diverticulosis: see “Treatment” in GI bleeding
Diverticulitis [18][13][21]
Approach
Uncomplicated diverticulitis
Conservative management [15][18]
Consider broad-spectrum oral antibiotics (e.g., ciprofloxacin PLUS metronidazole) in select patient groups.
Complicated diverticulitis
Inpatient management with broad-spectrum IV antibiotics is recommended.
CT-guided percutaneous drainage for abscesses > 4 cm
Emergency colectomy in patients with generalized peritonitis
Acute management
Uncomplicated diverticulitis [18][21][15]
Antibiotic therapy: Not routinely recommended [17][19][22]
Indications [17][22][23]
Signs of generalized infection (e.g., fever)
Pregnancy
Immunosuppression
Comorbidities
Consider one of the following commonly used broad-spectrum oral antibiotics [22][23]
Metronidazole
PLUS one of the following
Ciprofloxacin
Levofloxacin
Trimethoprim-sulfamethoxazole
Amoxicillin-clavulanate
Moxifloxacin
See also empiric antibiotic therapy for intra-abdominal infections
Duration of antibiotic therapy: 4–7 days [17]
Supportive care
Relative bowel rest: clear liquid diet until improvement of symptoms
Analgesics as needed (see acute pain management)
Antiemetics as needed
Additional considerations
Screening colonoscopy once symptoms have resolved.
Consider elective colectomy for recurrent uncomplicated diverticulitis.
GI consultation for patients with frequent, recurrent episodes or chronic symptoms [22]
Complicated diverticulitis [18][17][19][21] [22]
Antibiotic therapy: broad-spectrum IV antibiotics are routinely recommended (see empiric antibiotic therapy for intra-abdominal infections)
Supportive care
Complete bowel rest (NPO)
IV fluids (see IV fluid therapy)
Analgesics as needed (see acute pain management)
Antiemetics as needed (see antiemetics)
Management of complications
Abscess [18][24][15]
Size ≥ 4 cm
Ultrasound- or CT-guided percutaneous drainage
Consider laparoscopic or open surgical drainage if percutaneous drainage is not feasible.
Continue IV antibiotic therapy.
Send aspirate or pus for cultures and tailor antibiotic treatment accordingly.
Perforation with generalized peritonitis: emergency surgery [18][17]
Hemodynamically stable patients: laparoscopic or open colectomy and primary anastomosis with/without a temporary diverting stoma
Critically ill patients: Hartmann procedure
Bowel obstruction
Additional considerations
Inpatient treatment with surgery and/or IR consultation
Screening colonoscopy once symptoms have resolved
Consider elective colectomy if not performed during admission.
Subsequent management
Elective colectomy [14][19][15][18]
Indications
Routinely recommended 6–8 weeks after resolution of complicated diverticulitis
Select groups of patients after resolution of uncomplicated diverticulitis, including: [15][17][18]
Patients at high risk of recurrence with complications [15]
Patients with persistent abdominal symptoms after resolution of an acute episode
Chronic complications of diverticulitis (e.g., fistula, colonic strictures)
Procedure: laparoscopic or open colectomy [17][15]

Prevention of recurrence and disease progression [19][17]
High-fiber diet [19][25]
Fluid hydration
Weight reduction
Vigorous physical activity
Cessation of smoking
Avoid nonaspirin NSAID use, if possible.

Complications:
Diverticular bleeding [20][26]
Epidemiology
Diverticulosis is the most common cause of lower GI bleeding in adults.
Occurs in ∼ 5% of individuals with diverticulosis
Etiology: erosions around the edge of diverticula
Clinical findings
Painless hematochezia
Severe or ongoing bleeding: significant drop in hemoglobin → hemodynamic instability (hypotension, tachycardia, dizziness, reduced level of consciousness)
In 70–80% of cases, bleeding ceases spontaneously
Differential diagnosis: other causes of lower gastrointestinal bleeding (e.g., hemorrhoidal bleeding)
Treatment
Endoscopic hemostasis during colonoscopy (e.g., epinephrine injection, thermal coagulation, ligation)
Angiography with vessel embolization
Fistulas [27]
Colovesical (most common)
Symptoms
Pneumaturia and fecaluria
May cause recurring urinary tract infections, including urosepsis
Diagnosis: CT with oral contrast
Localized thickening of the colon and bladder
Air or contrast material in the bladder
Treatment
Resection and primary anastomosis
Antibiotics if surgery is not possible
Other forms: colovaginal, coloenteric, colocutaneous
Inflammation (diverticulitis)
Abscess
Peridiverticular localization
Causes symptoms similar to those of acute diverticulitis
Suspect an abscess in patients with persistent fever and abdominal pain despite antibiotic treatment.
Perforation
Symptoms of acute abdomen caused by
Rupture of an inflamed diverticulum → free communication with the peritoneum → generalized fecal peritonitis
Rupture of a diverticular abscess → generalized purulent peritonitis
Intestinal obstruction
Causes
Narrowing due to inflammatory swelling
Compression through abscesses
Ileus caused by localized irritation
Symptoms
Abdominal pain and distention
Constipation
Nausea, vomiting
Acute abdomen

49
Q

hepatitis signs and symptoms & investigations

A

Symptoms:
The most commonly recognised sign is an acute attack with icteric effects. Most people will make a complete recovery. Symptoms may last a few days to a couple of weeks, and include:
Fever
Malaise
Upper abdominal discomfort
Jaundice – often lasts a few days to a few weeks and then subsides. It is also one of the last symptoms to develop – it may not develop until 2 weeks after other symptoms, or indeed may not develop at all (anicteric hepatitis).
Symptoms usually last 3-6 weeks and then subside.
Development of ascites and oedema is very uncommon, but can still occur in the most serious cases.
The disease may have several ‘waves’ where the patient has several episodes of worsening symptoms before they make a full recovery
With some viruses (B, C &D) there is a chance of developing chronic disease.
Very few people die from acute viral hepatitis.
Fulminant liver failure can very rarely occur. The term ‘fulminant’ means the failure occurs very suddenly and is severe, but it is reversible. Technically, fulminant liver disease is where there is severe encephalopathy that develops within 2 weeks of the onset of jaundice.

Clinical examination
Some or all of the following features may be present:
Spider naevi (these will disappear after recovery). Up to 5 of these are normal – it is only when you have more that they suggest a pathological cause.
Jaundice
RUQ tenderness.
Mild hepatomegaly – although in fulminant liver failure the liver will shrink rapidly.
Splenomegaly is uncommon, and suggests either underlying pre-existing liver disease, or perhaps infection with a different virus, such as Epstein Barr, or cytomegalovirus.
Persistent nausea and vomiting suggest severe hepatitis, and these increase the risk of hypoglycaemia.
Hepatic encephalopathy, although rare, indicates the severity of the disease – basically indicating liver failure.

Investigations/Viral markers:

Remember that antibodies are produced by the immune system against pathogen proteins. Antigens are proteins that are targeted by the antibodies, in this scenario they are part of the virus.

Surface antigen (HBsAg) – active infection
E antigen (HBeAg) – marker of viral replication and implies high infectivity
Core antibodies (HBcAb) – implies past or current infection
Surface antibody (HBsAb) – implies vaccination or past or current infection
Hepatitis B virus DNA (HBV DNA) – this is a direct count of the viral load
When screening for hepatitis B, test HBcAb (for previous infection) and HBsAg (for active infection). If these are positive then do further testing for HBeAg and viral load.

HBsAb demonstrates an immune response to HBsAg. The HBsAg is given in the vaccine, so having a positive HBsAb may simply indicate they have been vaccinated and created an immune response to the vaccine. The HBsAb may also be present in response to an infection. The other viral markers are necessary to distinguish between previous vaccination or infection.

Hepatitis B c antibodies (HBcAb) can help distinguish acute, chronic and past infections. We can measure IgM and IgG versions of the HBcAb. IgM implies an active infection and will give a high titre with an acute infection and a low titre with a chronic infection. IgG indicates a past infection where the HBsAg is negative.

Hepatitis B e antigen (HBeAg) is important. Where the HBeAg is present it implies the patient is in an acute phase of the infection where the virus is actively replicating. The level of HBeAg correlates with their infectivity. If the HBeAg is higher, they are highly infectious to others. When they HBeAg is negative but the hepatitis B e antibody is positive this implies they have been through a phase where the virus was replicating and but the virus has now stopped replicating and they are less infectious.

50
Q

Hepatitis A

A

Aetiology and epidemiology
Hepatitis A is most common hepatitis virus. It is often seen in epidemics (usually in children), and most commonly occurs in the autumn
Often found in communities with overcrowding
Commonly found in water – partiuclarly beware salads washed in contaminated water – often patient may have had recent travel abroad
Can also be sexually transmitted
Very common, particularly in developing world. In some countries, 100% of the population has been infected by the age 10 – but the disease is often asymptomatic in children and so may go unnoticed
In the developed world, 5-40% of the population have had the infection
There are sometimes ‘mini-epidemics’ in children in nursey / day-care centres
Often found after flooding
Shellfish have also been implicated in transmission – possibly due to human sewage reaching their habitat
There isn’t a carrier state
The virus can be killed by boiling water for 10 minutes.
Vaccination provides immunity for 10 years (often considered life-long)
Pathology
Incubation 2-6 weeks
Oro-faecal transmission. Viral shedding occurs in the faeces at about the time of the onset of symptoms
RNA virus
Signs and Symptoms
Many patients are asymptomatic. They may never know they have had the disease
Very rarely it can be lifethreatening (mortality 0.3-2%)
Symptoms depend on age:
< 4 years – 90% anicteric
15 years – 40 – 70% icteric
May be a prodromal phase
Jaundice, malaise, abdominal pain, nausea & fever – usually lasting around 2 weeks
Jaundice may occur 1-2 weeks after other symptoms. As the jaundice worsens, other symptoms may subside, but the urine may become dark and the stools pale, due to intrahepatic cholestasis (i.e. the intrahepatic bile ducts get blocked)
10% of patients get splenomegaly resulting in a palpable spleen
Other non-specific symptoms – including distaste for cigarettes!

Investigations
Only one antigen has been found – HAV. However, it is not tested for as levels vary during the course of an infection. Instead we test for anit-HAV – the levels of which are more predictable.
You can see IgM in the blood for the first 6 weeks, then IgG after that.
Diagnosis – abnormal LFTs / +ve IgM anti-HAV
ALT > AST
AST may be > 1000
IgM HAV antibody may be positive for up to 6 months after clinical feautres subside
IgG antibody positive indicates past exposure
Prognosis – can be determined by INR
Treatment
Usually self limiting. Does not usually require hospital admission in uncomplicated cases.
Advise no unprotected sexual contact for 7 days after the onset of jaundice
Rest and dietary modification seem to have little effect – basically, you just have to sit it out!
In most people, the severity of the virus peaks 4 weeks after infection, and symptoms will be virtually gone 2 weeks later.
Complications
Acute fulminant liver failure is rare – 0.1 to 0.4%
Not associated with chronic liver disease
The mortality rate is low – around 0.3%, increasing to 2% with age and other co-morbities.
Extra-hepatic complications are very rare, but include arthritis, myocarditis and renal failure.
10% of patient’s will have a relapse before recovery.
Some patients may ‘feel ill’ for months after the disease – this is known as post hepatitis syndrome and it is a functional disease that is treated by reassurance
Prophylaxis
Hepatitis A vaccine is recommended for travellers before they visit:

Indian subcontinent
Africa
Central &amp; South America
The Far East
Eastern Europe
Hepatitis A vaccine schedule:

Initial dose 4-6 weeks before travel
Provides protection for 12 months
Booster 6-12 months later
Provides immunity for 10 years (often considered life-long)

51
Q

Hepatitis B

A

Introduction
Hepatitis B is an infection of the liver, caused by the hepatitis B virus (HBV). It is the most common cause of hepatitis worldwide, and there are believed to be over 350 million chronic cases. It is endemic in Asia and the Far East (up to 10% have chronic hepatitis B), but rare in the developed world. In the UK, bout 1 in 350 people have hepatitis B.

Spread occurs via infected blood – e.g. via sexual intercourse, vertical transmission from mother to baby, and historically it was also seen in blood transfusions.

The incubation period is between 6-23 weeks.

The serologically important factors, which help it determining immunisation status as well as disease status include:

Surface antigen – HBsAg, and surface antibody (anti-HBs)
Core antigen – HBcAg, and core antibody (anti-HBc)
e antigen – HBeAg
All patients with hepatitis B need assessment and management.

Hepatitis B is preventable by vaccination. In both the UK and Australia, this is now part of the routine vaccination schedule in children. Usually three doses are required to confer immunity.

Most cases in the UK are the result of injecting drug use or sexual exposure.

About 10% of cases of infection will go on to develop chronic hepatitis B. Chronic disease is defined as detectable levels of surface antigen (HBsAg) 6 months after infection.

Epidemiology and Aetiology
Major health problem – 350 million chronic cases worldwide
Incubation 1 – 4 mths
Parenteral transmission – sexual, IV, perinatal
0.5% of UK population are carriers, but this is as much as 10-15% in some countries in the developing world. in some far eastern countries, 1/3 of people are carriers
In UK, 1 in 350 have chronic hepatitis B infection
In recent years, rates have declined amongst younger populations due to vaccination
Risk factors
IV drug use
Multiple sexual partners
Occupational exposure – e.g. healthcare workers
Close family or contact who is infected or a carrier
Receiving regular blood products
Travel to high risk areas
Children born to mothers who are carriers or infected
Prison inmates
In the UK most cases are due to sexual exposure or IV drug use
Presentation
May be asymptomatic, or have minor flu-like symptoms. Symptomatic individuals may present with:

Fever
Malaise
Faitgue
Joint pains
Urticaria
Pale stools / dark urine / jaundice
Jaundice occurs in 10% of newly infected children and up to 50% of newly infected adults
Acute Hepatitis B
Incubation period 6 weeks – 6 months
1% of patients will develop liver failure
90% will recover
10% will go on to develop chronic hepatitis B infection
Jaundice, malaise, abdominal pain, nausea & fever – 1 to 3 months
70% anicteric (not jaundiced)
Maternal transmission is different
90% of newborns born to infected mothers will have chronic hepatitis B infection
Up to 25% will have chronic complications. Including:
Cirrhosis
Hepatocellular carcinoma
Chronic Hepatitis B
10% of cases of infection will go on to develop chronic hepatitis B. The risk of developing chronic disease depends on the age at which the infection is acquired. The risk in neonates from maternal transmission >90%, whilst in adults, it is around 10%, and in children who acquire the infection after birth the risk is even lower. Many patients are “healthy carriers” without any ongoing symptoms.

In healthy patients who are non-drinkers, with normal LFTs the disease often has a benign course
In those with more severe disease, then there is a risk of liver cirrhosis and hepatocellular carcinoma (HCC)
Diagnosis
Derranged LFTs
↑bilirubin
↑ALT / AST
↑ALP
Positive HBsAg
Present in blood for 1-6 months post exposure
If present after 6 months then this is diagnostic for carrier status
HBeAg
Present from 6 weeks to 3 months after exposure
May or may not be present in chronic disease
Indicates period of high infectivity – as it shows high levels of viral replication in the blood
Indicates worse prognosis
Anti-HBs
Antibodies to surface antigen
Appear 10 months after infection
Imply immunity
Also seen in previous vaccinated individuals
HBV genotyping
Quantitative hepatitis B virus DNA
Higher viral load is associated with worse prognosis and is correlated to liver damage
HIV and Hepatitis C
Patients should also be screened for these two diseases
Understanding Hepatitis B testing
There are several antigens and antibodies that are present during, and following Hepatitis B infection. Understanding which of these is present and when can help you identify if a persen has a current or past infection, and their immunity status.

At the most simple level:

HBsAg negative – do not have current HBV
HBsAg positive – have acute or chronic HBV and require further tests. If positive for >6 months are considered to have chronic HBV
Pathology
In chronic disease, viral DNA may become incorporated into host DNA. HbsAg is present on the surface of infected hepatocytes, and this causes T cells to induce apoptosis in these cells. The pathogenesis is different to that of HAV (Hepatitis A Virus) , where the apoptosis is not induced by the immune system, but by the infected cell itself – thus HAV does not have the ability to cause chronic disease.

Complications
Acute liver failure rare – 0.1 to 0.5%
Chronic Hepatitis B is specified as viraemia at 6 months after initial diagnosis/sympotms
Occurs in about 10% of adult infection, and 90% of vertical transmission cases
Chronic disease can be divided into e-antigen positive and e-antigen negative
E-antigen positive disease is associated with higher viral replication, and therefore higher risks of infectivity
Chronic infection benign in the majority of patients – especially in non-drinks with normal liver function
Chronic infection depends on age of initial infection:
90% of people who get the disease in vertical transmission (i.e. from their mum) will develop chronic disease
5-10% of adults who contract hepatitis B will develop chronic disease
30% of people with chronic infection will get cirrhosis
In chronic HBV 40% men and 15% women die from liver failure
Cirrhosis
HIV and HepC infection increase the likelihood of cirrhosis
Hepatocellular carcinoma
Can develop without cirrhosis
Management
If acutely unwell – will need hospital admission. Otherwise can be managed in primary care, with non-urgent referral to gastroenterologist or infectious diseases specialist upon confirmation of diagnosis (positive HBsAg).

Symptomatic management is often all that is required in the acute phase. Itching due to jaundice may be difficult to manage.

It is important to check for other commonly associated infections, including HepC and HIV.

10% of patients with HIV will also be infected with HBV
Liver complications are more common in those with co-infection with HIV. However, conversely, HBV is not thought to affect HIV disease progression.
If not already done, ensure diagnosis is confirmed with serological testing, and FBC and Lots are performed.

Antiviral therapy – only for selected patients with chronic infection – requires specialist input – usually those patients with HBeAg positive disease and liver disease
Interferon is often first line
48 week course
Many side effects
Lamivudine
Entecavir
Tenofovir (adefovir)
Variable success rates
The aim of treatment is to reduce the risk of complications. Therefore patients who already have cirrhosis are likely not to benefit. Other factors when deciding who to treat include:
HBe-antigen status
Elevated Transaminase
HBV DNA levels
Not cirrhotic
Prevented by vaccination
Lifestyle advice
Counsel about the diagnosis and the course of the disease, including advise not to donate blood and the route of transmission
Advise alcohol abstinence – especially if LFTs are abnormal
Safe sex advice
Advise partners become vaccinated, and to avoid sexual contact (including oral sex) until vaccination is complete, or until the patient becomes non-infectious
Reducing transmission
Vertical transmission

To babies of infected mothers:

Give HBIG within 24 hours of birth
Give full course of vaccination
These two can be given simultaneously (at different sites)
This reduces vertical transmission by 90%
Intimate contacts

Contact tracing may be required
This can be difficult in chronic infection
Infectious period is from 2 weeks before jaundice, until HBsAg negative serology
HBIG can be given to contacts after a single exposure (e.g. sexual contact or needle stick) – should be given as soon as possible (most effective in first 48 hours) but can be given up to a week after exposure
Prognosis
80% of patients will have full recovery
10% develop chronic hepatitis
Associated with cirrhosis and liver carcinoma
Risk of cirrhosis is 10-20% without antivirals
Once cirrhosis has developed, there is a high risk of culminant liver failure
5-year survival in decompensated cirrhosis is 15%
10% go on to be “carriers”

52
Q

Hepatitis C

A

Introduction
First discovered in 1989, the hepatitis C virus (HCV) is blood-borne form of viral hepatitis. The incubation period is 6-9 weeks.

Transmission is usually via blood products or vertical transmission from mother to foetus. Sexual transmission is possible but rare.

HCV infection can be acute or chronic. The acute infection is usually asymptomatic, and often not discovered for years after infection (frequently discovered incidentally). Chronic infection is often discovered when patients develop serious liver disease later in life. It can cause cirrhosis, decompensated liver disease, and death.

50% – 80% of those exposed to the virus will develop chronic infection, the remainder will spontaneously clear the virus. Spontaneous late clearance of the disease is very rare.

There is no vaccine available for the prevention of HCV.

In many cases, chronic HCV can be cured with antiviral combination therapy. Previously this involved genotyping of the HCV virus for each individual, and complex medication regimens, often self-administered SC (subcutaneous) injections, under the supervision of a gastroenterologist. However, in recent years, new oral agents (DAA – direct acting antivirals) which are effective against all strains (and thus don’t require genotyping) have been developed, with a cure rate of >90%, and in some cases (e.g. in certain circumstances in Australia) can even be prescribed by a GP. It is likely that the NHS in the UK will adopt a similar model (some areas in the UK may already be doing this as of 2019).

Epidemiology
Prevalence in the UK is about 0.02%. In Africa it is about 6%, and in Egypt is as high as 19%.
About 80% of those in the UK with haemophilia have hepatitis C – as a result of previously blood transfusion, received before hepatitis C was recognised or screened for.
Worldwide it is the leading cause of liver disease.
180 million people thought to be infected worldwide
95% of new cases in UK are due to IV drug use. Sexual transmission is also possible but less common
6 genetic subtypes – genotyping for the type of Hep C is important for treatment (less so with the advent of new oral treatments)
90% of cases are either type 1 or type 3
HCV prevalence in 1999
HCV prevalence in 1999. This file is taken from wikimedia commons and is licensed under the Creative Commons Attribution-Share Alike 3.0 Unported license.
Risk factors

IV drug use
Blood transfusion before 1991
Haemodialysis
Sexual contact with an infected partner
Needlestick injuries in healthcare workers
Vertical transmission from mother to baby
Co-existing HIV
Medical or dental treatments in countries where sterilisation procedures may be inadequate – particularly in high prevalence countries
Household transmission is extremely rare.

Presentation
The vast majority of patients are asymptomatic. Many cases are discovered after LFTs show raised amintransferase (ALT). Any patient with persistently elevated ALT should be screened for HCV.

85% anicteric (not jaundiced).
10-15% of patients will have jaundice and perhaps other general symptoms suggestive of hepatitis: jaundice, malaise, abdominal pain, nausea & fever – mild
Diagnosis
Blood tests

Abnormal LFTs
Raised ALT
Low albumin, raised bilirubin, raised INR
Signs of liver synthetic dysfunction and suggest cirrhosis
Positive anti-HCV antibodies
If positive – perform HCV PCR, and viral load
HCV antibodies do not give information about cure. HCV antibodies typically remain positive for life. HCV viral load status is required to know if cure is successful.
Also test for HBV and HIV if HCV is positive – vaccinate as appropriate depending on the results
FBC
Check for anaemia
Low platelets may be a sign of portal hypertension
AST to platelet ratio (APRI) can be used to screen for cirrhosis. A score of <1.0 with no other signs of cirrhosis indicates low risk
U+Es
Some treatments are not recommended if eGFR low
Liver Assessment

If the APRI (see above) is <1.0 and no other signs of cirrhosis, then no further investigations are required- as the risk of liver damage is negligible
APRI calculator
If APRI >1.0 – further investigation and specialist referral is required
Liver USS
Helps assess for hepatocellular carcinoma, portal hypertension, and evidence of cirrhosis
Fibroscan
A special type of ultrasound that measures the “stiffness” (elasticity) of the liver, and is used to diagnose cirrhosis
Shear wave elastograhpy is another similar technique, but less well evaluated. It can be performed in some centres as part of a liver USS. Fibroscan requires different equipment to regular USS.
If any evidence of cirrhosis – patient will need to be referred to a gastroenterologist for further management
Further Investigations

Consider these if known chronic HCV, and signs of liver failure

Gastroscopy – to exclude varices
Bone densiometry for osteoporosis
Other Signs

Acute liver failure rare – < 0.1%
Chronic infection very common – 85%.
1 – 4 % of these patients will have HCC (hepatocellular carcinoma) as a result
Management
Antiviral therapy is the mainstay of treatment for chronic infection. Treatment with Interferon has largely been superseded by use of DAA (direct-acting antivirals)

DAA:

Cure rate>90%
Oral medication – usually a daily regimen (may be poor adherence)
Treatment usually lasts 8-12 weeks
Sometimes up to 24 weeks
Does not require intensive monitoring (unlike interferon)
Side effects are uncommon and usually mild and include:
Fatigue
Headache
Insomnia
Nausea
Essential for patients to have follow-up blood test at 12 weeks to determine if treatment has been successful (re-check HCV viral load)
Exact medications used often depends on the HSV genotype
Examples include:
Sofosbuvir + ledipasvir (Harvoni®)
Sofosbuvir + Valpatasvir
Sofosbuvir + ribavirin
Side effects:
Usually well tolerated
Side effects may include headache, fatigue, nausea, cough. All are usually mild
Ribavarin is often associated with more significant and troublesome side effects (anaemia (haemolytic), rash, anxiety, insomnia, dyspnoea). Avoid is Hb is low before treatment is started. Strongly teratogenic for both male and female patients – advise to use two forms of contraception whilst taking the medication and for 6 months afterwards. Contraindications include pregnancy, anaemia, heart disease.
Be wary of patients with low eGFR (<30) as they may need adjusted doses or alternative drugs. Seek specialist advice
Monitoring
Check FBC, LFTs and U+Es every 4 weeks, and again 12 weeks after the end of treatment
Abnormal LFTs typically resole after infection has been treated. If they persist, look for another cause
If tests are normal at 12 weeks after treatment, no further ongoing monitoring is required
If FBC declines on ribavirin then reduce dose, or consider stopping ribavirin (discuss with specialist)
Sustained virological response (SVR) is the term used to describe a successful treatment, as determined by a negative HCV PCR at 12 weeks after the end of treatment
Advise safe drinking limits and living alcohol intake (to reduce the risk of developing cirrhosis)
Ensure testing and vaccination for HIV and HBV before commencing treatment
In Australia, patients are suitable for GP initiated treatment if:
No cirrhosis
No HIV or HBV
Not pregnant
Referral anybody else who does to meet the above criteria to a gastroenterologist
GPs need prior experience in prescribing under the supervision of a specialist before prescribing independently
Failure of treatment is rare. Consider:
Poor adherence
Re-infection
Drug resistance
Reducing transmission risk

Counsel the patient about methods of transmission
Blood-to-blood contact only – e.g. mother to baby, can occur sexually (not common), and receive blood products (blood screened in most but not all countries)
Advise to use barrier methods of contraception to reduce risk of sexual transmission
Cannot be caught through normal social contact, hugging, kissing, sharing food or cutlery, or shared toilet facilities. Mosquitos do not transmit HCV
Avoid sharing items used in intimate personal care:
Toothbrushes
Razors
Tweezers
Jewellery that involves body piercings
Scissors
Nail clippers
Cuts and abrasions
Cover with a waterproof dressing
Blood spills and contamination (e.g. after simple cuts at home)
Wash hands before and after cleaning up the blood spill
Use disposable gloves
Use disposable materials (e.g. paper towels) to mop up with
Clean the area with detergent and then with bleach
Remind them not to donate blood or organs
Advise patient to inform health care workers of their diagnosis
Advise the patient to inform their partner of their diagnosis
Pregnancy and Breastfeeding

Mother to baby transmission risk is about 5%
C-section does not reduce the risk of transmission
Test babies after the age of 18 months – as maternal antibodies can persist until this age
Breastfeeding is safe – unless nipples are bleeding or cracked – advise mums to discard this breast milk in these circumstances and can resume once nipples have recovered
Prognosis
30% of chronic HCV develop cirrhosis within 20 – 30 years
Male patients are more likely to develop fibrosis with chronic infection.
Progression from chronic hepatitis to cirrhosis takes 20-40 years. This happens more quickly in male patients, immunosupressed patients, and those who drink a lot of alcohol.
About 20% of HCV patients will develop cirrhosis within 20 years
5 year survival once cirrhosis has set in is 95%. 10-year is 81%
¼ of patients will develop complications, such as ascites. Once these have developed, 5-year survival drops to 50%
2-5% of cirrotic patients will develop hepatocarcinoma.
Successful treatment:
Removes infection risk
Can lead to regression of cirrhosis and liver fibrosis
Reduces risk of hepatocellular carcinoma
Reduces mortality
Does not prevent re-infection. Patients can contract HCV again.
If cirrhosis present, then even despite successful treatment, it is recommended patients have the following follow-up monitoring:
Liver USS every 6 months (screening for HCC)
Gastroscopy every 1-2 years (screening for varices)
Bone densitometry (DEXA) at diagnosis and every 2-3 years (screening for osteoporosis)

53
Q

Hep D

A

You can only contract this is you are also currently suffering from hepatitis B.
You can get two types of infection:
Normal co-existant infection – 90% of cases – this actually reduces the severity of the hep B infection! This is because infection with hepatitis D can reduce the replication rate of the hepatitis B virus. These people will usually make a full recovery from an unremarkable acute hepatitis.
Superinfection – 10% of cases – this greatly worsens prognosis. It is due to chronic infection with both viruses. It is not really known what causes this. It can be detected by very high levels of anti-HDV in the blood.
With both types of co-infection there is an increased risk of fulminant liver disease.
Common mode of transmission is IV drug use (in the UK). In other parts of the world it is transmitted by close personal contact.
It is common in some parts of the Mediterranean, as well as Africa and South America.
There is only one identified antigen – HDV. The test for infection with hep D is anti-HDV.
To prevent hep D, basically, you need to prevent hep B infection!

54
Q

Hep E

A

Hepatits E is clinically very similar to Hepatitis A.

Aetiology

Water borne similar to hep A(shell fish and water melons!). 30% of dogs, rodents and pigs carry the virus
Commonly seen in Asia and the Far East
Presentation
Very similar to that of hep A:

Many patients are asymptomatic. They may never know they have had the disease
Very rarely it can be lifethreatening
May be a prodromal phase
Jaundice, malaise, abdominal pain, nausea & fever – usually lasting around 2 weeks
Other Factors:

Does not have a chronic state
25% mortality in pregnancy. Often baby will also die. This does not happen with hep A.
During an acute infection, IgM antibodies (anti-HEV) are present.
There is no active or passive immunity to hepatitis E

55
Q

Autoimmune Hepatitis

A

Introduction
Autoimmune hepatitis (AIH) is a cause of chronic hepatitis. It is also sometimes referred to as chronic active hepatitis. Any hepatitis that typically lasts >6 months is referred to as chronic.
It is characterised by:
Increased IgG levels
Antibodies against:
Liver specific proteins
Non-live specific proteins
Mononuclear infiltrate within the liver
Monocytes, macrophages, lymphocytes, plasma cells, macrophages and mast cells
There are three types of autoimmune hepatitis, which can be distinguished histologically:
AIH – 1 – shows the presence of anti-smooth muscle antibodies (ASMA’s), andsometimes anti-nucelar antibodies (ANA’s)
AIH – 2 – has liver-kidney microsomal type 1 antibodies (LKM-1) and sometimes anti-liver cytosol 1 antibodies (anti-LC1).
AIH – 3 – has antibodies to soluble liver proteins or liver-pancreas antigen.
Actin antibody is related to prognosis. Those with actin antibody presence are more likely to require transplant than those without.
HLA-DR3 and HLA-DR4 are associated with increased risk for AIH.
In HLA-DR3 patients the disease tends to occur earlier, and is more likely to require transplantation.
ANA’s – antinuclear antibodies are present in many autoimmune diseases: Rheumatoid arthritis, SLE, scleroderma, polymyositis, dermatomyositis, as well as AIH. In many of these diseases the ANA blood test can help diagnosis.
Epidemiology
Autoimmune hepatitis is rare. In Western Europe and the US there are <1 case per 100 000
The M:F ratio is 1:3
Peaks of incidence: Age 10-20, and 45-70. The lower peak is more common and:
§ Accounts for about half of cases
§ Typically premenstrual girls
§ 80% of AIH-2 patients are children
Other autoimmune disease in the patient, or first degree relatives:
20% of patients have another autoimmune disease
40% of patients have a first degree relative with an autoimmune disease
Pathology
The disease is thought to occur in genetically pre-disposed individuals who come into contact with an environmental trigger.
Many patients with autoimmune hepatitis have a low CD8 count. They also often have a genetically determined reduced level of CD4 T cells, which are associated with fighting viral infection. Thus it may be possible that a viral infection triggers off the production of autoantibodies in susceptible individuals. For example, the hepatitis C virus has a close relation to LKM-1 antibody mentioned above.

Drugs and Environmental agents have also been cited as possible causes.

Very rarely, AIH may be the result of Hep A, Hep B, EBV
More commonly AIH is caused by Hep C
Autoimmune Hepatitis histology
Autoimmune Hepatitis histology – showing the characters feature of portal mononuclear cell inflammatory infiltrate extending into the lobule (interface hepatitis).
Presentation
May initially be acute presentation, but will always eventually become chronic.
Up to 50% of patients present as if an acute viral hepatitis.
In other cases, there is typically an insidious onset
Non-specific symptoms:
Fatigue
Anorexia
Weight loss
Amenorrhoea
Behavioural changes
Abdominal pain
Liver-specific problems:
Epistaxis – nose bleeds
Easy bruising
Jaundice
Hepatomegaly / splenomegaly (50-60% of cases)
Ascties
Signs associated with chronic liver disease: Asterixis, Palmar erythema, Spider naevi
Prognosis
Many patients will have severe cirrhosis by the time of diagnosis
About 25% of patients will have a liver transplant
Related Articles
Abdominal Artery Ischaemia and Occlusion
Abdominal Examination
Abdominal X-Ray
Alcohol and Alcohol Abuse
Alcoholic Hepatitis

56
Q

IBD

UC vs Crohn’s disease

A

Inflammatory bowel disease is the umbrella term for two main diseases causing inflammation of the GI tract: Ulcerative Colitis and Crohn’s disease. They both involve inflammation of the walls of the GI tract and are associated with periods of remission and exacerbation.

Crohn’s versus Ulcerative Colitis
Crohn’s and ulcerative colitis have features that are distinct from each other that are commonly tested in exams and dictate different management, so it is worth learning these.

Crohn’s (crows NESTS)

N – No blood or mucus (less common)

E – Entire GI tract

S – “Skip lesions” on endoscopy

T – Terminal ileum most affected and Transmural (full thickness) inflammation

S – Smoking is a risk factor (don’t set the nest on fire)

Crohn’s is also associated with weight loss, strictures and fistulas.

Ulcerative Colitis (remember U – C – CLOSEUP)

C – Continuous inflammation

L – Limited to colon and rectum

O – Only superficial mucosa affected

S – Smoking is protective

E – Excrete blood and mucus

U – Use aminosalicylates

P – Primary Sclerosing Cholangitis

Presentation
Diarrhoea
Abdominal pain
Passing blood
Weight loss

Testing
Routine bloods for anaemia, infection, thyroid, kidney and liver function
CRP indicates inflammation and active disease
Faecal calprotectin (released by the intestines when inflamed) is a useful screening test (> 90% sensitive and specific to IBD in adults)
Endoscopy (OGD and colonoscopy) with biopsy is diagnostic
Imaging with ultrasound, CT and MRI can be used to look for complications such as fistulas, abscesses and strictures.

Management of Crohn’s
This section is based on NICE guidelines last updated May 2016. Please see the full guidelines and talk to seniors before treating patients.

Inducing Remission

First line: Steroids (e.g. oral prednisolone or IV hydrocortisone)
If steroids alone don’t work, consider adding immunosuppressant medication under specialist guidance:

Azathioprine
Mercaptopurine
Methotrexate
Infliximab
Adalimumab

Maintaining Remission

Tailored to individual patients based on risks, side effects, nature of the disease and patient’s wishes. It is reasonable not to take any medications whilst well.

First line:

Azathioprine
Mercaptopurine
Alternatives:

Methotrexate
Infliximab
Adalimumab

Surgery

When the disease only affects the distal ileum it is possible to surgically resect this area and prevent further flares of the disease. Crohns typically involves the entire GI tract

Surgery can also be used to treat strictures and fistulas secondary to Crohns disease.

Management of Ulcerative Colitis
This section is based on NICE guidelines last updated June 2013. Please see the full guidelines and talk to seniors before treating patients.

Inducing Remission

Mild to moderate disease

First line: aminosalicylate (e.g. mesalazine oral or rectal)
Second line: corticosteroids (e.g. prednisolone)
Severe disease

First line: IV corticosteroids (e.g. hydrocortisone)
Second line: IV ciclosporin

Maintaining Remission

Aminosalicylate (e.g. mesalazine oral or rectal)
Azathioprine
Mercaptopurine

Surgery

Ulcerative colitis typically only affects the colon and rectum. Therefore, removing the colon and rectum (panproctocolectomy) will remove the disease. The patient is then left with either a permanent ileostomy or something called an ileo-anal anastomosis (J-pouch). This is where the ileum is folded back in itself and fashioned into a larger pouch that functions a bit like a rectum. This “J-pouch” which is then attached to the anus and collects stools prior to the person passing the motion.

57
Q

Gastroenteritis

A

Acute gastritis is inflammation of the stomach and presents with nausea and vomiting. Enteritis is inflammation of the intestines and presents with diarrhoea. Gastroenteritis is inflammation all the way from the stomach to the intestines and presents with nausea, vomiting and diarrhoea.

The most common cause of gastroenteritis is viral. It is very easily spread and patients presenting with gastroenteritis often have an affected family member or contact.

It is essential to isolate the patient in a healthcare environment such as a hospital ward as they can easily spread it to other patients.

Most people recover well but beware that it can be potentially fatal especially in very young or old patients or those with other health conditions.

Viral Gastroenteritis
Viral gastroenteritis is the most common. Viral gastroenteritis is highly contagious.

Rotavirus
Norovirus
Adenovirus is a less common cause and presents with a more subacute diarrhoea

E. coli
Escherichia coli (E. coli) is a normal intestinal bacteria. Only certain strains cause gastroenteritis. It is spread through contact with infected faeces, unwashed salads or water.

E. coli 0157 produces the Shiga toxin. This causes abdominal cramps, bloody diarrhoea and vomiting. The Shiga toxin destroys blood cells and leads to haemolytic uraemic syndrome (HUS).

The use of antibiotics increases the risk of haemolytic uraemic syndrome therefore antibiotics should be avoided if E. coli gastroenteritis is considered.

Campylobacter Jejuni
Campylobacter is a common cause of travellers diarrhoea. It is the most common bacterial cause of gastroenteritis worldwide. Campylobacter means “curved bacteria”. It is a gram negative bacteria that is curved or spiral shaped. It is spread by:

Raw or improperly cooked poultry
Untreated water
Unpasteurised milk
Incubation is usually 2-5 days. Symptoms resolve after 3-6 days. Symptoms are:

Abdominal cramps
Diarrhoea often with blood
Vomiting
Fever
Antibiotics can be considered after isolating the organism where patients have severe symptoms or other risk factors such as HIV or heart failure. Popular antibiotic choices are azithromycin or ciprofloxacin.

Shigella
Shigella is spread by faeces contaminating drinking water, swimming pools and food. The incubation period is 1-2 days and symptoms usually resolve within 1 week without treatment. It causes bloody diarrhoea, abdominal cramps and fever. Shigella can produce the Shiga toxin and cause haemolytic uraemic syndrome. Treatment of severe cases is with azithromycin or ciprofloxacin.

Salmonella
Salmonella is spread by eating raw eggs or poultry and food contaminated with infected faeces of small animals. Incubation is 12 hours to 3 days and symptoms usually resolve within 1 week. Symptoms are watery diarrhoea that can be associated with mucus or blood, abdominal pain and vomiting. Antibiotics are only necessary in severe cases and guided by stool culture and sensitivities.

Bacillus Cereus
Bacillus cereus is a gram positive rod that is spread through inadequately cooked food. It grows well on food not immediately refrigerated after cooking. The typical food is fried rice left out at room temperature.

Whilst growing on the food it produces a toxin called cereulide that causes abdominal cramping and vomiting within 5 hours of ingestion. When it arrives in the intestines it produces different toxins that cause a watery diarrhoea. This occurs more than 8 hours after ingestion. All of the symptoms usually resolves within 24 hours.

Therefore the typical course is vomiting within 5 hours, then diarrhoea after 8 hours, then resolution within 24 hours.

TOM TIP: The typical exam patient develops symptoms soon after eating leftover fried rice that has been left at room temperature. It has a short incubation period after eating the rice and they then recover within 24 hours. Examiners like this question because the course of bacillus cereus is easy to distinguish from the other causes of gastroenteritis.

Side Note: The other place you may come across bacillus cereus is in intravenous drug users (IVDU) that develop infective endocarditis. Staphylococcus is the most common cause of IE in IVDUs but bacillus cereus is one to keep in mind.

Yersinia Enterocolitica
Yersinia is a gram negative bacillus. Pigs are key carriers of Yersinia and eating raw or undercooked pork can cause infection. It is also spread through contamination with the urine or faeces of other mammal such as rat and rabbits.

Yersinia most frequently affects children causing watery or bloody diarrhoea, abdominal pain, fever and lymphadenopathy. Incubation is 4-7 days and the illness can last longer than other causes of enteritis with symptoms lasting 3 weeks or more. Older children or adults can present with right sided abdominal pain due mesenteric lymphadenitis (inflammation in the intestinal lymph nodes) and fever which can give the impression of appendicitis.

Antibiotics are only necessary in severe cases and guided by stool culture and sensitivities.

Staphylococcus Aureus Toxin
Staphylococcus aureus can produce enterotoxins when when growing in food such as eggs, dairy and meat. When eaten these toxins cause small intestine inflammation. This causes symptoms of diarrhoea, perfuse vomiting, abdominal cramps and fever. These symptoms start within hours of ingestion and settle within 12-24 hours. It is not actually the bacteria causing the enteritis but the staphylococcus enterotoxin.

Giardiasis
Giardia lamblia is a type of microscopic parasite. It lives in the small intestines of mammals. These mammals may be pets, farmyard animals or humans. It releases cysts in the stools of infected mammals. These cysts then contaminate food or water and are eaten to infect a new host. This is called faecal-oral transmission.

Infection may not cause any symptoms or it may cause chronic diarrhoea. Diagnosis is made by stool microscopy. Treatment is with metronidazole.

Principles of Gastroenteritis Management
Good hygiene helps prevent gastroenteritis. When patients develop symptoms they should immediately be isolated to prevent spread. Barrier nursing and rigorous infection control is important for inpatients to prevent spread to other patients.

A sample of the faeces can be tested with microscopy, culture and sensitivities to establish the causative organism and antibiotic sensitivities.

Assess patients for dehydration. Attempt a fluid challenge and if they are able to tolerate oral fluid and are adequately hydrated consider outpatient management. If not vomiting and tolerated then rehydration solutions (e.g. dioralyte) can be used. If dehydrated then intravenous fluids can be used to rehydrate them and prevent dehydration until oral intake is adequate again.

Slowly introduce a light diet in small quantities once oral intake is tolerated again. Advise them to stay off work or school for 48 hours after symptoms have completely resolved.

Antidiarrhoeal medication such as loperamide and antiemetic medication such as metoclopramide are generally not recommended but may be useful for mild to moderate symptoms. Antidiarrhoeals should be avoided in e. coli 0157 and shigella infections and where there is bloody diarrhoea or high fever.

Antibiotics should only be given in patients that are at risk of complications and once the causative organism is confirmed.

Post Gastroenteritis Complications
The are possible post-gastroenteritis complications:

Lactose intolerance
Irritable bowel syndrome
Reactive arthritis
Guillain–Barré syndrome

58
Q

IBS

A

Irritable bowel syndrome is a “functional bowel disorder”. This means that there is no identifiable organic disease underlying the symptoms. The symptoms are a result of the abnormal functioning of an otherwise normal bowel.

It used to be described as a “diagnosis of exclusion”, however this term is not used any more. The idea was that this was a diagnosis that could only be made when other pathology had been excluded, however there has been a shift towards making a firm diagnosis based on clinical symptoms. Despite this, it is always important to keep other possible diagnoses in mind, particularly where there are red-flags such as rectal bleeding or a change in bowel habit in an older adults.

It is very common and occurs in up to 20% of the population. It affects women more than men and is more common in younger adults.

Symptoms
Diarrhoea
Constipation
Fluctuating bowel habit
Abdominal pain
Bloating
Worse after eating
Improved by opening bowels

Criteria for Diagnosis (NICE Guidelines)
Other pathology should be excluded:

Normal FBC, ESR and CRP blood tests
Faecal calprotectin negative to exclude inflammatory bowel disease
Negative coeliac disease serology (anti-TTG antibodies)
Cancer is not suspected or excluded if suspected

Symptoms should suggest IBS:

Abdominal pain / discomfort:

Relieved on opening bowels, or
Associated with a change in bowel habit
AND 2 of:

Abnormal stool passage
Bloating
Worse symptoms after eating
PR mucus

Management
Making a positive diagnosis and providing reassurance that there is no serious pathology present is important.

General healthy diet and exercise advice:

Adequate fluid intake
Regular small meals
Reduced processed foods
Limit caffeine and alcohol
Low “FODMAP” diet (ideally with dietician guidance)
Trial of probiotic supplements for 4 weeks

First Line Medication:

Loperamide for diarrhoea
Laxatives for constipation. Avoid lactulose as it can cause bloating. Linaclotide is a specialist laxative for patients with IBS not responding to first-line laxatives
Antispasmodics for cramps e.g. hyoscine butylbromide (Buscopan)

Second Line Medication:

Tricyclic antidepressants (i.e. amitriptyline 5-10mg at night)

Third Line Medication:

SSRIs antidepressants

Cognitive Behavioural Therapy (CBT) is also an option to help patients psychologically manage the condition and reduce distress associated with symptoms.

NICE published clinical guidelines on the diagnosis and management of irritable bowel syndrome (IBS) in 2008

The diagnosis of IBS should be considered if the patient has had the following for at least 6 months:
abdominal pain, and/or
bloating, and/or
change in bowel habit

A positive diagnosis of IBS should be made if the patient has abdominal pain relieved by defecation or associated with altered bowel frequency stool form, in addition to 2 of the following 4 symptoms:
altered stool passage (straining, urgency, incomplete evacuation)
abdominal bloating (more common in women than men), distension, tension or hardness
symptoms made worse by eating
passage of mucus

Features such as lethargy, nausea, backache and bladder symptoms may also support the diagnosis

Red flag features should be enquired about:
rectal bleeding
unexplained/unintentional weight loss
family history of bowel or ovarian cancer
onset after 60 years of age

Suggested primary care investigations are:
full blood count
ESR/CRP
coeliac disease screen (tissue transglutaminase antibodies).

The management of irritable bowel syndrome (IBS) is often difficult and varies considerably between patients. NICE updated it’s guidelines in 2015.

First-line pharmacological treatment - according to predominant symptom

pain: antispasmodic agents
constipation: laxatives but avoid lactulose
diarrhoea: loperamide is first-line

For patients with constipation who are not responding to conventional laxatives linaclotide may be considered, if:
optimal or maximum tolerated doses of previous laxatives from different classes have not helped and
they have had constipation for at least 12 months

Second-line pharmacological treatment
low-dose tricyclic antidepressants (e.g. amitriptyline 5-10 mg) are used in preference to selective serotonin reuptake inhibitors

Other management options
psychological interventions - if symptoms do not respond to pharmacological treatments after 12 months and who develop a continuing symptom profile (refractory IBS), consider referring for cognitive behavioural therapy, hypnotherapy or psychological therapy
complementary and alternative medicines: ‘do not encourage use of acupuncture or reflexology for the treatment of IBS’

General dietary advice
have regular meals and take time to eat
avoid missing meals or leaving long gaps between eating
drink at least 8 cups of fluid per day, especially water or other non-caffeinated drinks such as herbal teas
restrict tea and coffee to 3 cups per day
reduce intake of alcohol and fizzy drinks
consider limiting intake of high-fibre food (for example, wholemeal or high-fibre flour and breads, cereals high in bran, and whole grains such as brown rice)
reduce intake of ‘resistant starch’ often found in processed foods
limit fresh fruit to 3 portions per day
for diarrhoea, avoid sorbitol
for wind and bloating consider increasing intake of oats (for example, oat-based breakfast cereal or porridge) and linseeds (up to one tablespoon per day).

59
Q

Coeliac disease Dr Deac Pimp

A

Coeliac disease is an autoimmune condition where exposure to gluten causes an autoimmune reaction that causes inflammation in the small bowel. It usually develops in early childhood but can start at any age.

In coeliac disease, auto-antibodies are created in response to exposure to gluten that target the epithelial cells of the intestine and lead to inflammation. There are two antibodies to remember: anti-tissue transglutaminase (anti-TTG) and anti-endomysial (anti-EMA). These antibodies relate to disease activity and will rise with more active disease and may disappear with effective treatment.

Inflammation affects the small bowel, particularly the jejunum. It causes atrophy of the intestinal villi. The intestinal cells have villi on them that help with absorbing nutrients from the food passing through the intestine. The inflammation causes malabsorption of nutrients and the symptoms of the disease.

Presentation
Coeliac disease is often asymptomatic so have a low threshold for testing for coeliac disease in patients where it is suspected.

Failure to thrive in young children
Diarrhoea
Fatigue
Weight loss
Mouth ulcers
Anaemia secondary to iron, B12 or folate deficiency
Dermatitis herpetiformis (an itchy blistering skin rash typically on the abdomen)

Rarely coeliac disease can present with neurological symptoms:

Peripheral neuropathy
Cerebellar ataxia
Epilepsy

TOM TIP: Remember for your exams that we test all new cases of type 1 diabetes even if they don’t have symptoms as the conditions are often linked.

Genetic Associations
HLA-DQ2 gene (90%)
HLA-DQ8 gene

Auto-antibodies
Tissue transglutaminase antibodies (anti-TTG)
Endomysial antibodies (EMAs)
Deaminated gliadin peptides antibodies (anti-DGPs)

TOM TIP: Anti-TTG and anti-EMA antibodies are IgA. Some patients have an IgA deficiency. When you test for these antibodies, it is important to test for total Immunoglobulin A levels because if total IgA is low because they have an IgA deficiency then the coeliac test will be negative even when they have coeliacs. In this circumstance, you can test for the IgG version of anti-TTG or anti-EMA antibodies or simply do an endoscopy with biopsies.

Coeliac disease is an autoimmune condition caused by sensitivity to the protein gluten. It is thought to affect around 1% of the UK population. Repeated exposure leads to villous atrophy which in turn causes malabsorption. Conditions associated with coeliac disease include dermatitis herpetiformis (a vesicular, pruritic skin eruption) and autoimmune disorders (type 1 diabetes mellitus and autoimmune hepatitis). It is strongly associated with HLA-DQ2 (95% of patients) and HLA-DQ8 (80%).

In 2009 NICE issued guidelines on the investigation of coeliac disease. They suggest that the following patients should be screened for coeliac disease:

Signs and symptoms
Chronic or intermittent diarrhoea
Failure to thrive or faltering growth (in children)
Persistent or unexplained gastrointestinal symptoms including nausea and vomiting
Prolonged fatigue (‘tired all the time’)
Recurrent abdominal pain, cramping or distension
Sudden or unexpected weight loss
Unexplained iron-deficiency anaemia, or other unspecified anaemia
Conditions:
Autoimmune thyroid disease
Dermatitis herpetiformis
Irritable bowel syndrome
Type 1 diabetes
First-degree relatives (parents, siblings or children) with coeliac disease

Diagnosis
Investigations must be carried out whilst the patient remains on a diet containing gluten otherwise it may not be possible to detect antibodies or inflammation in the bowel.

Check total immunoglobulin A levels to exclude IgA deficiency before checking for coeliac disease-specific antibodies:

Raised anti-TTG antibodies (first choice)
Raised anti-endomysial antibodies

Endoscopy and intestinal biopsy show:

“Crypt hypertrophy”
“Villous atrophy”

Coeliac disease is caused by sensitivity to the protein gluten. Repeated exposure leads to villous atrophy which in turn causes malabsorption. Conditions associated with coeliac disease include dermatitis herpetiformis (a vesicular, pruritic skin eruption) and autoimmune disorders (type 1 diabetes mellitus and autoimmune hepatitis).

Diagnosis is made by a combination of immunology and jejunal biopsy. Villous atrophy and immunology normally reverses on a gluten-free diet.

NICE issued guidelines on the investigation of coeliac disease in 2009. If patients are already taking a gluten-free diet they should be asked, if possible, to reintroduce gluten for at least 6 weeks prior to testing.

Immunology
tissue transglutaminase (TTG) antibodies (IgA) are first-choice according to NICE
endomyseal antibody (IgA)
needed to look for selective IgA deficiency, which would give a false negative coeliac result
anti-gliadin antibody (IgA or IgG) tests are not recommended by NICE
anti-casein antibodies are also found in some patients

Duodenal biopsy*
villous atrophy
crypt hyperplasia
increase in intraepithelial lymphocytes
lamina propria infiltration with lymphocytes

Rectal gluten challenge has been described but is not widely used
Associations
Coeliac disease is associated with many other autoimmune conditions:

Type 1 Diabetes
Thyroid disease
Autoimmune hepatitis
Primary biliary cirrhosis
Primary sclerosing cholangitis
Complications of Untreated Coeliac Disease
Vitamin deficiency
Anaemia
Osteoporosis
Ulcerative jejunitis
Enteropathy-associated T-cell lymphoma (EATL) of the intestine
Non-Hodgkin lymphoma (NHL)
Small bowel adenocarcinoma (rare)

Treatment
Lifelong gluten-free diet is essentially curative. Relapse will occur on consuming gluten again. Checking coeliac antibodies can be helpful in monitoring the disease.

The management of coeliac disease involves a gluten-free diet. Gluten-containing cereals include:
wheat: bread, pasta, pastry
barley: beer
whisky is made using malted barley. Proteins such as gluten are however removed during the distillation process making it safe to drink for patients with coeliac disease
rye
oats
some patients with coeliac disease appear able to tolerate oats

Some notable foods which are gluten-free include:
rice
potatoes
corn (maize)

Tissue transglutaminase antibodies may be checked to check compliance with a gluten-free diet.

Immunisation
Patients with coeliac disease often have a degree of functional hyposplenism
For this reason, all patients with coeliac disease are offered the pneumococcal vaccine
Coeliac UK recommends that everyone with coeliac disease is vaccinated against pneumococcal infection and has a booster every 5 years
Currrent guidelines suggest giving the influenza vaccine on an individual basis.
Complications
anaemia: iron, folate and vitamin B12 deficiency (folate deficiency is more common than vitamin B12 deficiency in coeliac disease)
hyposplenism
osteoporosis, osteomalacia
lactose intolerance
enteropathy-associated T-cell lymphoma of small intestine
subfertility, unfavourable pregnancy outcomes
rare: oesophageal cancer, other malignancies

60
Q

Colorectal cancer Dr DEAC PIMP

A

Colorectal cancer is the third most common type of cancer in the UK and the second most cause of cancer deaths. Annually there are about 150,000 new cases diagnosed and 50,000 deaths from the disease.

Location of cancer (averages)
rectal: 40%
sigmoid: 30%
descending colon: 5%
transverse colon: 10%
ascending colon and caecum: 15%

Colorectal cancer: referral guidelines
NICE updated their referral guidelines in 2015. The following patients should be referred urgently (i.e. within 2 weeks) to colorectal services for investigation:
patients >= 40 years with unexplained weight loss AND abdominal pain
patients >= 50 years with unexplained rectal bleeding
patients >= 60 years with iron deficiency anaemia OR change in bowel habit
tests show occult blood in their faeces (see below)

An urgent referral (within 2 weeks) should be ‘considered’ if:
there is a rectal or abdominal mass
there is an unexplained anal mass or anal ulceration
patients < 50 years with rectal bleeding AND any of the following unexplained symptoms/findings:
-→ abdominal pain
-→ change in bowel habit
-→ weight loss
-→ iron deficiency anaemia

Faecal Occult Blood Testing (FOBT)

This was one of the main changes in 2015. Remember that the NHS now has a national screening programme offering screening every 2 years to all men and women aged 60 to 74 years. Patients aged over 74 years may request screening.

In addition FOBT should be offered to:
patients >= 50 years with unexplained abdominal pain OR weight loss
patients < 60 years with changes in their bowel habit OR iron deficiency anaemia
patients >= 60 years who have anaemia even in the absence of iron deficiency

Colorectal cancer: genetics
It is currently thought there are three types of colon cancer:
sporadic (95%)
hereditary non-polyposis colorectal carcinoma (HNPCC, 5%)
familial adenomatous polyposis (FAP, <1%)

Studies have shown that sporadic colon cancer may be due to a series of genetic mutations. For example, more than half of colon cancers show allelic loss of the APC gene. It is believed a further series of gene abnormalities e.g. activation of the K-ras oncogene, deletion of p53 and DCC tumour suppressor genes lead to invasive carcinoma

HNPCC, an autosomal dominant condition, is the most common form of inherited colon cancer. Around 90% of patients develop cancers, often of the proximal colon, which are usually poorly differentiated and highly aggressive. Currently seven mutations have been identified, which affect genes involved in DNA mismatch repair leading to microsatellite instability. The most common genes involved are:
MSH2 (60% of cases)
MLH1 (30%)

Patients with HNPCC are also at a higher risk of other cancers, with endometrial cancer being the next most common association, after colon cancer.

The Amsterdam criteria are sometimes used to aid diagnosis:
at least 3 family members with colon cancer
the cases span at least two generations
at least one case diagnosed before the age of 50 years

FAP is a rare autosomal dominant condition which leads to the formation of hundreds of polyps by the age of 30-40 years. Patients inevitably develop carcinoma. It is due to a mutation in a tumour suppressor gene called adenomatous polyposis coli gene (APC), located on chromosome 5. Genetic testing can be done by analysing DNA from a patient’s white blood cells. Patients generally have a total colectomy with ileo-anal pouch formation in their twenties.

Patients with FAP are also at risk from duodenal tumours. A variant of FAP called Gardner’s syndrome can also feature osteomas of the skull and mandible, retinal pigmentation, thyroid carcinoma and epidermoid cysts on the skin

Colorectal cancer: screening
Overview
most cancers develop from adenomatous polyps. Screening for colorectal cancer has been shown to reduce mortality by 16%
the NHS offers home-based, Faecal Immunochemical Test (FIT) screening to older adults
another type of screening is also being rolled out - a one-off flexible sigmoidoscopy

Faecal Immunochemical Test (FIT) screening

Key points
the NHS now has a national screening programme offering screening every 2 years to all men and women aged 60 to 74 years in England, 50 to 74 years in Scotland. Patients aged over 74 years may request screening
eligible patients are sent Faecal Immunochemical Test (FIT) tests through the post
a type of faecal occult blood (FOB) test which uses antibodies that specifically recognise human haemoglobin (Hb)
used to detect, and can quantify, the amount of human blood in a single stool sample
advantages over conventional FOB tests is that it only detects human haemoglobin, as opposed to animal haemoglobin ingested through diet
only one faecal sample is needed compared to the 2-3 for conventional FOB tests
whilst a numerical value is generated, this is not reported to the patient or GP, who will instead be informed if the test is normal or abnormal
patients with abnormal results are offered a colonoscopy

At colonoscopy, approximately:
5 out of 10 patients will have a normal exam
4 out of 10 patients will be found to have polyps which may be removed due to their premalignant potential
1 out of 10 patients will be found to have cancer

Flexible sigmoidoscopy screening

Key points
screening for bowel cancer using sigmoidoscopy is being rolled out as part of the NHS screening program
the aim (other than to detect asymptomatic cancers) is to allow the detection and treatment of polyps, reducing the future risk of colorectal cancer
this is being offered to people who are 55-years-old
NHS patient information leaflets refer to this as ‘bowel scope screening’
patients can self-refer for bowel screening with sigmoidoscopy up to the age of 60, if the offer of routine one-off screening at age 55 had not been taken up

Patients diagnosed as having colorectal cancer should be completely staged using CT of the chest/ abdomen and pelvis. Their entire colon should have been evaluated with colonoscopy or CT colonography. Patients whose tumours lie below the peritoneal reflection should have their mesorectum evaluated with MRI.

Once their staging is complete patients should be discussed within a dedicated colorectal MDT meeting and a treatment plan formulated.

Treatment of colonic cancer

Cancer of the colon is nearly always treated with surgery. Stents, surgical bypass and diversion stomas may all be used as palliative adjuncts. Resectional surgery is the only option for cure in patients with colon cancer. The procedure is tailored to the patient and the tumour location. The lymphatic drainage of the colon follows the arterial supply and therefore most resections are tailored around the resection of particular lymphatic chains (e.g. ileo-colic pedicle for right sided tumours). Some patients may have confounding factors that will govern the choice of procedure, for example a tumour in a patient from a HNPCC family may be better served with a panproctocolectomy rather than segmental resection.

Following resection the decision has to be made regarding restoration of continuity. For an anastomosis to heal the key technical factors include; adequate blood supply, mucosal apposition and no tissue tension. Surrounding sepsis, unstable patients and inexperienced surgeons may compromise these key principles and in such circumstances it may be safer to construct an end stoma rather than attempting an anastomosis.

When a colonic cancer presents with an obstructing lesion; the options are to either stent it or resect. In modern practice it is unusual to simply defunction a colonic tumour with a proximal loop stoma. This differs from the situation in the rectum (see below).
Following resection patients with risk factors for disease recurrence are usually offered chemotherapy, a combination of 5FU and oxaliplatin is common.

Treatment of rectal cancer

The management of rectal cancer is slightly different to that of colonic cancer. This reflects the rectum’s anatomical location and the challenges posed as a result. Tumours located in the rectum can be surgically resected with either an anterior resection or an abdomino-perineal excision of rectum (APER). The technical aspects governing the choice between these two procedures can be complex to appreciate and the main point to appreciate for the exam is that involvement of the sphincter complex or very low tumours require APER. In the rectum a 2cm distal clearance margin is required and this may also impact on the procedure chosen. In addition to excision of the rectal tube an integral part of the procedure is a meticulous dissection of the mesorectal fat and lymph nodes (total mesorectal excision/ TME).

In rectal cancer surgery involvement of the cirumferential resection margin carries a high risk of disease recurrence. Because the rectum is an extraperitoneal structure (until you remove it that is!) it is possible to irradiate it, something which cannot be offered for colonic tumours. This has a major impact in rectal cancer treatment and many patients will be offered neoadjuvent radiotherapy (both long and short course) prior to resectional surgery. Patients with T1 and 2 /N0 disease on imaging do not require irradiation and should proceed straight to surgery. Patients with T4 disease will typically have long course chemo radiotherapy. Those with T3 , N0 tumours may be offered short course radiotherapy prior to surgery. Patients presenting with large bowel obstruction from rectal cancer should not undergo resectional surgery without staging as primary treatment (very different from colonic cancer). This is because rectal surgery is more technically demanding, the anastomotic leak rate is higher and the danger of a positive resection margin in an unstaged patient is high. Therefore patients with obstructing rectal cancer should have a defunctioning loop colostomy.

Summary of procedures

The operations for cancer are segmental resections based on blood supply and lymphatic drainage.

In the emergency setting where the bowel has perforated the risk of an anastomosis is much greater, particularly when the anastomosis is colon-colon. In this situation an end colostomy is often safer and can be reversed later. When resection of the sigmoid colon is performed and an end colostomy is fashioned the operation is referred to as a Hartmans procedure. Whilst left sided resections are more risky, ileo-colic anastomoses are relatively safe even in the emergency setting and do not need to be defunctioned.

61
Q

Acute Pancreatitis Dr Deac Pimp from passmed

A

Acute pancreatitis is usually due to alcohol or gallstones.

Pathophysiology:
- autodigestion of pancreatic tissue by the pancreatic enzymes, leading to necrosis

Features:
Severe epigastric pain that may radiate through to the back
Vomiting is common
Examination may reveal tenderness, ileus and low-grade fever
Periumbilical discolouration (Cullen’s sign) and flank discolouration (Grey-Turner’s sign) is described but rare

Rare features associated with pancreatitis include:
ischaemic (Purtscher) retinopathy - may cause temporary or permanent blindness

Investigations:
raised amylase is seen in 75% of patients.

Scoring systems

There are several scoring systems used to identify cases of severe pancreatitis which may require intensive care management. These include the Ranson score, Glasgow score and APACHE II.

Some common factors indicating severe pancreatitis include:
age > 55 years
hypocalcaemia
hyperglycaemia
hypoxia
neutrophilia
elevated LDH and AST

Note that the actual amylase level is not of prognostic value.

Causes:
The vast majority of cases in the UK are caused by gallstones and alcohol.

Popular mnemonic is GET SMASHED
Gallstones
Ethanol
Trauma
Steroids
Mumps (other viruses include Coxsackie B)
Autoimmune (e.g. polyarteritis nodosa), Ascaris infection
Scorpion venom
Hypertriglyceridaemia, Hyperchylomicronaemia, Hypercalcaemia, Hypothermia
ERCP
Drugs (azathioprine, mesalazine*, didanosine, bendroflumethiazide, furosemide, pentamidine, steroids, sodium valproate)
*pancreatitis is 7 times more common in patients taking mesalazine than sulfasalazine

Management of Acute Pancreatitis in the UK

Diagnosis
Traditionally hyperamylasaemia has been utlilised with amylase being elevated three times the normal range.
However, amylase may give both false positive and negative results.
Serum lipase is both more sensitive and specific than serum amylase. It also has a longer half life.
Serum amylase levels do not correlate with disease severity.

Differential causes of hyperamylasaemia
Acute pancreatitis
Pancreatic pseudocyst
Mesenteric infarct
Perforated viscus
Acute cholecystitis
Diabetic ketoacidosis

Assessment of severity
Glasgow, Ranson scoring systems and APACHE II
Biochemical scoring e.g. using CRP

Features that may predict a severe attack within 48 hours of admission to hospital
Initial assessment	
Clinical impression of severity
Body mass index >30
Pleural effusion
APACHE score >8
24 hours after admission	
Clinical impression of severity
APACHE II >8
Glasgow score of 3 or more
Persisting multiple organ failure
CRP>150

48 hours after admission
Glasgow Score of >3
CRP >150
Persisting or progressive organ failure

Table adapted from UK guidelines for management of acute pancreatitis. GUT 2005, 54 suppl III

Management

Nutrition
There is reasonable evidence to suggest that the use of enteral nutrition does not worsen the outcome in pancreatitis
Most trials to date were underpowered to demonstrate a conclusive benefit.
The rationale behind feeding is that it helps to prevent bacterial translocation from the gut, thereby contributing to the development of infected pancreatic necrosis.

Use of antibiotic therapy
Many UK surgeons administer antibiotics to patients with acute pancreatitis.
A recent Cochrane review highlights the potential benefits of administering Imipenem to patients with established pancreatic necrosis in the hope of averting the progression to infection.
There are concerns that the administration of antibiotics in mild attacks of pancreatitis will not affect outcome and may contribute to antibiotic resistance and increase the risks of antibiotic associated diarrhoea.

Surgery
Patients with acute pancreatitis due to gallstones should undergo early cholecystectomy.
Patients with obstructed biliary system due to stones should undergo early ERCP.
Patients who fail to settle with necrosis and have worsening organ dysfunction may require debridement, fine needle aspiration is still used by some.
Patients with infected necrosis should undergo either radiological drainage or surgical necrosectomy. The choice of procedure depends upon local expertise.

62
Q

Chronic pancreatitis Dr deac pimp

A

Chronic pancreatitis is an inflammatory condition which can ultimately affect both the exocrine and endocrine functions of the pancreas. Around 80% of cases are due to alcohol excess with up to 20% of cases being unexplained.

Other than alcohol, causes include:
genetic: cystic fibrosis, haemochromatosis
ductal obstruction: tumours, stones, structural abnormalities including pancreas divisum and annular pancreas

Features
pain is typically worse 15 to 30 minutes following a meal
steatorrhoea: symptoms of pancreatic insufficiency usually develop between 5 and 25 years after the onset of pain
diabetes mellitus develops in the majority of patients. It typically occurs more than 20 years after symptom begin

Investigation
abdominal x-ray shows pancreatic calcification in 30% of cases
CT is more sensitive at detecting pancreatic calcification. Sensitivity is 80%, specificity is 85%
functional tests: faecal elastase may be used to assess exocrine function if imaging inconclusive

Management
pancreatic enzyme supplements
analgesia
antioxidants: limited evidence base - one study suggests benefit in early disease